Anda di halaman 1dari 321

LIMIT, CONTINUITY AND THEORY OF EQUATIONS

by narayanadash
With Love For Family, Yogmaya, Sriram And Sudha
Preface

The book contains fundamentals of Limit, Continuity, and Theory

Of Equations for +2 level in CBSE, ICSE boards and councils in

India and equivalent courses abroad. Theory Of Equations are

dealt in more detail in a separate volume under differentiation but

quite a lot of it is in this book. The concepts of theory of equations

(including Des’Cartes law of signs) have been integrated with limit

and continuity. The concept of differentiation and numerical

evaluation of roots of polynomials have been introduced though


they are better taught after differentiation.some aspect of minima

and maxima and asymptotes are also discussed (which does not

require the concept of differentiation). Besides fundamental

treatment, the book is designed to also cover fundamentals for

IITJEE and other engineering entrance examinations .Most of the

problems are either solved or with suggestions sufficient for self

study. The book is with an intention to build confidence in the

student along with laying ground work.

Mistakes in the book, and viewing problems may be mailed to the

author at nyn_3@rediffmail.com and shall be received with

gratitude. Suggestions for improvement are also most cordially

invited. The author is completely at your disposal for any help in

understanding the theory or any of the problems for free; but a


week’s time is required for response. The book has been verified

with a good preview with e-ink paper-white. Please report any

viewing problems at once.

Sincerely,

Narayanadash, author.
TABLE OF CONTENTS

Table of contents
Chapter 1:The concept of Limitsof a function Part I
§1.1:Concept of Limit:
§1.2:Extending the concept of limit; Limit When x  
§1.3:A further extension of concept of limit; Infinite limits:
§1.4:Algebra of limits
§1.5:Some Important Limits and methods of finding them:
A) Exponential series.
B) Hint on preparing a log table:
§1.6:Many facets of the definition of limit
A) An example :a limit that does not exist anywhere : true meaning of definition of limit.
B) Remember the following five results.
c) Do the following curves approximate st lines at large distances?
d) Function of many variables
§1.7:Order of smallness and order of largeness , comparison of infinitesimals:
§1.8:Equivalent infinitesimals used in calculation of limits
§1.9:Notes on infinitesimal and Hyperreal numbers.
A) Order completeness of real numbers
B) Infinitesimals
Chapter 2 : The Concept Of Limit Of A Function Part II
Chapter 3:Continuity And Theory Of Equations
§3.1:The Concept of Continuity.
§3.2: A further result in Algebra of limits: (limit of a composite function)
§3.3:Removable and non-removable discontinuity; discontinuity of 2nd kind.
§3.4:A little more about real number sets and sequences., limit points and bounded sequences
§3.5:Properties of continuous functions
§3.6:Further Properties Of Continuous Functions
A) Intermediate value theorem

B) When the function f  x  is continuous in [a, b] and f  a  . f  b  < 0, there is a root of f  x  in [a, b].
C) When the function f  x  is continuous in [a, b] and f  a   f  b  , f  x  takes every value between f  a 

and f  b  .

D) When the function f  x  is continuous in [a, b] and Maxf  x   M , min f  x   m the function takes every
value between M and m . In fact, [M, m] is the range of the function.
E) Wavy curve method to analyse an algebraic function
§3.7:Monotonic functions and continuity of inverse functions.
§3.8:Continuous linear operators
Chapter 4: Further theory OF EQUATIONS AND POLYNOMIALS
§4.1:Theory of equations ( polynomial equations)
§4.2:The division algorithm,
Remainder theorem and Synthetic Division
§4.3:Evaluation of a polynomial and other functions
§4.4:Synthetic division (Due to Horner).
§4.5: Prove that a polynomial of n-th degree has n roots,

§4.6:A polynomial of degree n, f  x   p0 x  p1 x


n 1
n
 ............. pn1x  pn cannot have more than n roots.

If a polynomial of degree n, f  x   p0 x  p1 x
n 1
§4.7:
n
 ............. pn1x  pn has more than n roots it must be an
identity, i.e., it must be identically 0.
§4.8:Show that a polynomial of odd degree with real coefficients has at least one real root.

§4.9:Every polynomial of even degree p2n  x   a0 x  a1x2n1  ...........  a2n  0 whose absolute term is negative ,
2n

has at least two real roots.

§4.10:Take any polynomial p2n  x   a0 x  a1x2n1  ...........  a2n  0 of even degree. If it attains at least one value
2n

, the sign of which is opposite of that of a0, then show that it has at least two real roots.

§4.11: If a polynomial p  x  , of only odd powers of x has all its coefficients of same sign, it has no other real root
except x = 0.

§4.12: If a polynomial p  x  , of only even powers of x has all its coefficients of same sign, it has no real root.

§4.13:Descartes’ Rule Of Signs.

§4.14:The number of negative roots in a polynomial of n-th degree f  x   0 cannot exceed the number of changes

in sign in f   x 

§4.15:If a  b is a root of polynomial equation f  x   0 , then a  b is one root too.

§4.16:Just in the same manner, show that if a  ib is a root of polynomial equation f  x   0 , then a  ib is one root
too.
§4.17:Except conjugate surds no other irrational (transcendental) number can be a root of a polynomial.
§4.18: Rational root theorem :
§4.19:Integer roots as a special case:
§4.20: Location theorem
.§4.21: Evaluation of root of a polynomial
§4.22: The following points may be carefully noted for testing whether a given number is a root.

§4.23:Horner’s method for evaluating f  x  h  from f  x 

§4.24: Relation between roots and coefficients.


§4.24a: Relation between roots and coefficients may be used to find out roots of a polynomial when one of the
roots is known or under some given condition
§4.25: Upper bound and lower bound theorem
§4.26: Transformations of equations
§4.27:Reciprocal equation
§4.28:Project; Solving Cubic equation by removing second term
§4.29:Symmetric functions of roots
Chapter 5: MORE ABOUT THEORY OF EQUATIONS, andROOTS BY NUMERICAL METHODS
§5.1: Extended synthetic division:
§5.2: Sums of powers of roots and other symmetric functions of roots.
§5.3: Multiple roots :
§5.4: Roots of transcendental functions
To prove: ei = cos  + i sin :
§5.5: de’ Moivre’s Theorem : (cos  + i sin )n = cos n + i sin n)
§5.6: All the roots of polynomial xn + xn – 1 + xn – 2 +……………+1:or xm – 1 = 0 : the problem of inscribing a regular n-gon
inside a circle.
§5.7: Comparison with number system:
§5.8: Area of a circle :
§5..9: Approximate Roots and Fixed Points of Functions : Picard’s Theorem :
§5.10: Approximate Roots Newton’s method or method of tangents
§5.11: Approximate roots by Secant method
CHAPTER 1:THE CONCEPT OF LIMITSOF A FUNCTION

PART I

§1.1:Concept of Limit:

In the figure 2. 1. above, for all values of x less than 1 we have f(x) = y = x i.e., when x is

very near to 1 remaining less than 1 at the same time, y is also near 1. Similarly when x

is greater than 1, but very near to 1 at the same time, y = 2 – x is also near to 1. So y is
nearing 1 from either side when x is near to 1. We say this fact in Calculus language as “

limit” of y is equal to 1 when x approaches 1.No matter whether f(1) is same as that limit

or different ; even function at that point may not have been defined. The limit does not

depend upon the value of the function at the point in question. For example you can chose f(1) =4

which very different from the limiting value lim y  lim f  x   1 .This is a way of writng “the limit of
x 1 x 1

function f is 1 when x tends to 1, in symbols.

A function f(x) is said to tend to a limit ‘L’ at x = a, if the value of f(x) can be made

arbitrarily near to L, if x is chosen sufficiently near ‘a’ [ f(a) may or may not be defined).

In symbolic notation, Lim


x a
f(x) = L. (The word ‘arbitrarily’ is stressed upon because,it means we

may choose values of x as near to 1 as we please,x = 0.999,x = 1.0001, x = 0.9999,x = 1.00001, x =

0.99999,x = 1.000001,etc.)

In the above example, (fig.2.1) , we can make 1 – f(x) <  if we take the values of x such that 1 – x

<  as long as x <1;

(for ,y = x for all x < 1) .Thus we write Lim f(x) = 1 or say, f(x) tends to 1 from left or from
x  (1  0)

below or say left handed limit of the function at x = 1. Similarly we can see that we can make

f(x) –1 <  if we choose the values of x such that x – 1 <  ( as f(x) = 2 – x for all x more than
1 but less than 2). We write this as Lim f(x) =1 and say that the function f(x) tends to 1
x  (1  0)

from right or from above or say the right handed limit of f(x) at x = 1 is 1. Imagine what would have

been the result if our function had been defined as f(x) = 3 – x, instead of 2 – x for x values above

2; the right handed limit would have been 2 instead of being 1.We could define f(1) = 2; no

restriction on us.

We say that the limit of a function f(x) exists at a point x = a if the left-handed and the right-

handed limits both exist and are equal, otherwise we say the limit does not exist.(The definition of

limit should be unequivocal and unambiguous, otherwise which one should we choose for our limit?)

Both the above statements of criteria for a limit of f(x) at x = a to be ‘L’ may be combined

in a single statement:

Lim
x 1
f(x) = L for every  such that  f(x) – L  <  , however small, we can find a  such that

x – a <  Only one case of failure would ascertain non-existence of limit of f(x) at x = a .

(Two symbols ‘ L’ and ‘a’ are per chance same ( 1 each) in the example taken, they need not be

the same in general).Note also the triangle inequality [ p < q  –p<q< p ] for

understanding the combined statement.


The phrase ‘however small’ needs some explanation. Different people have different standards of

smallness. For example a grain of sugar may be negligible for a sugar merchant, but it is more than

own weight of an ant. The order of smallness or negligibility can be different for different purposes

too. For example, a small grain of copper sulphate is by no means negligible for spectroscopic

investigation. So, for Mathematics, the standard of smallness must be universal, for all people and for

all purposes; therefore the word ‘arbitrarily small’. ( do not confuse this with order of smallness ,a

totally different concept.)

Note : ε and δ may be thought of as small positive numbers unless otherwise stated.

x2  9
EXAMPLE 1: Find
Lim y , if y
x3 x3

Straight putting x = 3 renders the expression in 0/0 form and is useless. One cannot

substitute x = 3 direct here. A great scientist Einstein had said that a problem is solved as

soon as it is correctly known. So let us take x = 3 +  where the latter is a variable small

number near zero which can be made as small as we please .(Imagine this symbol  to be a

box which encloses the arbitrarily small variable , we need not bother about the variable, we could

take the box anywhere we please easily and toss it away mercilessly at last, when we could

do so. This is the beauty of using symbols exactly where difficulty is met with) .
x2  9 ( 3  ) 2  9 9  6   2  9
 6
x3 =
3 3 

The last  can be discarded at last , then the answer = 6. So our arbitrarily small number (+ve or

–ve) has been enclosed within the symbol  and we have chosen no standards of smallness to

be neglected; this can be made as small as one pleases.

The objective of this example is to show that , in working out problems, we may cancel out common

factors from numerator and denominator (for sake of short cut, for , while taking limits, we may

replace zero factors by non-zero infinitesimals, even if the functional value is undefined, as there is

no dependence of limit on functional value). This is by substituting the factor 0 by h, an

infinitesimal. But care must be taken to see that there is no division by 0. See the note below.

NOTE :

Division by 0 is an impossible operation, for example, if you divide a basket containing ten apples

to a number of people giving 0 to each, an infinite number of people you may count. But this is not a

division, since the basket of apples is never exhausted in the process. This is in contrast to the fact

0 2
that dividend 20 is exhausted in the process if divided by 6, casting a quotient  . Division of 0 by
0 3
0 2 0 4
0 also not possible since  is obtained from 2.0  3.0 ( which is true)and  is also obtained
0 3 0 7

0
from 7.0  4.0 ( which is true again). Thus is not unique, and in fact arbitrary. So this is no division;
0

20 2
where as  3 is definitely unique. The process of division must end up with exhausting the
6 3

dividend and unique dividend.

x2  9
y
Now, instead of tossing away x3 by just saying that x = 3 does not belong to the domain of

the function, we are just scavenging for its possible value by assuming x  3  h or x  3  h , taking h

to be very small nonzero values. In the above example, we observe that in both cases x  3  h or

x2  9
 6h
x  3 h , x 3 . In the last step, we may toss away h to get the limiting value of the function =

6.

Again, if the limit of y for x  3  h is different from the limit of y for x  3  h , we do not accept the limit

and say that the limit does not exist. The two limits are referred to as Right handed limit RHL and

left handed limit LHL. The limit exists only if the LHL and RHL are same. It does not exist when

one or both of RHL and LHL do not exist or they are unequal. The whole concept of limit centers
around this idea. A function f  x  is said to be continuous at x  a only if f  a  is same as the limit

lim f  x  is same as f  a  . This is dealt in the next chapter.


x a

lim h3  3
x 3
Example2 Find h

In this example, we cannot substitute 0 by h direct, as may be done in a variety of problems, since t

0
involves division by 0. Some manipulation must be done to avoid . This is done by rationalizing
0

the denominator. See below.

lim h3  3
 lim
 h3  3  h3  3  lim h 33

1

1
   
x 3 x 3 x 3
h h h3  3 h h3  3 33  3 3 3

lim h3  3
x 0
Example3 Find h

lim h  3  3 lim
 x 0
 h3  3  h3  3  lim h 33

1

1
   
x 0 x 0
h h h3  3 h h3  3 03  3 2 3
There is a special significance of this problem. This limit is rate of change of the function h near 0.

Example4

Find
ax  a
. Ans.
1
. Hint :Rationalize the exp. lim
 ax  a  ax  a 
 
lim
x 0 x 2 a x 0
x ax  a

x 1
 lim 
x 0
x  ax  a  2 a

EXAMPLE5 : Find limit of f(x) = x2 /x2 at x = 0,

The function is identically 1 for all values of x  1 and is not defined for x =

2
Lim x
1.The latter fact does not prevent us from taking limit at x = 1. So, x 1 2 1
x

For this reason note here that the functions f(x) = x2 /x2 and f(x) =1 are different functions. Are their

domains different?

x
EXAMPLE 6 : Let y  .We have x  x for x  0 and x  x for x  0  y = 1 for x  0 and y
x

= -1 for x  0  that the left handed limit is -1 and the right handed limit is 1 when

x  0 . Hence lim
x0
y does not exist . (If x1 and x2 are any two points in the opposite

sides of 0, we have f(x1) – f(x2) = 2, which cannot be made arbitrarily small, say < .3
whatever small interval for x1 – x2 we may choose. If we assume y1  y 2 < .03,in which

interval around x=0 which is not possible . Evidently we cannot get a  for this case).

EXAMPLE7 : Consider the step function y = [x] ,the greatest integer not greater than x .

In the interval [2 , 3[ the value of the function is constant and = 2; but at x = 3, y = 3.So

the function’s value suddenly increased by 1, for virtually no increase of x. In other words, if

we imagine arbitrarily y to increase only by 0.01 around x = 3, we find no small interval

around x = 3, in which the function increases only by 0.01.For, however small interval we take

around x = 3, the function increases by 1.(Note also that Lim


x( 20)
y 2 whereas Lim
x ( 2 0 ) y  3 and

are different and hence the limit does not exist.)

EXAMPLE 8 : Can you construct another step function? Try y = [tan x] . Point out the points

where limit does not exist.

(Ans. x = tan-1 2, x = tan-1 3, x = tan-1 3 etc)

EXAMPLE 9 : Examine limit of sin(1/x) when x tends to 0.

When x tends to 0, 1/x becomes larger and larger and sin(1/x) takes all possible values

from -1 to 1. Hence no limit exists in this case.

EXAMPLE 10 : Examine limit of sin(1/x) when x tends to 

When . x tends to  , 1/x tends to 0 and sin(1/x) tends to 0 accordingly.


EXAMPLE 11 : If x= 1/y, x as y 0.An interesting outcome of this trivial result is as under:

A quadratic equation ax2 +bx +c =0, is supposed to have two roots anyway. Suppose c = 0, the

b 1
case gives, one root x = 0, and the other  . Putting it in a different way, let x  ; so the equation
a y

becomes a  by  cy 2  0 .If c = 0, the equation virtually ceases to be quadratic as it becomes

a  by  0. y 2  0 . But still we cling to the nomenclature and call it trivially quadratic. Repeating the step

again, if we put b = 0, the equation becomes ax 2  0 so that both its roots become 0. but this reduces

the equation to a = 0. We can still continue to call it a trivial quadratic equation.

Note:

It is not always that we can not substitute the value of the variable direct for calculating the

0  0 
limit. If the expression does not have indeterminate forms such as , ,0 ,1 etc, we could
0 

direct substitute the value of variable to get the limit . It is only when the expression involves

indeterminate forms we resort to methods of avoiding indeterminate forms , such as

employing an infinitesimal ( є, δ definition), rationalizing, L’Hospital’s rule and other various

tools.

For example lim  3x 2  2   3*22  2  3*4  2  10 , as it does not involve indeterminate forms.
x 2
§1.2:Extending the concept of limit; Limit When x 

There may be limits of functions even when the independent variable tends to infinity, particularly

when the independent variable appears in the denominator. A minor modification must be made in

case the function f(x) tends to a definite number L when x tends to  .

For any arbitrary ε , however small, we can find a number M, sufficiently

large, such that for all x, such that |x| > M , we get | f(x) – L | < ε true.

For example, f(x) = 1/x tends to 0 as x tends to  . For |1/x – 0| to be less than 0.00025,

we can find a number 4000 such that , for all x, such that |x| > 4000, we have |1/x –

0| < 0.00025.

You might think that the definition should have included something like |x - ∞| < δ . The phrase |x| > M

means the very same idea in a presentable manner.

§1.3:A further extension of concept of limit; Infinite limits:


In a different situation, if f(x) tends to  as x tends to ‘a’, then for any large positive

number M ,such that , we can find a sufficiently small number  such that |f(x)| > M for

all x such that |x-a|<  . .

(Please do not confuse here if you are a student. You are advised not to confuse anywhere

in fact. If at all you do, leave the point, proceed appreciably further ,then return to a point

prior to the point of confusion and proceed again leisurely . Either your confusion shall

dissolve or you may be able to find mistake/s at that point ! )

Infinite limits are only nominally limits as we do not have much to do with infinity.

Lim k
EXAMPLE 12 : Examine the x x when i) k > 0, ii) k = 0, iii) k < 0. i) When k >

1
0, for any large number M, xk > M  x > Mk . So , for any arbitrary large number M we

1
can find a sufficiently large number N = Mk such that xk > M for all x > N. So in this case,

Lim k
x x = 

ii) When k = 0 , xk = 0 identically, for all x > 0 . So Lim k


x x = 0.

iii) When k < 0, -k > 0  when xk = 1/ x-k <   x-k > 1 /  


x >(1/  )-k = N say, so , for any  such that , xk <  , we can find an N such that x >

Lim k
N. i.e., we can make xk arbitrarily smaller by choosing N sufficiently large.  x x = 0 for

k< 0 .

EXAMPLE13: Examine Lim


p  zp for cases i) z >1, ii) z = 1and iii) |z| < 1

Lim
i) for z > 1, zp increases indefinitely as p increases indefinitely. Hence p zp = 

Lim
ii) for z = 1, zp = 1 for whatever p may be. Hence p zp = 1

iii) for |z| < 1, let z = 1/y so that 1/y is either greater than 1 or less than - 1.Then zp or

Lim
(1/y)p decreases to 0 through positive or negative values. Hence p zp = 0.

lim tan  . We know from its graph that tan θ = p/b and p increases
EXAMPLE 14: Find 

2

indefinitely and b decreases at the same time so that , for any large number M we can find a δ such

that |π/2 – θ| < δ.

Though the concept of limit is intuitive, it evolved through centuries since Leibniz and Newton

developed Calculus independently, even before that. It did not take shape until the last century and

the present form of definition was contributed by Cauchy and finally Weisstras formulated it. The
concept is so fundamental that it is at the root of all other concepts of Calculus like differentiation,

integration etc.

The concept itself does not give any method or clue to calculate any limit. But if we guess a limit

intuitively or by direct substitution, the definition only confirms whether the number is truly the limit

or not. So far we have had many typical examples and methods of finding limits as given above the

most common method , of course is a cautious guess by direct substitution and by practice only, you

would gain confidence of calculating limits. After the chapter on differentiation, a more formal method

0 
due to L’Hopital would be introduced with wider applicability in , ,1 ,  0 and 0 etc. forms . For
0 

which standard technique of differentiation of numerator and denominator is used, such as

rationalization as mentioned above, or changing the variables, considering order of smallness (

explained towards the end of this chapter) or L’ Hospital’s rule ( explained after differentiation

chapter,).

§1.4:Algebra of limits

Infinitesimals are themselves not of any use (Are they?). But infinitesimally increasing or

decreasing sequences, if have limits, the limits are important and throughout Calculus we are to

only study these limits. So we need Algebra of limits to work with limits.(Proofs or details
may be omitted at first reading ;the results are obvious and useful for working out

problems)

Limits of functions behave like algebraic numbers in a manner (provided they exist) that if the

functions are added, limit of the addition function is addition of their limits and such other

results as follows:

(a)If Lim f(x) = L and Lim g(x) = m, Lim (f(x) + g(x)) = L+ m = Lim f(x) + Lim g(x)
x a x a x a x a x a

Lim f(x) = L and Lim g(x) = m, for each 1 and  2 such that │f(x) - L│< 1 and │g(x) - m │< 2
x a x a

we can find  1 and  2 such that │x - a│<  1 and │x - a│< 2 or can find a  where  =

min ( 1, 2) such that │x - a│< 

Or for │f(x) – L + g(x) – m │ ≤ │ │f(x) – L │+│ g(x) – m │ │ ≤ 1 + 2 we can find a  such

that │x - a│<  . .And  should be arbitrarily be small; so we may choose 1 = 2

= ε /2 imposing no restrictions on it arbitrariness.

Hence for every ε arbitrarily small, such that │(f(x) – g(x)) –( L + m) │ ≤ ε we can find a  such

that │x - a│< 

That proves result (a) of Algebra of limits.


(b) If Lim f(x) = L , Lim (p . f (x)) = p . L = p . Lim f(x)
x a x a x a

If Lim f(x) = L , for each 1 such that │f(x) - L│< 1 we can find  1 such │x - a│< 1
x a

Or for each │ p│. 1 such that │p │. │f(x) -L│ < │p │. 1 we can find 1 such

│x - a│< 1 (multiplying by a positive quantity does not change the direction of inequality.

Or for each  such that │p . f(x) – p. L│ < │p │. 1 we can find  1 such │x - a│<  1

where we can choose 1 =  / │p │.

Or for each  such that │p . f(x) – p .L│ <  we can find  = 1 such │x - a│<  setting 

= │p │. 1 and  = 1 which does not restrict any arbitrariness. (-ve value of p does not

affect │p │ , and for p = 0 the inequality holds good trivially)

(c) If Lim f(x) = L and Lim g(x) = m, Lim (f(x) - g(x)) = L - m (to prove)
x a x a x a

Setting p = - 1 in result (b) above, we get Lim ((-1) . g(x)) = ( -1) . m or, Lim -g(x) = - m
x a x a

And putting -g(x) in place of g(x) in result (a) above, we get, Lim ( f(x) – g(x) )
x a

= Lim ( f(x) + (- g(x) ) ) = Lim f(x) + Lim (- g(x)) = Lim f(x) - Lim g(x) = L – m(d)
x a x a x a x a x a
d) If Lim f(x) = L and Lim g(x) = m, Lim (f(x) x g(x)) = L x m (to prove)
x a x a x a

If Lim f(x) = L and Lim g(x) = m, for any 1 and 2 such that, for │ f(x) - L │< 1 and
x a x a

│ g(x) - m│ < 2 ,we can find 1 and 2 such that │x - a│< 1 and │x - a│< 2

or a  such that  = min( 1 2) where │x - a│< 

Or │( f(x) – L ) (g(x) - m) + m . ( f(x) - L) + L.( (g(x) - m) ) = │ f(x)g(x) - m f(x) - L g(x)

+ Lm + m f(x) - Lm + L g(x) - Lm │

= │ f(x)g(x) - Lm │ < │( f(x) - L) (g(x) - m) │ + │m . ( f(x) - L) │ + │L.( (g(x) - m) ) │ <

1 2 + m 1 + L 2 for │x - a│< 

For any arbitrary  we can choose 1 and 2 such that 1 2 + m 1 + L 2 <  where we can

find a  such that │x - a│<  (proved)

1
(e) If Lim g(x) = m, Lim = 1/ m (to prove) if m  0
x a x a g( x )

Let us prove this by disproving its contradiction .If Lim 1/ g(x) is not 1/ m let
x a

Lim 1/ g(x) = r Utilizing the result (d) Lim ( g(x) (1/g(x) ) = m .r , not equal to 1.
x a x a
But identically, Lim ( g(x) (1/g(x) ) = Lim 1 = 1 (Identically means ,independent of x or g(x)) That
x a x a

proves m .r must be = 1, i.e., r = 1/m.

(f) If Lim f(x) = L and Lim g(x) = m, Lim (f(x)  g(x)) = L  m (to prove) if m  0 Lim (f(x)
x a x a xa x a

 g(x)) =(f(x) x 1/ g(x)) = Lim f(x) x Lim 1/ g(x) ( by result (d)


x a x a

= Lim f(x) x 1 / Lim g(x) = L/m


x a x a

x g( x )
( In the example y  if we set y = f(x), g(x)  x and h(x) = x, so that y  f ( x )  ,
x h( x )

g( x )
Lim
x 0 g(x)  0 xLim
0 h(x) but
Lim
x0 does not exist. The situation arises because division by 0 is not
h( x )

allowed. But nobody prevents us direct calculating the limit which is earlier shown to be 1)

x3  4 x 2  3 13  4.12  3 2
Lim
x 1  
Example x2  6 12  6 7

(g) Lim g(f(x)) = g(f(a)) = g( Lim f(x) ) i f Lim f(x) exists at all. (Without
x a x a x a

proof, for details vide next section)

(h) If the functions f, g, and h in the same domain are such that
f(x) ≤ g(x) ≤ h(x) for all values of x and if

Lim f(x) = L = Lim h(x) . and Lim g(x) exists, then Lim g(x) = L (Without proof)
x a x a x a x a

Some call it Squeezing Theorem or Sandwich Theorem. The result is quite simple to

understand and is useful for working out problems. You can also work out a sketch proof by taking

Lim g(x) = M  L and then arriving at a contradiction. is to show that limits of rational functions can
x a

be found by substitution provided the denominator is not zero.

Note : Triangle inequality

Before proving these results the following inequalities may be useful

|p+q| ≤ |p| +|q| ……………………………………………....(1)

(Triangle inequality ;Sum of two sides of a triangle is greater than the third.)

| p.q| = │p│. │q│…………………………………..…………. (2)

p.│q │ ≤ │p .q │…………………………………………..…..(3)

│ |p|-|q|│≤ |p-q|………………..…………….………………..(4)

(Difference of two sides of a triangle is less than the third. This is deducible from (1);
put p + q = a, p = a – q.  │a │ ≤ │ a - q │ + │ q │ ;  │a │ - │ q │ ≤ │ a - q │  │ │a

│ - │ q │ │ ≤ │ a - q │ )remember

Example16:

an
Show that for a > 0, lim  0.
n n !

Hint : If the limit exists, it must be  0 , being sum of all positive terms. Now take a positive integer m

such that m  a  m  1.

a n a.a.......m times a.a.......  n  m  times am a a a a a


Now  .  . . ...  n  m  times , as  
n! m!  m  1 m  2  .....upto n m! m  1 m  1 m  3 m  2 m 1

etc.

nm nm
am  a  am  a  a
The latter is .  . For any m, is constant, but lim    0 as  1 . Hence the
m!  m  1  m! n 
 m 1  m 1

result.

The general results given below would serve as examples to illustrate the methods of limit

calculation.
Extend the algebra of limits to any finite number of sums, and products of functions step by step.

§1.5:Some Important Limits and methods of finding them:

Remember the results

a) Find Lim [ xn – a n ] / ( x – a )
x a

= Lim [ xn – 1 + xn – 2 a + xn – 3 a 2 + xn – 4 a 3 ….+… + an – 1,, n terms] (By straight division)


x a

= [an – 1 + an – 2 a + an – 3 a 2 + an – 4 a 3 ….+…. + an – 1,, n terms] = n .an – 1

It is worth observing that limits of polynomials are found by substitution. ( in substituting, what we

tacitly do is cancel 0’s from numerator and denominator replacing 0 by h →0.)

Alternatively, we can replace x by a + h where h→0,

xn  an (a  h ) n  a n
Lim  Lim
x a x  a h 0 h

= Lim [ an +n. an - 1 .h + n(n – 1 ) an - 2 . h2 / 2! +…..….h n - an ] / h (by Binomial Theorem)= n .


h0

an – 1 neglecting h and its higher powers


sin  tan 
Lim  1  Lim
b)  0   0  remember

In the figure1.13, an arc of radius OP = perpendicular drawn on OP meet OQ at S. We

have OQ = r . Let the perpendicular from P on OQ meets OQ at R and

PR ≤ arc PQ ≤ PS  PR / r ≤ arc PQ / r ≤ PS /  sin θ ≤ θ ≤ tan θ

 sin  ≤ θ / θ ≤ tan   sin 


≤ 1 ≤
tan 
( PR / r and PS / r both tend to 1 as θ
   

tend to 0) see the graph below


sin 1 x tan 1 x sin  x  a  tan  x  a 
A variant of the result is often useful . lim  1  lim , lim  1  lim .
x 0 x x 0 x xa xa x a xa
sin  tan 
So Lim = 1 = Lim
0  0 

sin x cos x sin x 1


Compare this with lim  lim  0 .For, 0  lim  lim  0 . Remember
x  x x  x x  x x  x

Exercise1

sin x
Show that lim  1. Hint : x   x for x < 0.Remember
x 0 x

example 17
tan x sin x sin x tan x
a) Which of the two limits exist, lim  and lim 
x 0 x x x 0 x x

tan x sin x tan x sin x


Ans. Since 1,  1 , so,   0 . Hence the former exists.
x x x x

b)

tan 1 x sin 1 x sin 1 x tan 1 x


c) Which of the two limits exist, lim  and lim 
x 0 x x x 0 x x

tan 1 x sin 1 x tan x sin x


Ans. Since  1,  1 , so,   0 . Hence the latter exists.
x x x x

cos.x -1
c) Lim
x 0 =0
x

remember

Lim cos.x  1 cos.x  cos 0 Lim


x
2sin sin
x
x 0  Lim  x 0 2 2 1 x  1.1.0  0
x 0
x x x x
. 22
2 2

d) The exponential Series: remember

nx
 1
Lim  1+  = 1+ x + x2 / 2! + x3 / 3! + x4 / 4! + x5 / 5! ..upto  = ex
n ∞  n

[ The symbol e stands for Napier’s number as introduced in Chapter 1.and the

series is called exponential series]

The Napier’s number, the exponential series, the logarithmic series introduced below are

enormously important in the field of Algebra, Trigonometry, Coordinate Geometry, Calculus, Real and

Complex analysis, Physics, etc. The results involving the exponential and logarithmic series are

extremely useful in series summations, preparing logarithmic table etc. It has given Mathematics,

tremendous insight and practical utilities.

1 nx
Lim (1  ) = Lim [1 + nx.1/n +nx(nx-1)/2!. (1/n2) + nx(nx-1)(nx-2)/3! . (1/n3)+..up to ]
n  n n 

( by Binomial Theorem)

= Lim [ 1 + x + x( x - 1/n) / 2! + x(x - 1/n)(x - 2/n) / 3! .. +………up to 


n 

= 1+ x + x2 / 2! + x3 / 3! + x4 / 4! + x5 / 5! ......up to  neglecting 1/n

.1/n2 etc. as n  .

Similarly, Lim ( 1 + 1/n ) n = 1 + 1 + 1/2! + 1/3! + 1/4! ....................up to  is denoted by e


n 

(Napier’s number) by putting x = 1. The series definitely converges to a finite limit


x
 1
nx
 1  n 
which is a transcendental number between 2.71 and 2.80So Lim 1   = Lim  1    =
n ∞  n n ∞  n  



x
 1  n 
Lim  1    = ex
n ∞  n  


Hence ex = 1+ x + x2 / 2! + x3 / 3! + x4 / 4! + x5 / 5! ......up to  which is called the

A) Exponential series. remember

Again putting x = -1 in the series for ex , we get, e-1 = 1 - 1 + 1/2! - 1/3! + 1/4!

....................up to 

Logarithms to the base e are called natural logarithms .In this manner, log e a is written as ln a . e)

Lim ( x + 1/x )x = e
x 

The result has already been proved for x being any positive integer in the previous section.

If x is not an integer, we can find an +ve integer n such that

1 1 1 1 1 1 1 n 1 1
n  x  n 1     1  1   1   (1  )  (1  ) x  (1  ) n 1
n 1 x n n 1 x n n 1 x n

1 n 1 1 n 1
(1  ) (1  )
 n 1 1 1
 (1  ) x  (1  ) n 1 
Lim n 1
1
1 x n n   1 1
n 1 n 1
Lim 1 Lim 1
 (1  ) x  (1  ) n 1 for , n  ...when...x  
x x n n

Lim 1 n 1
(1  )
 n n 1 Lim 1 Lim 1 Lim 1 e Lim 1
 (1  ) x  (1  ) n (1  )   (1  ) n  e.1
Lim 1 x x n n n n 1 n n
1
n   n 1

Lim 1
 (1  ) x  e
n x

n
 x
1    e
Lim
f) n
 n

n
 x  x n(n  1) x 2 
1    1  n  .......upto... 
Lim Lim
n  n
 n
2
 n 2! n 

 x2  1  
 Lim
n 1  x  1    ..........upto... 
 2!  n  

x2 x3
 1 x   .......... upto...  e neglecting 1/n and its higher powers.
2! 3!
x a x a
 1
x a
Lim  1  1  Lim  1
1    e We have, , x   1   xLim 1   .x   1    e.1  e
Lim
g) Prove x   
 x  x  x  x

x .a x
Lim  1  a
x   1    ea  Lim
x  1  
h) Prove
 x  x

a
Lim  1
x .a
  1 x 
  Lim  1   
a
 Lim  1  x 
x   1  x    x  1     e a
We have,
 x  x  
  x  

x
x a
Lim  a  Lim  a a
a
 x

Again, x   1   x 1     a 
 x 1     ea
Lim
a
.
 x  a   x  a   x  
 

h2
1  h  .... upto  1
Lim e  1 Lim
h
 h 0 2!
i) h0 = 1 neglecting h and higher powers.
h h

Lim e
ha
1 1  ha 
h2 a 2
....upto  1 eha -1
Similarly, h 0  Lim
h 0
2! a i.e.,
Lim
h 0 a
h h h

remember
j) ax = exln a, For let ln a = y  ey = a  ax = (ey)x = (ex)y =exy=exln a

a x  1 Lim e x ln a  1
Lim
x 0  x 0 .ln a  1.ln a  ln a if a > 0, Similarly, if a < 0, -a > 0,
x x ln a

(a) x  1 Lim e x ln(  a )  1


Lim
x 0  x0 .ln(a)  1.ln(a)  ln( a) .So in either case,
x x ln(a)

a x  1 Lim e x ln|a|  1
Lim
x 0  x0 .ln | a |  1.ln | a | ln | a | So,
Lim
x 0
a x -1
 ln | a |
x x ln | a | x

abx  1 Lim ebx ln|a|  1


Similarly,
Lim
x 0  x 0 .b.ln | a |  1.b.ln | a | b ln | a |
x bx ln | a |

 , if a 1

Remember that lim a   1 x
if a 1
x 
0 0  a 1
 if
ln(1  x ) ln(1  x) Lim 1
Lim
x 0  1. , We Lim
 x0 ln(1  x) x  ln Lim
1
k) have x 0 (1  x)  ln.e  1.
x
x 0
x x

l)The logarithmic series:

ln (1+x)= loge(1+x) = x – x2/2 + x3/3 – x4/4……….up to . remember

The series converges for x  1

y 2 (ln a)2
To derive it, note that a
y
e y ln a
 1  y ln a 
2!

y 3 (ln a)3 y 4 (ln a)4


  ......upto....
3! 4!

y ln(1 x ) y 2{ln(1  x)}2


Putting a = 1+x , (1  x)  e
y
 1  y ln a 
2!

y 3{ln(1  x)}3 y 4{ln(1  x)}4


  ....upto 
3! 4!
y ( y  1) 2 y ( y  1)( y  2) 3
But by Binomial theorem, for x < 1, (1  x ) y
 1  yx  x  x
2! 3!

y ( y  1)( y  2)( y  3) 4
 x .....upto....
4!

which is just another way of expressing the above series, and identical with the first one. Hence,

equating the coefficient of y from both the expressions, we get,

(1) 2 (1)(2) 3
ln(1  x)  x  x  x 
(1)(2)(3) 4
x .........upto ,
1.2 1.2.3 1.2.3.4

x 2 x3 x 4
or, ln(1  x)  x    ......upto........
2 3 4

1
or, ln(1  ) 1 1 1 1
 2  3  4 .....upto....
n n 2n 3n 4n
 n 1 1
 ln    ln(n  1)  ln n and ln(1  )   1  1 2  1 3  1 4 ....upto....
 n  n n 2n 3n 4n

 n 1 
 ln    ln(n  1)  ln n
 n 

Subtracting the latter series from the former, we get,

ln(n  1)  ln(n  1)  2  1  1 2  1 3 ....upto..  which is a rapidly converging series.


n 3n 5n 

Putting n = 3 and using ln 1 = 0, we get ln 4 – ln 2 = ln (4/2) = ln 2. Putting n=9, we get ln 10 – ln 8

and ln 8 = 3 ln 2, we get ln 10. once we get ln 10, we can find log x , (i.e., log 10 x, it is customarily

written in Arithmetic as log10x).

B) Hint on preparing a log table:

The logarithmic series provides the clue to prepare a log table, starting at ln 1 =0 and choosing

incremental values of x. But the series is not rapidly converging ,i.e., we cannot have good

approximation values of logarithms of numbers by summing only a few terms from the beginning. But

the logarithmic series provides us designing other series not only for preparing a log table but

opening up a number of possibilities in other areas.

Exercise2

Show that lim x 2  x  1  1
x 1

Exercise3

sin x
Show that lim 0 remember
x  x

sin x 1 1
Hint ; Make  sin x. 
x x x

Exercise4

cos2 x
lim
Find x x .Ans. 0.

Exercise5

sin 2 x  sin 2 a
lim
x2  a 2 sin 2 x  sin 2 a  sin  x  a  sin  x  a 
Find xa . Ans. 1 Hint :

x  sin x  sin x 
lim x 1  
x  x  cos2 x x  sin x  x 
Exercise6:Find .Ans. lim  lim 1
x  x  cos2 x x  cos 2 x 
x 1  
 x 

lim2 x sin 2 x
Exercise7:Find x .
0  lim2 x sin 2 x  lim2 x lim2 x  0 lim 2 x sin 2 x  0
, as 0  sin 2  1 .But
x
Ans. : We have x x x , so x .

1 1

Exercise8Find lim  log3 3x  lim  log3 3x 


1
log3 x
. Ans.
log3 x  lim  log3 3  log3 x  log 3x
x1 x 1 x 1

1
 lim 1  log3 x 
1
lim log3 x .
log3 x e x1 log3 x
 e1  e
x 1

c
lim e x sin
Exercise9:Find
x  ex .

c
sin
c e x sin ce x
c lim e sin x  c lim
x
 c lim c
sin 0 x  e x  1
c x
x  ce  x

Ans. Since ex as x→∞, we have e

x 1
lim 2
Exercise10: Show that
x1 x 1

lim 1  x  1  x 2 1  x3  ............ 1  x n  


1
Exercise11:For |x| <1 show that
n  1 x
Lim c sin x  x sin c
x c
Exercise12: Find cx .

Lim c sin x  x sin c Lim c sin x  x sin x  x sin x  x sin c


x c x c
We have cx = cx =

sin x  sin c sin x  sin c


 Lim
x c sin x  x c xx c
Lim Lim
 sin c  cxLim  sin c  c cos c
cx c
xc

  cos2 x 
lim sin  2 
Exercise13:Find
x 0
 x .

Hint :

sin  cos 2 x  sin    sin 2 x  sin  sin 2 x  sin 2 x


lim  lim  lim lim 
x 0 x2 x 0 x2 x 0  sin 2 x x 0 x2

1
Lim sin x
x 0
Example 17 : Find x . Put sin-1x = θ, so that x = sin θ, We have θ  0 when sin

θ  0.

1
Lim sin x  1 1
x 0  xLim
0   1
x sin  Lim sin  1
 0
or x  0. Now, 
Exercise14:

 sin x   sin x   sin  h     sinh 


lim    lim  1   0
 x  . x  h0  h   h0 h 
lim  lim
Find
x0
Hint :
x0
 , as sinh  h also

 sin x 
lim   0
x 0 
 x  .

lim  tan 1 x   1 lim  tan 1 x   2


Exercise15: Show that x but x Hint :

 
 1     lim 
 tan 1
x 
      2
lim tan x     1 , but x 
 2
x 
 
 2
.

 x2 
Exercise16: Show that x0 
lim 0.
 sin x tan x 

lim 1  x   x  1  1  x  1
Exercise17: Show that x1
 tan x   tan x  tanh tan   h  tanh
lim    1  lim   1  1
Exercise18: Show that  x  x 0 
x  0 
 x  . Hint tan x  x , so h , also h h

1  sin 2 x
lim
x 4x  
Exercise19: Show that 4 does not exist.

1  sin 2 x 1  sin 2 x 1  sin 2 x 1  sin 2 x 1


Hint : lim  lim lim 1
x
 4x   x

 4 x    1  sin 2 x x   4 x    1  sin 2 x 1  sin 2 x

4 4 4

1  sin 2 x 1 1
lim  lim 1  sin 2 x lim  1.lim 1
x  4 x    x x  4 x    x  4 x   
lim
x 4x   
Now 4 4 4 4 . But 4 does not exist.

 sin  x    sin x 
lim   lim  0
Exercise20: Show that
x 0
  x 
does not exist, but
x0
 x 
.

Exercise21:
 1  n 1n  22  n 2n  32  ...  n  m  1n  m2 
Show that lim lim  1
m  n   m2  2
 

  1
n
 2
n
 m 1 
n

 1  2    1  3    1  ......  m 
2 2
 1 
  lim 1  2  3  ...  m  lim m  m  1  1
n n n

2 3  m 
Hint : lim lim 
m  n   m
2
 m m2 m 2 2
 
 

Lim e
ax
 e bx
x 0 ab
Example 19 : Prove x remember

eax  ebx
Lim
We have x 0
 Lim eax  1 Lim ebx  1
 x 0  a b
x x 0
x x

Example 20 :

Lim
f ( x)  a0 x n  a1 x n 1  ........  an
Examine x f ( x ) where and , b0  0
b0 x m  b1 x m1  ........  bm remember
a1 a
x n (a 0   .....  nn )
a 0 x n  a1x n 1  ........  a n x  x n  m g( x ), say
 x
f (x)  b b
x m ( b0  1  ....  m
b0 x m  b1x m 1  ........  b m x xm
)
We have,

a0 a0
Lim
x   g( x )  Lim
Lim
x   g( x ) x   x
Lim nm
 00
Now,
b0 ;So, if n<m, x
f ( x ) =
b0

nm a0 a
Lim
Lim Lim
x   g( x ) x   x  .1  0
if n=m, 
x f (x) = b0 b0

if n> m, it would be  or -  according as a0/b0 is +ve or –ve.

an
Lim
x 0 g( x) 
Similarly
bn .

Exercise22
ax 2  b
lim a  0 . Hint
Find x  x  c when : If a < 0, by above formula, limit is - ∞. If a = 0,

0.x 2  b
lim 0
x  x  c

Exercise23

 x2  1 
lim   ax  b    .
 x 1
Find all values of a and b if x 

 x2  1   x 2 1  a    a  b  x  1  b 
Hint : Since lim   ax  b   lim     and degree of numerator > degree
x 
 x 1  x   x 1 

1 a
of denominator, then  0  a  1 . The b could be any real number.
1

 x2  1 
If lim   ax  b   0 , find all values of a and b.
exercise24
x 
 x 1 
 x 2 1  a    a  b  x  1  b 
Hint : lim    0 and the degree of numerator > degree of denominator, and
x
 x  1 

this is only possible only when the degree of numerator < degree of denominator.

Both the statements are possible only when 1  a  0 and a  b  0 , i.e., only when a = 1 and b = 1 this

 1 b 
becomes lim 
x  x  1
0
 

Exercise25 remember

 x2  1 
If lim   ax  b   c  0 , find all values of a and b.
x 
 x 1 

 x2  1 
Hint : Since x  
lim  ax  b   c  0  lim  x 1  a    a  b  x  1  b   c and the degree
2

x  
 x 1   x 1 

of numerator should have been equal to the degree of the

denominator. This is possible only when 1  a  0  a  1 .


 1  b  x  1  b  1 b
Now, putting a =1 in the limit we get, lim    c . But the limit is , so
x
 x 1  1

1 b  c  b  1 c .

Example 21 :

Lim cos nx cos nx 1


Find n , n  N We have, 0  cos nx  1  0   So
n n n

Lim cos nx
n   0, as, n  
n remember

Exercise26

1
e 1
x

Evaluate lim f  x  if
f  x  1
e 1
x 0 x

1 1
e h 1 1/ e h  1
1 1
1
Hint : lim f  x   lim f  0  h   lim 1
 lim 1
 1 , as h  0     e    1 / e h  0
h
x 0 h0 h 0 h 0 h
e h
1 1/ e h  1

1 1
e h  1 1  1/ e h
1 1
1
Now, lim f  x   lim f  0  h   lim 1  1
 1 , h  0     e h    1 / e h  0 So left
x 0 h0 h 0 h
e h  1 1  1/ e h

handed limit  right handed limit.


Exercise27

1  cos2  x  c 
Show that the lim does not exist.
xc xc

1  cos 2  x  c  1  cos 2  c  h  c  1  cos 2h sinh


Hint : lim  lim   lim   2 lim  2
x c  xc h 0 chc h 0 h h 0 h

1  cos 2  x  c  1  cos 2  c  h  c  1  cos 2h sinh


But lim  lim  lim  2 lim  2 . So no limit.
x c  xc h 0 chc h 0 h h 0 h

Lim e
x
 cos x
Example 22 : Find x  0
x2

 x4 x6   x2 x4 
1  x    .......
x2
Lim e  cos x 2
1    ......
x 0 Lim  2! 3!  Lim  2! 4! 
We have, x2 = x 0 2 x 0 2
x x

 1
x 2 1    higher.. powers..of ..x 2
 2 1 3
 1 
2
x 2 2

Example23
lim sin  sin 1 x  
Find x 1 . Ans. : 0. Note that RHL is not required as the function is not

defined for x > 1.

Example24

Show that lim cot x does not exist . Hint : Show LHL = ∞, and RHL = - ∞. So left handed limit 
x0

right handed limit. Mark it as a technique.

Example 25

Show that lim cot 1 x   0 . Hint take cot 1 x  u


x 

Example 26 : Immediate ‘rate of change’ of a function f(x) is given by

Lim f ( x  x )  f ( x )
x0 and is called differential coefficient of the function at any point x,
x

where

δx is difference in x between two points x and x + δx .

If potential function in a gravitational field is given by V(r)= - GM/r, find the field strength E=-

space rate of potential.

r  (r   r )
GM GM
 
We have, E   r 0
Lim r r r  GM Lim (r   r )r  GM Lim 1 GM
 2
 r 0 r 0
r r (r   r )r r
Differential coefficient , the next volume, thus may be seen as an application of Limit

calculation.

§1.6:Many facets of the definition of limit

a) Whether target values are controllable

A function may be regarded as the output of an operation on the independent variable which may be

regarded as input. For example mixing Ammonia and Hydrochloric acid would produce Ammonium

Chloride gas. The rate of reaction depends upon concentrations of inputs and of course,

circumstantial variables like temperature, pressure etc. in other words, The volume of Ammonium

Chloride at any time t , say, V(t) is a function F, say, of concentration of Ammonia CA, that of

Hydrochloric acid CH, temperature T, pressure P of the system . In symbols, V(t) = F(C A,CH, P, T, t),

where F is a function of many variables or may be a set of functions, different for different ranges of

temperature. At any particular time t1, with given values of CH, P and T, V(t1) is a function of CA. If we

set a target value L , can we be able to control the input CA near a control value CA1 ? In other
words, can we control the output to be within a range of prescribed error ε i.e. (VA(t1) – ε < L < VA(t1)

+ ε) by controlling the input in a range within (CA1 – δ < L < CA1 + δ) ?. If this is so, we say that V(t1)

has a limit L at the point CA = CA1. If you closely observe the meaning, you would understand that the

actual output does not depend upon this L, which is a result of Mathematical considerations only; it

may be different, or altogether may be absent (when suppose, the factory is under lock up) .

Otherwise, when this L is equal to VA(t1), we understand that the production process is smooth or

continuous, a concept dealt in the next chapter.

b) For every ε we need a δ

Take the example y  f  x   2 x2  3 and we are asked to find lim f  x  . We guess a limit L at the point
x 5

x = 5, by direct substitution , 53 and make back calculation for finding δ for every ε, such that

2 x 2  3  53   .Proceed backwards as 2 x 2  50   , or , 50    2 x 2  50   ,

50   50   50   50  
or , x , or , 5  x 5  5
2 2 2 2

50   50  
We can take bigger of the two numbers 5 and  5 as a δ. We need not look
2 2

for best possible δ but any workable δ would do.


To find a δ is thus solving a problem of inequality. Conversely, if we do not find a δ for any one

ε, the limit does not exist.

Take an example : show that the step function y = [x] , the greatest integer not greater than x does

not have a limit a x = 3. Observe that for values of x such as 2  x  3 , y = 2 and for x = 3, y = 3. Take

an ε = 0.1 and we try to put all values of y between 2.4 to 2.6 and observe that there is no δ, which

can make [x] lie between 2.4 to 2.6 as it suddenly changes from 2 to 3 at x = 3.

A) An example :a limit that does not exist anywhere : true meaning of

definition of limit.

 1 when x is rational
Try a function f  x    for investigating limit at a rational point. Obviously the
0 when x is irrational

function oscillates too much there and has no limit. You may remember that there

is a rational number between any two irrational numbers and there is an irrational number between

any two rational numbers.

But for every δ > 0 such that x  a   some ε > 0 for which f  x   L   , say , for example, ε = 1.1.

The definition of limit is certainly different from this consideration; it says for every
ε (such that f  x   L   ) we can find a δ (such that x  a   ), and not the other way round. So the

statement that lim


x a f  x   L for any δ > 0 such that x  a   we can find an ε

such that f  x   L   , ε > 0 does not assure existence of limit at x = a.

So also the statement that for any ε > 0 such that f  x   L   we can find an x near ‘a’ is not

sufficient for guarantying existence of a limit.

 1 when x is rational
Note : The function f  x    is called characteristic function of rational
0 when x is irrational

numbers. It simply separates the rational numbers from irrationals simply by attaching

different labels 1 and 0 to them. It is also called Dirichlet function after the mathematician Dirichlet .

But the reverse of the above statements is always true. If an ε can not be found so that f  x  does not

lie near L, i.e. L    f  x   L   is not true or equivalently f  x   L   is not

true for when x  a   , for some δ, then L is not the limit.

d) Sequences and limits

On first time reading , falsity of the above two statements may appear strange . For any δ we are

finding an ε and still the limit may not exist ! But we see that it is so. Do we have to always check is
backwards whether a suspected limit is actually so and there is no direct method. In fact there is one,

but of a little practical use. See how it is.

The independent variable x may tend to a in many ways, actually in infinite number of ways.

For example if x→0, we may imagine it is along the sequence

1 1 1 1 1 1 1 1 1 1 1 1
, , , ......... n , n1 .........infinite terms or , , , ......... n , n1 .
2 4 8 16 2 2 2  1 4  1 8  1 16  1 2 1 2 1

........infinite terms and through infinite number of such sequences as you can see to it. And similarly x

→ a through infinite number of sequences converging to ‘a’. It may be assured that if all the

sequences f  x1  , f  x2  , f  x3  , f  x4  ,......infinite terms converge to L for every sequence

x1 , x2 , x3 , x4 ,......................infinite terms converging to ‘a’ then lim f  x   L . In other words, the


x a

function f  x   L along every path of x leading to ‘a’. Can you practically test a limiting value in

this manner ? How many such paths can you take into consideration ? But there is one practical utility

in this development. If you want to disprove that a number L is not the limit of a function f(x) as x → a,

you can present a sequence x1 , x2 , x3 , x4 ,......................infinite terms along which lim f  x   L . Only one
x a

example is sufficient to disprove a proposition. But before giving examples for such a method , it

should be shown that the two definitions of limit are equivalent – the ε - δ method and the sequence

method, the definition is after the mathematician Heine.


Before that let us see a little closely the meaning of convergence of a sequence. A sequence

x1 , x2 , x3 , x4 ,...........xn ...........infinite terms is said to converge to a limit ‘a’ if for every positive δ, we

can find an integer M such that |x – a| < δ for all n > M. The definition is for any sequence ,e.g.

the sequence f  x1  , f  x2  , f  x3  ...... f  xn  ... ....infinite terms is said to tend to the limit L if for any ε > 0,

we can find a positive integer N such that f  xn  L    for all n > N.

Now the equivalence of Heine definition of limit and that due to Cauchy – Weisstras may be easily

seen to be equivalent, Starting from Heine’s definition , for any ε > 0, for f  x  L    we can find a

sequence f  xn  such that f  xn  L    assures a sequence xn such that |x – a| < δ, for some δ ,

for some integer > N. Thus it leads to Cauchy – Weisstras definition. Then starting from the Cauchy –

Weisstras definition the Heine’s definition follows easily, for, if f  x   L , it tends to the limit through

every path or through every sequence xn leading to ‘a’.

2x 1
example27: To find lim in sequence method.
xa 3 x  2

2 x1  1 2 x2  1 2 x3  1
The limit would be limit of the sequence , , ,............ as n   , i.e. xn  a
3 x1  2 3 x2  2 3 x3  2

.So
2 xn  1 xlim
n a
 2 xn  1 2a  1
lim  
xn  a 3 x  2
n lim 
xn  a
3 xn  2  3a  1 . (But this method is of little practical use; direct

substitution method is preferable).

example28: To show that lim sin x does not exist.


x 

To evaluate the limit, assuming it exists, let us evaluate it along the path or sequence xn  n and in

 1
another way, along the sequence yn   2n    .
 2

Now, along the path xn  n , we have lim sin x  lim sin n  lim 0  0 .
x  n n

 1  1
Now, along the path yn   2n    , we have lim sin x  lim sin  2n     lim1  1 .
 2 x  n
 2 n

Since the two limits are different, the limit does not exist.

Misc exercises

1
28. Show that the limit lim sin does not exist. Hint see above example.
x 1 x 1

29. Show that the limit lim cos x does not exist. Hint see above example.
x 

 1
 n , for n  2m  1
30. Find the limit of the sequence xn  
 n , for n  2m
 n  2
1 1 2 1 3 1 4 1 n 1
Hint : The sequence is precisely 1, , , , , , , ................ . Now lim  0 , But lim  .
2 3 3 5 5 7 5 n  n n  n2 2

Thus the limit is not unique hence does not exist.

 1
1  n1 , for n  2m  1
 22
31. Find the limit of the sequence xn   . Ans. No limit exists.
 1 , for n  2m
 n2
 2

an
32. For any positive number ‘a’, show that lim 0
n n !
remember

an a a a a
Hint : Take any natural number m  2a . Then , for n > m, we have,  . . ............
n! 1 2 3 n

nm
a a a a a a a a 1 m 1 
  . . ......   . . ......   a m     2a   n 
1 2 3 m   m 1 m  2 n  3 n 2 2 

 1  an
Now  n   0 . So lim  0.
2  n n !

33. Find lim n n . Remember


n
n  n  1 2
n  1  xn  n  1  xn   n  1  nxn 
n
Hint : Since n
n  1, take n
xn  .....  xn n , xn  0 ,
2!

n  n  1 2 n  n  1 2 nx 2 2
 n  1 xn  n  1  xn  1  n  xn  . (As n >1)
2! 2! 2 n

2 2
As lim  0 , lim n n  1  lim 1
n n n n n

1
34. Find lim a for a > 0. Hint : lim a  lim a  a 0  1 .
n n n
Remember
n n n

35. Find lim


n!
. Hint : Take xn 
n!
, then xn1 
 n  1! so that
xn 1

 n  1! / n!  .
 n  1  n  1 nn
n n n n 1 n 1
nn xn

 n  1! . nn  n! n  1 . nn   n  1 . nn  nn 1
 n  1 n!  n  1 n!  n  1 n  1 1  n  1
n 1 n 1 n n

Thus xn1  xn . The sequence is bounded below and has a limit L say.

 n  1
n
 n 1   1 
n n
1 nn 1 1
Now    1  n   1  n. n  ....  2 , so   xn1  xn . As both
 n     n  1
n n
n 2 2

1
lim xn1  L, lim xn  L , So L  L , also 0  L . Thus L =0
n n 2

n2  n  1
2
13  23  33  ...  n3 1
36. Show that lim  . Hint : 1  2  3  ...  n 
3 3 3 3
. Divide numerator
n  1
n 2
2 4 4

and denominator by n 4 .
B) Remember the following five results.

a x  bx
37. Show that lim  1 if a  b  1 . Hint : Divide both numerator and denominator by a x .
x  a x - b x

a x  bx
38. Show that lim x x  -1 if b  a  1 . Hint : Divide both numerator and denominator by b x .
x  a - b

a x  bx
39. Show that lim  1 if 1  a  b . Hint : Divide both numerator and denominator by a x .
x  a x - b x

a x  bx
40. Show that lim x x  -1 if 1  b  a . Hint : Divide both numerator and denominator by b x .
x  a - b

41. Take n in place of x in above 4 consecutive questions and prove the results again, n +ve

integer.

42. Show that lim 1.n  2.  n2  12  3.2 n  2  2....  n.1  1 .


n 1  2  3  ...  n 2

1.n  2.  n 1  3.  n  2  ....  n.1   n  1  r   r 2 


n n
 n  1   n  1 2n  1 as
2
Hint : the
2 6

general term is r  n  r  1   n  1 r  r 2
n n
1.n  2.  n  1  3.  n  2   ...  n.1 2  n  1  6  n  1 2n  1
2

Now lim 
n  12  22  32  ...  n2 n
 n  1 2n  1
6

n n
 n  1   n  1 2n  1 3  n  1
2

2 6 3n  3  2n  1 n  2 1
 1    as n   .
n
 n  1 2n  1  2n  1 2n  1 2n  1 2
6

43. Find lim


 n  2 !  n  1! Ans.: 1 Hint :  n  2 !   n  1! n  1 .
n  n  2 !  n  1!

44. Find the limit as n   of the sequence a , a  a , a  a  a ..............n terms .

45.

Hint : Let the lim xn  a  a  a.  ... n terms  L . Squaring L2  a  L  L 


n

1  12  4.1.a 1  1  4a
 .
2 2

Since a > 0, and a sequence of positive terms is positive, the minus sign is avoided. So

1  1  4a
L .
2

Of course the sequence is bounded; that is to be proved at first to ensure existence of a finite limit if

at all. If a  0 ,
an upper bound is aa  a 2 , for a  a, a  a  a  a  a 2, a  a  a

 a  a 2  a 1  2  2a and so on a  a  a.  ... n terms  2a .

46. Prove lim xn  2  2  2  2... n terms  2


n 

an   1
n
2am  1 2a  1/ m a
47. lim lim  lim 
bn   1 . Ans. If n is even, n = 2m, 2bm  1 2b  1/ m
n  n m  
Find m b.

2am  2a  1 2a   2a  1 / m a
 lim lim 
When n is odd, say n = 2m+1, m  2bm  2b  1 m  2b   2b  1 / m b also.

1 1
lim 
 sin x 
n 1 n
2 2
48. If , then find all the values of x.

1 1
lim 
 sin x  2 lim  sin 1 x   0
n n
n 1
2
Ans. Since , n .
1
This is possible only when sin x is a proper fraction, i.e., 1  sin 1 x  1  x   1,1

d) Do the following curves approximate st lines at large distances?

a) Show that for some a and b, remember

 x2  1  x2  1 1 x x2  1
lim  - ax - b   0 , Hint.  x , Show that lim  lim  ax  b  for a = 1, b = -1.
x 
 x 1  x 1 1 x x  x  1 x 

Evidently y  ax  b is a st line.

 1

b) lim
x - 
 
x 2 - x  1 - ax - b  0 , 
x  
 1 1
 x x 

Hint lim x 1   2   ax  b 
2


 

  1

  1 1 2   11 1  
 lim x 1    2   ax  b   lim x
 1    2  ...  ax  b   0,
x   
x x    2 x x  
x 
 

1
for a  1, b 
2

1
 1 1 2
(expanding the binomial   2  and neglecting higher powers)
x x 
x2  1 1
So the curves y  and y  x 2  x  1 are st lines y  x  1 and y  x  when x   or x  
x 1 2

respectively.

Those st lines are called asymptotes of the concerned curves.

Example28 :

Prove that lim sin x does not exist.


x 

  
Choose a sequence xn  2n  so that lim sin xn  lim sin  2n    1
2 xn  xn 
 2

Choose another sequence xn  n so that lim sin xn  lim sin  n   0


xn  xn 

Since the limits are different in two different paths, the limit does not exist.

Exercise48

a) Show that lim cos x does not exist.


x 

b) Show that lim tan x does not exist


x 

The use of Heine’ definition is not only useful for proving non-existence of limits. Take the following

example.

x 1
lim
Example29 : Find the limit x 3 2 x  3
x 1
f  x 
Let us consider any sequence xx converging to 3 in the domain of definition of 2 x  3 i.e.

3
xn  
avoiding any 2 which evidently is not in domain of f  x  , for, it makes f  x  infinite.

xn  1 x  1 x2  1 x3  1 x 1
Now consider the sequence , i.e., 1 , , ...... n .....infinite terms
2 xn  3 2 x1  3 2 x2  3 2 x3  3 2 xn  3

3 1 4
Clearly the limit of this sequence is  , which is a constant , i.e. independent of choice of xn.
63 9

x 1 4
So, lim  . The Heine’s definition also gives us method of testing a limit. ( you can see it is no
x 3 2x  3 9

x 1
better than method of direct substitution in f  x   ).
2x  3

Exercise49

  
Find lim sin 1 x  . Hint : lim sin 1 x   lim t      1 , putting sin 1 x  t so that t  as x  1
x 1 x1 t
2
2 2

tan 1 x  sin 1 x
exercise 50 : Find lim .
x 0 sin 3 x

x
1 1
tan 1 x  tan 1
tan x  sin x 1  x2
lim  lim
x 0 sin 3 x x 0 sin 3 x
x3 lim 3
x 0 x
 x 
 x 
tan 1  1  x2   x 1  x2  x 
 1  x.x  tan 1   x 1  x2  x x 1  x2  x
   x2  1  x2 
 1  x2   lim   lim x 2  1  x 2  1.lim x 2  1  x 2
ans.  lim
x 0 x 3 .1 x 0
x 1  x2  x x 0 x3 x 0 x3
x2  1  x2

1  x2 1 1  x2  1 1
 lim 
x 0
x2  1  x2 1  x2  1 2
x2

x a b
Note that sin 1 x  tan 1 and tan 1 a  tan 1 b  tan 1
1 x 2 1  ab

d) Function of many variables

But definitely the Heine’s definition gives good theoretical insight into the problem of existence of

limits. It will help understanding the path concept we have introduced informally in case of function of

two variables z    x, y  . The domain of such functions is entire xy plane or some part of it. And the

graph of the function gives us a surface in a three dimensional (x, y, z) space as y  f  x  gives us a

curve in two dimensional plane. For example x 2  y 2  a 2 gives us the surface of a cylinder of radius a

standing on the xy plane. Another example, z  x 2  y 2 cot  gives us a cone with vertex at the origin
and half vertical angle α. And so on. We can extend the definition of limit for such functions as

follows:

A function z    x, y  approaches to a limit L as  x, y    x0 , y0  if, for ε > 0 such that x  x0  

  x, y   L   , we have a δ such that  x  x0    x  x0    , or equivalently x  x0   and


2 2

y  y0   .

The Heine’s definition takes the shape as A function z    x, y  approaches to a limit L as

 x, y    x0 , y0  if, it approaches to L along every path y  f  x  in the domain of definition of z

and vice versa.

It is easy to understand that a curve y  f  x  is a path in xy plane and if a function z of two variables

x and y has to have a limit at (x0, y0), it has to have that limit though every path leading to the point

(x0, y0).

§1.7:Order of smallness and order of largeness , comparison of

infinitesimals:
So far we are talking about ε and δ tacitly assuming them small positive numbers , but when it come

to a formal statement or definition , we are evidently avoiding the word ‘small’ ; on the pretext that

definitions and tools of Mathematics are independent of ‘order of smallness’ which may vary from

person to person or vary as purposes behind. Remember that a grain of rice is negligible for a grain

merchant but never for an artist who want to inscribe a whole book on it. But mathematical

formulations should be of universal application and should not be for the grain merchant alone nor for

the above artist. Let us closely look at what is meant by order of smallness and then formally proceed

towards its definition.

First , by an infinitesimal function   x  as x → a we mean, lim  x   0 . Similarly by an


x a

infinitesimal function   x  as x → ∞ we mean, lim   x   0 . Similarly , by an infinite function


x 

  x  as x → a we mean, lim   x    . Similarly by an infinite function   x  as x → ∞ we mean,


x a

1
lim   x    . Evidently if   x  is an infinitesimal function as x → a or as x → ∞, then is an
x    x

infinite function and vice versa.


1 1 1 1 1
Intuitively we know the series  3
i.e., 3  3  3  3 ......infinite terms is more rapidly converging
n 1 n 1 2 3 4

1 1 1 1 1
than the series 2
,i.e., 2  2  2  2 ...infinite terms ,
n 1 2 3 4
i.e., for a desired approximation , we can we can do with less number of terms of the former series

than the later one.

Let us take two infinitesimal functions   x  as x → a and   x  as x → a. we say that they are

lim   x 
infinitesimals of the same order if xa
 c if c is a non-zero constant. In that case
lim   x 
xa

lim   x 
we write   x     x  If xa
 0 then we say that   x  as x → a is of higher order than   x 
lim   x 
xa

as x → a. A similar definition may be given for x → ∞.

lim   x 
Similarly If xa
  then we say that   x  as x → a is of lower order than   x  as x → a. A
lim   x 
xa

similar definition may be given for x → ∞.

To verify whether these definitions are in consistent with our intuitive idea, the following examples

may be tried.

Example 30

x2 1 x2 1
a) Show that as x → 1 is infinitesimal. Evidently so, since lim 0
2 x3  3 x 1 2 x 3  3
It is easily verifiable that

b) The sum and product of any finite number of infinitesimal functions as x → a are themselves

infinitesimal functions as x → a. Same is true for x → ∞.

c) The product of an infinitesimal function   x  as x → a with a bounded function   x  , (

bounded means it varies within two numbers A and B, may or may not have a minimum and

maximum, e.g., the set {x: x 2  2 } has no maximum but has an upper bound x  2 ) is an

infinitesimal function as x → a. Similar is the case for x → ∞.

Example 31

1
Show that f  x    x  2  cos
3
is an infinitesimal function as x → 2.
x2

1
The lim f  x   0 , as lim  x  2   0 and cos
3
can not be more than 1 or less than– 1 .From the
x 2 x 2 x2

latter example, it seems that using the concepts of infinitesimal may facilitate calculation of limits ; yes

it is so, we would return to that later on. Look at the following examples for understanding

comparison of infinitesimals.

Example 32
Show that both the functions f  x   16  x  4 and g  x   x are infinitesimals for x → 0 and compare

their orders.

Taking limits of the functions for x → 0, we see that both the limits are 0 and

hence both are infinitesimals for x → 0. Now lim


16  x  4
 lim
 16  x  4  16  x  4 
x 0 x x 0
x  16  x  4 

x 1
 lim  , a non-zero constant. So f  x   g  x  .
x 0
x  16  x  4  8

Example 33

Show that the functions f  x   2x3  x  1 and g  x   2 x2  3x  1 are both infinite functions as x → ∞

and compare their orders of largeness.

2 3 1
 2 3
2 x  3x  1 x x
2
x  0 .Hence
Both the functions are infinite functions as x → ∞ . Now lim  lim
x  2 x 3  x  1 x  1 1
2 2  3
x x

g  x  is infinite function of smaller order than f  x  .


§1.8:Equivalent infinitesimals used in calculation of limits

Remember

The following propositions are easily proved and shall enable us to work with infinitesimals freely.

One infinitesimal can be replaced with another one of same order (taking into account the

proportionality constant , of course.)

a) An immediate consequence of the above is that , if   x  ,   x  ,   x  , and   x  , are

infinitesimals

  x   x   x   x
such that   x     x  and   x     x  then lim  lim .Also lim  lim .
xa   x x  a   x x  a   x x  a   x

b) If   x     x  as x → a and   x     x  as x → a, then   x     x  .

c) If   x     x  as x → a and the constant of proportionality is 1, then their difference is an

infinitesimal of lower order and vice versa.

   x     x    x 
0  lim   1  lim   ,  lim   x   lim   x     x     x  x →a
x a   x  x a   x x a x a
   
d) Sum of two infinitesimals of different order is equivalent to the one of them which is of

lower order; for the other tends to 0 faster and is insignificant.

Example34 : 4sin   3 3  4sin   4 , since 3 3 is an infinitesimal of higher order.

Misc. Exercise Using the above, prove and remember the following.

51)   x   sin   x   tan   x   arcsin   x   arctan   x   ln 1    x   if   x  is an infinitesimal x→

a.

52) a  x   1    x  ln a as x→ a., a > 0 . Put a = e and e  x   1    x  .

  x
1    x 
 1
53)  1    x  from ln 1    x     x  . If   , n 1    x   1 
n n

Misc Exercise Using the above, find the following.

sin 3x
lim
x 0 ln 1  5 x 
54) . Hint: use sin5x  5x , ln 1  4 x   4 x and on simplification, the limit is 5/4.

1  cos x
lim
x 0 x
1  cos 2 2

55) 2 . Hint : 1  cos x  2sin 2 x  2  x  , 1  cos x  2sin 2 x  2  x  so Ans. 4.


2 2 2 4 4
arctan 2 x
56) lim Hint : arctan2x  2x , arcsin3x  3x ; So the limit is 2/3.
x  0 arcsin 3 x

57) Calculate 1.03 approximately and estimate the error.

x   x
Ans. Since 1 x  1 (using the formula n 1  x  1 ), we have
2 n

0.03
1  .03  1   1.015 .
2

x
  1
 1
  1
  
2
To estimate the error,  1 x 1  x  2 1  x  2  1  x  2 1  x .1  1  1 x 1 .
2 2 2 2

2
x 1
 1 x x2
2
But again 1 x  , so the error is 1  x 1     . If x = 0.03,
2 2 2 2 8

x 2 0.009
  0.0011 , 0.11%.
8 8

1
58) Show that 1   x . as x  0
1  2x

Hint : 1 
1
1  2x

1
1 2x
 1  2x 1   1  1 
1  2 x  1  x
1 0  2 

3
59) Show that sin x x  x 4 . Hint use   x   sin   x  .

1  cos x   6 x3
2

60) Find the limit when x  0 of


5 x3
x x4
Hint : 1  cos x   6 x3  4sin 4  6 x3  4  6 x3  6 x3 , the one with lower order in the sum of
2

2 16

two infinitesimals. Hence the limit is 6/5.

sin 3 x
61) Find lim . Hint : sin 3 x  3 x , arctan x  x .
arctan x 
x 0 2

ln 1  3 x 
. Hint : ln 1  3x   3x , e5 1  53 x
3
x
62) Find lim
x 0 e 53 x
1

3
163) Show that 1  cos3 x  sin 2 x .
2

3cos x  cos 3 x 3 1  cos x   1  cos 3x 


Hint : cos 3x  4 cos3 x  3cos x  1  cos3 x  1  
4 4

1 3x 
2 2

  3.2sin 2  2sin 2  .  .6.    .2.    x 2 Also sin 2 x  x 2


x 1 x 1 3x 3
4 2 2  4 2 4  2  2

Exercise64: remember.

Cauchy’s necessary and sufficient condition for existence of a limit. The earlier discussion that

the limit, if it exists, does not depend upon any sequence along which the limit is calculated. The idea

is formally stated in the form of Cauchy’s necessary and sufficient condition for existence of a limit of

some function f  x  .
In other words, The lim f  x  exists if and only if for every ε > 0 such that f  x1   f  x2    , we can
x a

find a δ such that x  a   whenever x1 , x2 ]x  a, x  a[ .

Hint : use triangle inequalities.

Exercise65

1 1
Show that the following limits do not exist. a) lim cos b) lim 1
x 0 x x 0
2e x

Exercise66 :Miscellaneous questions and Problems :

Work out and remember these frequently used limits. remember the following ten

results

sin x tan x
a) lim  1  lim cos x  lim
x 0 x x  0 x  0 x

tan -1 x sin -1 x
b) lim  1  lim
x 0 x x 0 x

a x -1 e x -1
c) lim  ln a, a  0 ; in particular lim  ln e  1 a  e ln a a x  e x ln a
x0 x x 0 x (Hint – use , and expand

e x ln a
as an exponential series)
x
1
 1
d) lim 1  x  x  e  lim 1   also x could be replaced by natural number n.
x 0 x 
 x

nx
 1
lim 1    e x
e) n
 n

cos.x -1
Lim
x 0 0
f) x

log a (1  x ) 1 Lim ln(1  x )


Lim
x 0  log a e  x 0  1.
g) x ln a ,in particular x

x 2 x3 x 4
ln(1  x)  x -   .....
(Hint – expand 2 3 4 )

1  x 
m
-1
lim m
h) x 0 x (Hint. take help from (c) and (g)

ln x
lim  0, m0
i) x  x m

n
xn - an  h
 na n-1 a  h  a 1  
n n
lim
k) xa x - a prove by using binomial theorem  a  for all values of a or

xn - an   x  a   x n 1  ax n  2  a 2 x n 3 .....a n 2 x  a n 1 
factorize

Exercise67

a x -1
lim  ln a, a  0 , prove that when x→0 and f  x   0 , remember
x0 x
Using
a f  x  -1 f  x
lim  ln a lim if the latter limit is L. remember
x0 g  x  x0 g  x 

a f  x  1 a f  x  1 f  x
Hint. lim  lim lim  ln a.L
x 0 g  x  f  x  0 f  x  x 0 g  x 

Exercise68 Remember for further problems of form 1∞


.

f  x 1
If x→0 and f  x   0 ,and lim  L then lim 1  f  x   g  x   e L .
x 0 g  x x0

f  x
lim f  x
1
 1
 x0 g  x lim
lim 1  f  x   g  x   lim 1  f  x   f  x e e
g x
x0

L
Hint.
x 0
 f  x  0 

g x
Exercise69: If x→0 and f  x   0 ,and lim f  x  g  x   L then lim 1  f  x   eL . Remember
x0 x0

x
tan
 a 2
2a 
example35 : lim  2   e 

Show that
xa
 x .

x x
tan
  a 
tan
 a 2a 2a
lim  2    lim 1  1   
Ans.
xa
 x xa
  x 
 a x  h   h
lim 1  tan  xa   x  h    a h lim   tan     h  h
e e e h0 a  h   2 2a 
xa  x 2a lim   tan lim   tan  lim   cot  
xa  x  2a
e h0 a  h  2a
e h0 a  h   2a 

h
2a 2a
 lim lim
h0   a  h  h0 h
tan   2a 2
e  2a   .1 
e a
e 

lim f  x   1 lim g  x   
x a xa
Exercise70 When and , show that remember

f  x -1
g x
lim  f  x  
g  x
lim 1  f  x  -1
lim
g  x
 e
xa

x a x a

x
1
 1
lim 1  x  x  e lim 1    e
Also its particular cases already proved above, namely,
x 0
, b)
x 
 x ,

 
1 x

lim 1   x  x  e lim 1    e 
c)
x 0
, b)
x 
 x ,

lim  log 3 3 x  e
log x 3
Ex 71: Show that x 1

lim  log3 3x   lim  log3 3  log 3 x   lim 1  log 3 x 


log x 3 log x 3 log x 3
x 1 x 1 x 1
Ans.

1
 lim 1  log3 x 
1
log3 x e
log3 x
x 1
e log3 x
xc
 xa
lim  
x  x  b
Exercise 72: Find   .

xc
 xa x c
xa lim    lim 1   x  a  1 
x  x  b
Hint : As x→∞, we have x  b →1, and x + c →∞. So   x 
 xb
 

 x c 
x c lim    a b 
 e
  a  b   a b  a b 
 lim 1    e
lim 
x x b 
 x  c 
e x x b 
x 
  x  b 

1 1
  a  b  x  x2
1
 1  ax  x 2 2
 1  ax 2  x2
2

lim 
x 0 1  bx 2
 lim 
x 0 1  bx 2
  lim
x 0
1  
1  bx 2 
Exercise73: Find   . Ans.   

lim
 a b  x 2 . 1
 e x0 1bx2 x2
 e a b

   
1 1 1
lim tan 2 x .
lim 1  tan 2
x 2x
lim 1  tan 2
x 2x
e x0 2x

x 0 x 0
Exercise 74: Find . Ans.

2
1  tan x 
lim   1
x 
e
2 x0 2
e
lim 1  sin  x 
cot  x
lim 1  sin  x 
cot  x

x 1 x 1
Exercise 75. Find . Ans.

limsin  x.cot  x limcos  x


 ex1  ex1  e1

1 1
   x    x 1

lim  tan   x   lim  tan   x    1  tan x  x


 lim 
4  
Exercise 76: Find
x 0
 4  . Ans.
x 0
 x 0 1  tan x
 

1
 2tan x  1
 2 tan x  x  2tan x  1 lim  
 lim 1   e
lim  
x0 1 tan x  x
e x0 x 1 tan x
 e2
x 0
 1  tan x 

1
   ln x
lim  tan   ln x    e2
Show that
x 1
 4 
Exercise77:

1
2 tan  ln x   ln x
1

1
   ln x  1  tan  ln x   ln x
lim  tan   ln x    lim    lim 1  
Ans. x 1
 4  x 1 1  tan  ln x  
 
x 1
 1  tan  ln x  

tan  ln x  1 tan  ln x  1
2lim . 2lim .
x11 tan  ln x  ln x ln x 1 tan  ln x 
e  e2
x1
e
2

If lim 1  ax  bx 2
 x  e3 Show that then a  1 and b may be any real number.
x 0 2
Exercise78 :

2
  2x
1  ax  bx 
lim ax bx 2
Ans. lim
2 x  e  e x0
3
 e3  e 2a
 e3  a 
3
x 0 2

More and difficult problems may be found in the next chapter , the chapter dealing in

L’Hospital’s rule and in the chapter for improper integrals using Newton-Leibnitz formula.

§1.9:Notes on infinitesimal and Hyperreal numbers.

A) Order completeness of real numbers

Describing a real number as a Dedikind’s cut is best understood with the help of this example. A

hypotenuse of a square of unit side each may by symbolically denoted by 2 which is a definite

number as it can be constructed by a ruler and compass. But to express it with only 10 symbols of

digits from 0 to 9, we can only approximate it to any desired degree; like 1.4, 1.41,1.414 ……. etc.

from below 2 or like 1.5,1.42, 1.415 …… etc. from above 2 . The two sequences of numbers are

said to be convergent to the limit 2 . This is a consequence of assuming “order completeness

property” of real numbers. If A and B denote the set of all real numbers less than or equal to 2 and
B denote the set of all real numbers greater than 2 , then the two sets for a ‘cut’ on the real number

line at 2 . One can restate the above statement by taking A to be the set of all real numbers less

than 2 and B as the set of all real numbers greater than or equal to 2 . In this way any real

number is a cut on the real number line, and the set of real numbers is said to be “order complete”.

The rational numbers do not have this “order completeness” property, which is a consequence of

least upper bound property. One cannot “arrive at” 2 by advancing on the sequences of rational

numbers 1.4, 1.41,1.414 ……. etc. or 1.5,1.42, 1.415 …… etc. The set of rational numbers tends to a

limit 2 which is not a rational number ( has been proved before). But all converging sequences of

real numbers are real numbers, i.e., included in the set of real numbers. In topological language, the

set of real numbers is thus called “order complete. The successive approximations that are

“improving” , converge to a real number. ( Thus “ bounded” and “Cauchy” sequences of real numbers

where tm  tn   for every m, n  N converge to real numbers.

The field of complex numbers do not have this order completeness property or Archimedean

property. Given two complex numbers one cannot say which is greater or smaller for that matter.

B) Infinitesimals
The use of infinitesimals is done cautiously; as to where to neglect them and where they cannot be

neglected. To give them an algebraic structure, Abraham Robinson extended the real number system

to include infinitesimals and the extension of R is called the set of hyper real numbers.

1 1 1 1 1 1
A number  was defined as infinitesimal if      .....   ......  and
2 3 4 5 6 m

two infinitesimals a and b are said to be equal i.e., a  b if b – a is an infinitesimal. Also 0 is an

infinitesimal. The hyperreal numbers are not Dedekind complete in the sense that there are no

positive infinitesimals in the set of ordinary real numbers R, (obviously). But the same rules of Algebra

may be applied to hyperreal numbers ( real numbers including infinitesimals) as applied to real

numbers. In this idealization of infinitesimals we can divide a real number by an infinitesimal and the

1
result would be a ‘huge’ number   .Conversely we could divide a real number by  and the result

would be an infinitesimal. The we can make computation rules for finite and infinitesimal numbers as

follows.

a) If a and b are finite, so also a+b and a.b

b) If  and  are infinitesimal, so also    and  .

c) For any finite number a and infinitesimal  , a is infinitesimal


d) Division by 0 is still undefined.

Entire calculus can be rewritten just like ordinary algebra without extra caution for infinitesimals using

the hyper real numbers system.

Remember that  is not a number either in R or in the set of hyper real number set. ; since

.        1  0 which is absurd as it implies   1 or   0


CHAPTER 2 : THE CONCEPT OF LIMIT OF A FUNCTION

PART II

The ideas of the previous chapter are put to use in this exercises. Mathematics is no reading,

but doing. By working out only we become able to recognize and understand the underlying

concepts

Misc. Exercises

Find the following limits. Use binomial expansion, e – series or logarithmic series or any series expansion you

know if necessary. To list :

x x2 x3
a) e x  1     ...................
1! 2! 3!

1 x x x 2 x3
b)  e  1     ........ , putting – x in place of x.
ex 1! 2! 3!

x ln a x  ln a  x3  ln a 
2 2 3

c) a  e
x x ln a
 1    ...................
1! 2! 3!

nx n  n  1 x n  n  1 n  2  x3
2

1  x   1    ..... x  1 binomial theorem n  R


n
d)
1! 2! 3!
x 2 x3 x 4
e) ln(1  x)  x -   ....... logarithmic series.
2 3 4

Also you may use the following series expansions of trigonometric functions, which would be derived

later on.

x3 x5 x 7
a) sin x  x    ........
3! 5! 7!

x2 x4 x6
b) cos x  1    ........
2! 4! 6!

Lim x s Lim x s Lim ( x )2  ( s )2


1) xs
x s . Ans xs
x s = xs
x s
 Lim
x s  
x s 

2 s , as x  s  0

x s  s x  s  s Lim ( x  s  s )( x  s  s )
2)
Lim
x s . Ans.
Lim
x s  xs
x x x( x  s  s )

xss 1 1 1
 Lim  Lim  
xs
x( x  s  s )
xs
xs  s 2s  s s ( 2  1)

Lim  1 Lim  1  Lim 


  x 0  x    0    
 1
x0  x  x0  x
3)  x  ,Ans.  x=  x
xh xh
xh  x Lim x  h  x Lim x ( x  1) x(
x
 1)
 Lim
h0 , Ans. h0  h 0  Lim
h 0
h h h h

1
x 1
 h 2
x [1    1] x [1 
h
 1]
 
 hLim  x  hLim 2x 2x 2 x
0 0
h h

(neglecting higher powers of h/x.)

4) Show that x  
lim x  x 2  x    1
2

Hint : Rationalizing, (remember rationalizing as a technique) 


lim x  x 2  x
x 

 lim
x  x2  x  x  x2  x   lim x
 lim
x / x

1
x 
x  x2  x  x 
x  x2  x  x   1
 x / x  1 
x
2

5) Find lim x4  2 x3  3x2  bx  2  lim x4  2 x3  5x2  3x  d .


x  x 

Ans. Rationalizing, lim x4  2 x3  3x2  bx  2  lim x4  2 x3  5x2  3x  d  lim


x  x  x

x 4  2 x3  3x 2  bx  2  x 4  2 x3  5 x 2  3x  d
x 4  2 x3  3x 2  bx  2  x 4  2 x3  5 x 2  3x  d

8 x 2   b  3 x  2  d b3 2 d 8
 lim 8
 2  4
x 
x  2 x  3x  bx  2  x  2 x  5 x  3x  d
4 3 2 4 3 2  lim
x 
x x 11
2 3 b 2 2 5 3 d
1  2  3  4  1   2  3  4
x x x x x x x x
x 

6) Find lim cos tan 1 sin  tan 1 x  .  Ans.
1
2

7) If lim x4  px3  3x2  qx  2  lim x4  2 x3  rx2  3x  s  4 , find, p,q,r,s. Ans. P = 2, r = 5.


x  x 

1  tan x  sin x  tan x  sin x  tan x  sin x ..


Ex9: Find lim if x  0
x 0
1  x  x  x  ....................
3 3 3

Ans. Putting y  tan x  sin x  tan x  sin x  tan x  sin x ... , we get, y  tan x  sin x  y . Solving it , we

1  1  4  tan x  sin x 
get, y  , since y  0
2

Similarly, putting z  x3  x3  x3  .......... , we get, z  x3  z  z 2  z  x3  0 . Solving it we get,

1  1  4 x3
z .
2

1  1  4  tan x  sin x 
1  1  4  tan x  sin x   1
The given expression becomes, lim 2  lim
x 0 
1  1  4 x3 x 0 
1  4 x3  1
1 
2
 lim
1  4  tan x  sin x   1  1  4 x3  1   1 lim  sin x  1  cos x   lim 1  4 x3  1
 
x 0 
4 x3  1  4  tan x  sin x   1  4 x0  x  x 2 cos x   x 0 
1  4  tan x  sin x   1

  x  
  2sin 2 
 lim  sin x  2   .1  1
x 0   x   x 2
 2
  4   cos x  
  2 

10) Find lim  


cos 2 1  cos 2 1  cos 2 1  cos 2 ...  .
x 0   x4 2  
sin  
 x 
 

 
 
 
cos 2 1  cos 2 1  cos 2 1  cos 2 ..    
 cos 2 sin 2 sin 2 sin 2  sin 2 x  .....  
Ans. lim  lim  
x 0  
sin 
x4 2 

 x 0  
 sin   lim x4 2 
x42  



x 

  x0
 
x x42    


  
lim cos 2 sin 2 sin 2 sin 2  sin 2 x  ....
x 0
 cos 2  0 
   2
 1   
sin   lim  sin  
 x 0 x  4  2  4

  x 2   1
 x2  1
11) Find lim f  x  the function defined by f  x    x 2  1 , for
x 1
0 x2  1
 for
 1
 x 2  1 , for 0  x2  1

Ans. It may be deduced that f  x   0 for x2  1
 1 1
 2 , for 1  x2  2
 x 1

 1
 x 2  1 , for 0  x2  1


0 for x2  1

0, for 1  x2  2

1
Now lim f  x   0 and lim f  x   lim f 1  h   lim   . Limit does not exist.
1  h  1
x1 x 1 h1 h 1 2

12) Show that limsin  x  sin x   0


x

 sin 1 x 
13) lim  1   1
Show that x0  tan x 

1  cos 1  cos x  1
14) Show that lim 
x 0 x4 8

 
15) Show that lim sin  cos tan x   1
x 0
2 

 sin x 
16) Find lim  . Ans. 0
x 0
 x 

 sin 1 x 
17) Find lim  . Ans. 1
x 0
 x 

 tan x 
18) Find lim  . Ans. 1
x 0
 x 
1
19) Show that lim e
cot x 
, lim e
cot x
does not exist. Ans. lim ecot x  1
  e 
x x  x 
2 2 2

20) Show that limsec  x sec2 x  tan 2 x  1  1


x 1

    1  
21) Show that lim tan  cos  sin 3 x    1
x 0
 4    

x sin  x   x 
22) Find lim . Ans. Does not exist
x 1 x 1

ln  x 
23) lim
x  x
Find . Ans. 0.

ln x n   x 
24) Find lim . Ans. -1 .
x   x

 a 2 xx  1
25) Show that  , for x  0 has no limit at x = 0.
f  x    2  x   x
 ln a x0
 for

 a  x  x  1
Hint : Use  x   x  x , so that the function becomes f  x     x   x , for x  0.
 ln a x0
 for

a   1 a   1
h h  h h
ah 1 a 1h  1 1
Now lim f  x   lim  lim  ln a ; but lim f  x   lim  lim   1
x 0 h 0  h   h h 0 h x 0 h 0   h   h h 0 1  h a

.So the limits are different and the limit does not exist.

26) Find all values of a, if lim


x 
 
x 4  x 2  1  a  x 2  1 exists. Ans. 1.
ln  2  x   x 2 n sin x
27) If f  x   lim , find lim f  x  .
n 1  x2n x 1

ln 3  sin1 1
Ans. We have f 1  lim   ln 3  sin1 .
n 2 2

ln  2  x   x 2 n sin x
Now, for x 2  1 , we get f  x   lim  ln  2  x  , as x 2 n  0 , for x 2  1 .and for
n  1 x 2 n

ln  2  x   x 2 n sin x
x 2  1 , we get, f  x   lim   sin x , as x 2n   , for x 2  1 .
n 1 x 2n

So the limit does not exist for x = 1.

1
 1
x
28) Show that the limit lim 
 1   x  x

x 0 e  does not exist.
 
 

1
 1
x
 1   x  x
 1  1 
Ans. Take l  
e   ln l   x   x ln 1   x   1
   
 

1  1  11 
Now, lim ln l  lim
x 0 

x 0   x   x
ln 1   x   1  x 0 x  x ln 1  x   1 , as  x  x as x  0  .
  lim
 

11 x 2 x3   1  x x2   1 x  1
 lim   x    ......   1   lim  1    ......  1  lim     ......   
x 0  x x
  2 3   x 0  x 2 3  x  0 
 2 3  2

1  1 
But when x  0  ,  x  1  . So, lim ln l  lim
x 0 

x 0   x   x
ln 1   x   1   11ln 1  1 1  ln 2 1 So
 

the two limits are different.


1
29) Show that if lim f  x    then lim 0
x a x a f  x

30) Show that lim a h  1 lim ah  A  limln ah  ln A  lim h ln a


h0 . Ans. Suppose h0 h0 h 0

 ln A  0  ln A  A  e0  1

 1 2 3 n 
lim  2  2  2  .......... 2 
31) 
n  n n n n .

 1 2 3 n  1 n  n  1 1 1  1
lim  2  2  2  .......... 2   lim 2 1  2  3  ...  n   lim  lim    
 n  n n  2n  2
n  n n 2 n  2
Ans. n n 2n

12  22  32  .....n2
lim
n  n6 12  22  32  .....n 2 n  n  1 2n  1
.Ans. lim 3
 lim 
n  n n  6n 3

1  1  2  1
lim 1   1   
n  6
 n  n  6

 n 2r  1 
lim   r 1 3 3 
32) Find
n 
 1  2  3  ...r  .
3 3
Ans. 4

x n  nx n 1  1
33) lim
Find
x  xn Ans. 0
13  23  33  .....n3 n 2  n  1 n 2  n  1
2 2
13  23  33  .....n3
34) lim . Ans. lim  lim  lim
n  1 n  1 4  n 2  1
n  2 2 2 2
2 n  2 n  n 
 1 
4n 4  1  2 
 n 

2
 1
n lim 1  
4


n 
 n  1
2
 1  4
4n lim 1  2 
4
n 
 n 

1.n  2.  n  1  3  n  2   ....  n.1


35) lim .
n 12  22  32  .....n2

n  n  1 2n  1
  rn  r  r    n  1  r 1 r   r 1 r 2 
n
 r  n  r  1   n  1 
n n 2 n n 2
Ans. r 1 r 1
2 6

2n  1   n  1  n  2 
 n  1  n  1 
n
   n  
2  3   2  3 

n  n  1 n  2 
1.n  2.  n  1  3  n  2   ...  n.1 6 n2 1
lim  lim  lim 
n 1  2  3  .....n
2 2 2 2 n  n  n  1 2n  1 n  2n  1 2
So,
6

1 1 1 
lim  0.2 
log 5
  ......... n terms 
 4 8 16 
36) Find n  .

 1 
1  log
1  2n 

 5
2
1 1 1  1
1 5

Ans. lim  0.2   lim  0.2 


2log 2
  0.2 
5
  ......... n terms   
5  
log
 
log log log 2 5
 4 8 16  5
4  1 1  5
2 5
 
n  n   2  5

 5
log 4
4
5
2
sin 2 x
 1 sin1 x 
1 1 1 sin x

 1 1 1
 1

 lim n     2  sin x  ..   n  1  sin  n  sin x 


2 2 2 2

 2 sin x  3sin x  ...  n sin x    lim n


x
37) lim 1sin
2 2 2 2
x
     
x 0  x 0  n  n  n  n  x 0
   


   
2

 
sin x
2
1 1 2 1

 1 sin x  n 1
1 1 1 sin x lim lim
1 2
 n  1
2 2 2

       .... 
n  1  sin x
x 0
sin x x 0
sin x n sin x
2 sin x   ..... 
  1
2 2 2

 n  n  n    n n n 
   
 n  0  0  0  .....  0  1  n
0

r 1
lim  r 1
n

38) Find n  1  3r  r . Ans. 2 .
2 4

3n   1
n

lim
4n   1 6k   1
n 2k
n 
3
39) Find . Ans. If n is even, 2k say, then lim   . If n is odd, say 2k +1,
8k   1
k  2k
4

 2 k 1
3  2k  1   1 6k  3  1 3
then lim  2 k 1
 lim  .
k 
4  2k  1   1 k  8k  4  1 4

n
r3  8
40) Find the infinite product lim  .
n 
r 3 r3  8

r3  8
n
 33  8   43  8  53  8   n3  8   3 2 
Ans. lim  3  lim  3  3  3  ........  3   lim  
r 3 r  8  3  8   4  8  5  8   n 8  3 2 
n  n  n

 32   2  3  22   4  2   42   2  4   22   n  2   n   2  n   2 
2 2

 2 2 
   2 2 
 ........... .   2 2 
 3   2  3  2   4  2   4   2  4   2   n  2   n   n  3  2 

  3  2  4  2   n  2    3   2  3  2   4   2  4   2   n 2   2  n   22 
2 2 2 2
 lim     .......    2 2   2 2 
......  2 2 
n 
  3  2  4  2   n  2    3   2  3  2   4   2  4   2   n   n  3  2 

 1.2.3.4.5.6.7.8.....  n  2    19.28.39.52.63....  n2  2n  4  
 lim    
 5.6.7.8.9.........  n  2    7.12.19.28.39.52.63.....  n  2n  4  
n  2
1.2.3.4  5.6.7.8............  n  2    19.28.39.52.63....  n2  2n  4   7
 lim     (need not worry about last
7.12 n  5.6.7.8.9.........  n  2    19.28.39.52.63.....  n2  2n  4   12
 

terms in series when n   )

41) Find lim an where an1  2  an and an  0


n 

Ans. Since lim an  lim an 1 , lim an  lim an 1  lim 2  an  2  lim an .


n  n  n  n  n  n 

if l  lim an , this gives, l  2  l  l 2  2  l  0   l  1 l  2  0  l  2 since l  1 , being sum of


n 

positive numbers.

1  1  4a
42) Find lim an when an 2  a  an 1 and a0  a . Ans. .
n  2

a n  bn
lim
43) Find n a  b where a  b  1 . Hint : divide numerator and denominator both by a . Ans. 1.
n n n

44) Show that lim


 n  2 !  n  1!  1
n   n  2  !  n  1 !

 
1

 r 1 r n
m n
45) lim
n 
Find . Ans. M

n
 2 
46) Find lim 1   r 1 n  .
n

n 
 Cr 
2 1 1  1 1 1  1 1
Ans. Let an  1   r 1 n  1  2. n  2 n  2  n  n  .....  n   1  2. n  2 n 
n

Cr C1 C2  C3 C4 Cn  C1 C2

 1 1 1  1 1
 n  n  .....  n  , n  2 terms , since n
 n etc.
 C3 C3 C3  C3 C4

2 4 n2 2 4 6  n  2 2 10 2 10
 1   n  1    1  ; i.e. an  1   .
n n  n  1 C3 n n  n  1 n  n  1 n  2  n n  n  1 n n  n  1

n
 2 10 
n
 2 5 
n
2  2
Moreover, an  1  n . Thus, lim 1    lim ann  lim 1      
 n  n  n  1  
lim  1  1
n  
 n  n  n n  n  1 
C1 n 

Thus both left and right sides of lim ann  e2 .


n 

n
 a
47) Find lim 1  sin  . Ans. ea .
n 
 n

1
48) Find all values of x for which lim 2n
 0.
n 
4 1 
 tan 2 x 
 

2
4  4 4
By plain reasoning ,  tan 1 2 x   1  tan 1 2 x  1 or tan 1 2 x  1 , i.e.,
   

    1
tan 1 2 x  or tan 1 2 x    2 x  tan  1, or , 2 x  tan  1  x 
4 4 4 4 2

1 1
49) Find all values of x for which lim  .
 cos x 
n  1 n
2 2
1 1
By plain reasoning , lim   cos 1 x  1 , i.e., 1  cos x  1.
 cos x 
n  1 n
2 2

    1
50) tan 1 2 x  or tan 1 2 x    2 x  tan  1, or , 2 x  tan  1  x 
4 4 4 4 2

1
51) lim 2 x
x 0 . Ans. does not exist

 1 1 1
lim  sin  x sin  x 2 sin 
52)
x 0
 x x x  . Ans. lim  sin 1  x sin 1  x 2 sin 1 
 
x 0
 x x x

1  1  x3  1
 limsin lim    limsin .1 .The former does not exist
x 0 x x 0  1  x  x 0 x

 1 1
53) lim  2 x  2 x  x  . Ans. does not exist.
x0
 2 

1
  1  
2
x x x
 1 
54) lim  1  2  (hint put it  lim 1  2  
x 
 x   x  x  

1
Show that lim 1  5 x  x 2  sin x  e5
55) x 0

x 1
 3x3  x 2 
lim  3 2 
0
56) Show that x   5 x  nx 
1
 x2  f  x   x x2  f  x 
lim 1   lim 0
57) Show that x 0  x2  exists only when
x 0 x2 and find it.

1
x2  f  x   x2  f  x   x
lim 1   
Ans. Evidently if x2 does not tend to 0, x 0  x2  .

1
 x2  f  x   x  f  x
2
x
lim
lim 1   e
x0 x3
 e0  1
Now , x0  x2  .

  1  
f  0 lim  f  x   ln 1  f  x    ln f  x    0
58) Find , if x  0
  e   . Ans. 0

asin x  1
lim  ln a
59) Show that x0 x cos x

3x  5 1 3n  5
lim  f n 
60) If x  9 x  4 3 , find how many points of 9n  4 are outside the interval  1  1 , 1  1  .
 3 1000 3 1000 
3n  5 1 19 19
Hint ; we have     . We have to find out n so that
9n  4 3 3  9 n  4  3  9n  4 

18988
19 1 n  703.259
 .Thus 27 .
3 9n  4  1000

Thus 703 points are outside the interval.

3x 2  1 3n 2  1 3
lim 2 lim 2 
61) Find x 5 x  1 . From this show that n 5n  1 5

 sin x   sin x  sinh


62) Show that lim    0 , but xlim    1 . Hint : 1.
x 0  0
 x   x  h

 1 1  1
63) Find  lim  r 1 r  . Ans. We have lim  r 1  r 1 2r   0
n n n
 1  0 from GP formula. So  lim
 n  2  n  2r n 

 1 
64) Find lim  2  cot x  Ans. ∞

x 0 x

tan x  sin  tan 1  tan x  


65) Find lim taking +ve value of square root only.
x tan x  cos2  tan x 
2


We have, lim

tan 1  tan x   x   , i.e. when x  .
x  2
2

tan x  sin  tan 1  tan x   tan x  sin   x  tan x  sin x


So, lim  lim  lim

x  tan x  cos 2  tan x  
x  tan x  cos  tan x  x   tan x  cos 2  tan x 
2 
2 2 2
sin x
1
tan x 1  cos x 1 0
 lim  lim  1

x  cos  tan x  x  
2  cos  tan x 
2
1 0
2 1 2 1
tan x tan x


Also lim

tan 1  tan x   x , i.e. when x  .
x  2
2

tan x  sin  tan 1  tan x   tan x  sin x


So lim  lim  1 as before.

x  tan x  cos  tan x 
2 
x  tan x  cos 2  tan x 
2 2

 n sin x   n tan x 
66) Find lim  , lim  .
x 0
 x  x 0  x 

 sin x  sin x  sin x 


Ans. As n  1   n   n , as  1 , then  n x   n  1
 x  x

 tan x  tan x  sin x 


But n   n   n  1 , as  1 , then  n x   n
 x  x

3
x  x  x x 3 7
67) Find lim . Ans. .
x 1 x 1
3
9

 3 3
 
x  x  x x 3
3
  x 1 x 1 x 2 1  1 
Hint : lim  lim    .
x 1 x 1
3 x 1  x  1
 x 1 x  1  x  x  1 
2

  
x
68) Find lim
x  3
x
x 3
x
x 3
x
x
x  .........

x x x
Ans. Let y    i.e.,
3
x x 3
x 1
x x x 2
y
3 3
x x x
x x
3
x
3 3
x x
x x
x  ......... x  .........

5 5
3
x x 3
4x
y  . Solving it we get, y  neglecting – sign as y > 0 .
x
1 5
 5
10 5 
2
y x y
3
2  x  x  4x3
3
3

x 3  
 

5
4x 3 2 2
Taking limits, lim y  lim  lim  1
x  x   5 10 5  x  4 11
2  x 3  x 3  4x 3  1 1
 
5

  x 3
69) For a > 0, find lim
x 
 a 2 x 2  ax  1  a 2 x 2  1  1
Ans. . Hint : rationalize
2

r
70) Find lim  r 1
n
.
n  1 r  r4
2

1 1  r  r   1  r  r 
  2 2
r r 1 2r 
Ans. Let tr  . Then tr   
1 r2  r4 1  r 2  r 4 2  1  r 2 2  r 2  2 1  r 2  r 1  r 2  r 
 

1 1 1 
   
2  1  r  r  1 1  r  r  1 

1 1 1  1 1 1 1 1  11 1
Now, t1       1   , t2        ,
2  1  11  1 1  11  1  2  3  2  1  2  2  1 1  2  2  1  2  3 7 

1 1 1  11 1 
t3        ,
2  1  3  3  1 1  3  3  1  2  7 13 

1 1 1  1 1 1  1 1 1 
t4         ,……………., tn    .
2  1  4  4  1 1  4  4  1  2  13 21  2  1  n  n  1 1  n  n  1 

r 1 1  1
Adding up and taking limit, lim  r 1
n
 lim 1   
n  2 
1 r  r  1  n  n  1  2
n  2 4
x n 1   n  1 x  n n  n  1
71) Show that lim  , if n is a positive integer > 1.
 x  1
x 1 2
2

x n 1   n  1 x  n x n 1  x  nx  n x  x n  1  n  x  1 x  x 2  x 3  ...  x n  n
Hint : lim  lim  lim  lim
 x  1  x  1  x  1 x 1
x 1 2 x 1 2 x 1 2 x 1

, dividing numerator and denominator by x – 1  0

 x 1 x2 1 xn 1  n  n  1
 lim    ...   , putting n  1 1  ..... 1 , n terms.  1  2  3  .......  n 
x 1 x  1 x 1 x 1 
 2

 1  1  1
72) Find lim 1  1   ...... 1   , given a1  1, an  n 1  an1  for n  2 .
n
 a1  a2   an 

 a  1  a  1   a 1 
Hint : lim  1  2  ......  n  , also from an  n 1  an1  ,
n
 a1  a2   an 

an  a  1  a  1   an  1   a  a   an 1 
we have  1  an 1 . So, lim  1  2  .....    lim  2  3  .....  
n n
 a1  a2   an  n  2a1  3a2    n  1 an 

an 1 1 1  an 1 a 1 1 a
 lim  lim  lim  lim n  lim  lim  lim n1
n  n  1 n! n  n! n  n! n  n! n  n! n   n  1! n   n 1!
1 1 1 1 1 a
 lim  lim  lim ......   1  e , as a1  1
n  n ! n   n  1 ! n   n  2  ! 2! 1! 1!

n
  1  1  1 
  n  1  n  2  n  22  ....  n  2n1    e
 n2 2
73) Show that lim n
n 
     
  n  1
     
n
1 1 1
n n ... n 
n  2 n 1

  1  1  1    lim  2 2 2

lim n  n   n  1  n  2  n  22  .....  n  2n1  
2
Hint : n

 
n
     n
n 

 

n n n
 1  1   1 
n   n  22  n  2n 1
 n 1   
n

 lim  2
     .....  
n 
 n   n   n   n 
     

2n 2n1 n
n n n n
 1  1   1   1  1  2  1  2n1
 lim 1   1   .... 1  n 1   lim 1   1   ..... 1  n 1 
n 
 n  2n   2 n n 
 n  2n   2 n
1 1 1 1
1  .....up to  terms .
 ee 2 e 4 ................up to  terms  e 2 4
 e2

2 x 1
 x 2  2 x  1  2 x 2 1
74) Show that lim  2  
x  2 x  3 x  2
  2

 x2  2 x  1  1 2x 1
Hint : lim  2   and lim 1
x  2 x  3 x  2 x  2 x  2
  2

ln 1  x 
75) Show that lim 1 remember
x 0 x

x2 x3
Hint: ln 1  x   x    ...........
2 3

log 1  x 
76) Show that lim  log e , i.e. log e remember
x0 x
Hint : log 1  x   ln 1  x  log e

ln 1  x  a 
77) Show that lim 1
xa xa

ln  2  x   ln  2  x 
78) Show that lim 1
x 0 x

 x  x  x  x
ln 2 1    ln 2 1   ln 2  ln 1    ln 2  ln 1  
ln  2  x   ln  2  x   2  2   lim  2  2
Hint : lim  lim
x 0 x x 0 x x 0 x

 x  x
ln 1   ln 1  
lim
1  2   lim 1  2   1  1  1 , using the result lim ln 1  x   1
x 0 2 x x 0 2 x 2 2 x 0 x .

2 2

ln 1  2 x   2 ln 1  x 
79) Show that lim  1 .
x 0 x2

  2x  2x 
2 3

 2x    .....   x 2 x3 
   x    ..... 
ln 1  2 x   2ln 1  x  2 3   2 lim  
 lim 
2 3
Ans. lim 2 2 2
x 0 x x  0 x x  0 x

 x 2  2 x 3  ....
lim  1
x 0 x2

axe x  b ln 1  x   9 xe x
lim 2
80) If x 0 x 2 sin x , find a and b. Ans. A = 3, b = 12
x 2 x3 x 2 x3 x 4 x3 x5
ex  1  x    ..... ln 1  x   x     .... sin x  x    ..................
Hint :Use 2! 3! , 2 3 4 , 3! 5!

x cos x  ln 1  x  1
lim
x 0 x2 2
81) Find . Ans.

e x cos x  e x
lim
82) Find x0 x  sin x . Ans. 0.

c
lim e x sin
83) Find x  e x . Ans. C.

etan x  e x e tan x  e x e x e tan x  x  e x e tan x  x  1


lim lim  lim  lim e x lim  1.1  1
tan x  x 0 tan x  x
84) Find x0 tan x  x . Ans. x0 tan x  x x0 tan x  x x 0

f  x g a  g  x f a f  x  f a g  x  g a


85) Find lim , if f  a   1 , g  a   2 , lim  2 , lim 1
x a xa x a xa x a xa

Ans. -3.

x sin a  a sin x
86) Find lim . Ans. sin a  a cos a .
xa xa

sin 2 x  sin 2 y sin 2 y


87) Find lim . Ans. . Hint ; sin 2 x  sin 2 y  sin  x  y  sin  x  y 
x y x y
2 2
2y

 2 
 x 5
x
 x 5 2 
x x
 lim  x
88) Find lim   .Ans. lim    lim 1    e x x3   e2
x  x  3
  x  x  3
  x   x  3 
1 1 1  2 x2  1
 1  7 x 2  x2  1  7 x 2  x2  2 x 2  x2 lim  
x0 15 x 2  x 2
2
89) Find lim   . Ans. lim    lim 1   e  
 e  e2
1 0
x 0 1  5 x 2 x 0 1  5 x
 1  5x 
2 2
    x 0

xa  a x 1  ln a
lim
90) Find x a x  a . Ans. 1  ln a
x a

ax  bx ln a  ln b
91) Find lim . Ans.
x  c x  d x ln c  ln d

92) If lim xa sinb x sin c x  0 and a, b and c are non zero, Show that a  b  c  0
x0

sin b x sin c x
Hint : lim xa sinb x sin c x  lim xa bc lim b
lim c
 lim x a bc .1.1 exists and nonzero only if
x 0 x 0 x 0 x x  0 x x 0

a bc  0 .

cos x   cos 2  sin x 


93) If  is an acute angle, Show that lim  cosc  ln  cos    sin c  ln  sin   .
x c xc

ae x  b
94) If lim  c , then show that a  b  c .
x 0 x

 x 2 x3  x2
a  1  x    ....   b a
ae  b  a b 
 c ,then lim  
x
2! 3! x
Ans. If lim  c  lim    lim a  lim 2!  ...  c
x 0 x x  0 x x  0
 x  x  0 x x 0 x

 a b  x  a b 
 lim    a  lim a ...all 0 terms  c  lim    a  0 ...all 0 terms  c
x 0
 x  x 0 2! x 0
 x 
 a b
If lim   has to be finite, then a  b  0 . Now, substituting this in the above, a  c .
x 0
 x 

x P 1   P  1 x  P
95) Find lim if P is a positive integer.
 x  1
x 1 2

x P 1  Px  x  P x  x P  1  P  x  1 x 1  x  x 2  ....  x P 1   P
Ans.: lim  lim  lim
 x  1  x  1 x 1
x 1 2 x 1 2 x 1

lim
x  x 2
 ....  x P 
 lim
1  1  .....P times   lim  x  1   x2  1  ....   x p  1
x 1 x 1 x 1 x 1 x 1 x 1

 x 1 x2 1 xP 1 
lim 
x 1 x  1


x 1
.....  
x  1  x1

 lim 1   x  1   x 2  x  1  .......   x P 1  x P 2  ...  1 
P  P  1
 1  2  3  .....  P 
2

1
1 b n 1
n n lim
 b 1  n b 1  b 1
n
n n a 1 1 1 1
lim n ln b
lim  1    e  ea  eln b  b a
a
 lim  1  e n a n
n
 a  . Ans. n  a 
96) Find

2
 tan x 
  x1
1

   
1 1 lim   1
lim tan 2 x
2 x0 x 
lim 1  tan 2
lim 1  tan 2
 e x0 e  e2
2x 2x 2
x x
97) Find x0 x 0
. Ans.

x  x   x  x  sin x
lim cos   cos  2  cos  3  .......cos  n 
98) Show that n 2 2  2  2  x .
 x Hint : lim
sin 2n  n  n

 lim  2   lim sin x / x  sin x . 1



sin x 1 sin x
. 
n n  x  n  x  x x  x x x 1 x
2 sin  n  sin  n  / n lim sin  n  / n
2  2  2 n 2  2  x  x   x 
cos   cos  2  ...2sin  n 
2 2  2 
 x 
2sin  n 
2 
 x  x  x   x   x  x  x  x 
sin   cos   cos  2  ....sin 2  n  sin   cos   cos  2  ...sin 22  n 
 lim 2 2 2   2   lim 2 2 2   2   ......  ... If
n  
x n  
x
2sin  n  22 sin  n 
2  2 

x3 x 4
e  x
cos 2 x  cos x  e x cos x 2 2 is a non-zero real number, find the integer n.
the lim  lim
x 0 xn x 0 xn

 cos xs  1  cos x  e x   
x3 x 4 x3 x 4
cos 2 x  cos x  e x cos x  e x  
Ans. lim 2 2  lim 2 2
x 0 xn x 0 x n

 x2 x4 x6  x3 x 4 x3 x4 x5 x3 x4
     ...      ......  
 x3 2 x5 
 lim  
2! 4! 6! 2 2  lim 2 2 12 2 2
  x  x 2
   ..... 
 x 0
lim n n
x 0 xn  3! 5!  x x 0 x

x5
 ......Higher powers of x
 lim 12 . So this is non-zero and finite if x = 5
x 0 xn

1  cos  ax 2  bx  c 
99) Find lim if  is a root of ax 2  bx  c  0 .
 x  
x  2
ax 2  bx  c
1  cos  ax  bx  c 2 2sin 2
2
Ans. If the other root is  , then lim  lim
 x    x  
x  2 x  2

a  x    x   
2sin 2 a2  x     x   
2 2

 lim 2 2 lim
a  x     x    x 4 x  
x  2 2 2

2b c
2

 2.lim
a2  x   

a 2    
2 2



a 2      4 a
 
 2 4 
a a
2


b 2  4ac 
x  4 2 2 2 2

a
 1x  2 x  3x  ..........  n x  x
100) lim  
Find x0  n 

a 
1  2 3 .......... n n a 1x 1 2 x 1 n x 1  a
 1x  2 x  3x  ....  n x  n  x
x x x x
 lim  lim ...... lim .
lim . x0 x  n
Ans. lim 1    e x0 n x
e  x0 x x0 x 
x 0
 n 

a
 ln1ln 2.....ln n . a
  eln n!  n   n! n
a
e n

x e, for n 1
 1  
101) Show that lim 1  n   1 for n 1 remember
x 
 x  
 for n 1

x
 1   1  lim x
It is proved already lim  1  n   1 , for n = 1, so lim 1  n   1x  1 . Also it is trivial to prove
x 
 x  x 
 x 

 1 
lim 1  n    for n < 1.
x 
 x 
1
xn lim
 1  
x x n1
n
x x
 1   1 x n1

Now, lim 1  n   lim 1  n    lim 1  n   .
x 
 x  x 
 x   x  x  
 

xn
 1 
 0 for n > 1 and lim 1   e.
1
Now lim
x n1 n 
x x 
 x 

1  m 1m  2m  m 2m  3m  ......
102) Find lim lim
m  n  n2

m m
1 2
1  2    1  3m    1  ......
m
1  m 1m  2m  m 2m  3m  ...... 2 3
Ans. lim lim  lim lim 2
m n  n2 m  n  n

1  2  3  ......  n n  n  1 1
 lim  lim 
n  n 2 n  2n 2 2

ax p  bx p 2  c a
lim p  pq pq
x  dx  ex q  2  f
Ex105: Find . Ans. if ,0 if and d if p = q remember

1
 f  x  x
f  x  lim 1  x   e
3
x 0
Ex106: Find the polynomial if  x  .

1 1  x2  f  x  1 
 f  x  x  x2  f  x   x lim 
x0

x   x2  f  x  1 
Ans. lim 1  x      e3  lim 
x
 e 3
lim  1    e e 
3
x 0 x 0 x 0
 x   x   x x

 x2  f  x  1 
f  x   a0 x  a1x
n n 1
 ......  an lim  3
If , then x 0  x x
 x 2  a0 x n  a1 x n1  ....  an 
 lim  2 
a a

x 0 x

 3  lim  n2  n 1  an 2  1  x a0 x n3  a1 x n4  ....an2   3
x0
 x  x

an  0, an1  0, an2  1  3  an2  2


Equating coefficients of both sides, and other coefficients may

be whatever real numbers

1  cos x
Ex107: If a  min  x 2  2 x  3 and b  lim
n

x 0 x2
, find a b
r 1
r nr
.

x
2sin 2
2
 
Hint : a  min  x  2 x  3  min  x  1  2  2 , b  lim
2
x 0
1  cos x
x2
 lim
x 0 x2
2 1
 . Ans.
2
4n 1  1
3.2 n
4.
4

Ex108: If a square is inscribed in a circle of radius R, a circle is inscribed inside the square, again a square is

inscribed in the circle and the process is repeated for ever, find the sum of all the squares.

Hint. If a1 is the side of the 1st square, a1 2  2R  a1  2R . If a2 is the side of 2nd square, a2  2a1 . Thus

the sum of the areas of the squares is

 a2 a2 
lim  a12  a2 2  a32  .......  lim  a12  1  1  .......   2a12  4 R 2
n  n 
 2 4 

Exercise 109:
An isosceles triangle ABC is inscribed in the circle ABC with AB = BC . Another isosceles triangle ABD is drawn

inscribed in the same circle whose base is AC, one of the equal sides of triangle ABC. If inscribing isosceles

triangles is continued indefinitely in this fashion, show that ultimately the triangle shall be equilateral in the

limiting case.

Ans. Taking the base angle ( angle B) for the first triangle ABC to be Ө, observe that at the same time, Ө =

angle ADC. The base angle of the second triangle ADC , i.e., the angle DAC = angle

1800   
DCA =  900  . Similarly the base angle of the third triangle shall be
2 2


900  
2  900   90     900  90   . This is just putting 900  in place of Ө mechanically. The base
0 0
90 
0
 
2  2 4 2 4 2

 1  . Now taking this to the


n 1
900 900 900
angle of the n – th isosceles triangle shall be 90 
0
   .............
2 4 8 2n1

900
limit n   , we get  600 . If both the base angles of the triangle in the limiting
1
1
2

case become 600 , the triangle becomes equilateral.

Exercise110
Show that the triangle of largest area inscribed in any circle is equilateral.

Ans. ABC is a triangle inscribed in a circle ABC. If CD is a diameter, B  ADC and DAC  900 . Then

AC b
sin B  sin ADC   where CD  2R , R being radius.
CD 2 R

b c a a b c
Thus  2 R . Similarly  2 R and  2 R . So the relation    2R ,
sin B sin C sin A sin A sin B sin C

The area of the triangle is given by ( drop the symbols for angles for sake of brevity)

1 1
  bc sin A  2 R sin B 2 R sin C sin A  R2  2sin B sin C  sin A  R 2 cos  B  C   cos  B  C   sin A
2 2

If A be fixed, the expression must be maximum if cos  B  C   1 , i.e., maximum i.e.,

B = C. For another value of A also the condition B = C would give a triangle of maximum possible area in the

circle. So, when all the angles vary, the condition would force

A = B = C to give the inscribed triangle of maximum area.


CHAPTER 3:CONTINUITY AND THEORY OF

EQUATIONS

§3.1:The Concept of Continuity.


It is easier to understand discontinuity than to understand continuity. We all understand what

continuity and discontinuity concepts are. But the concepts are to be formalised for mathematical use.

In Mathematics the concepts of continuity and discontinuity are abundantly used. Let us start with

discontinuity. In the accompanying graph, you may observe that you cannot draw the graph with a

pencil in a single go, i.e., without lifting its head out of the paper even for once. That is exactly

continuity of the graph. Now it remains to translate this idea into language of Mathematics. When x

tends to 1 from below, y tends to 1,when x Consider the same graph of the previous section

repeated here and the same equation y = x, for x [0,1[, y = 2 – x, for x  ]1,2] and y = 2 for

x=1.When x tends to 1 from above, also y tends to 1. But when x is exactly equal to 1, y = 2 ,as if

leaving a ’ hole’ in the graph at x = 1. This indicates two things, 1) the graph would have

been continuous if this ‘hole’ would not have been there and 2) we could make the graph

continuous by ‘plugging’ this hole by defining y = 1 instead of 2 , at x = 1.In other words, the

graph would have been continuous at x=a.

If Lim f(a + h) = f(a) a function f(x) is said to be continuous at x = a. If it is


h0

continuous at all points the function is simply said to be continuous.

The statement Lim f(x) = f(a) can be restated as : for every  such that
xa
 f(x) – f(a)  <  ,we can find a  such that  x – a  <  (If definition of limit is applied by taking

L = f(a) as per above statement.

These are the two statements describing the concept of continuity in mathematical language; the first

one describes continuity in terms of the concept of limit . Though this is sufficient for the purpose of

describing continuity, the second sentence above describes the concept of continuity in a manner

similar to the    description of the concept of limit. This gives real insight into the understanding of

the concept, while description of continuity in terms of limit is like a formula in mathematics with which

it is easier to work out.

A function which is continuous at any point of an interval is called continuous in the interval.

Example1

sin x  
Show that e x is continuous in R, tan x  is continuous at 0, 2  .
cos x

A function shall be discontinuous at x = a , naturally, if


a) the limit of the function at x = a does not exist ( or the left and right handed limits exist

but they are not equal)

Example2 The function [x], the largest integer not larger than x at x = 3 , or at any integral

value of x is discontinuous, as lim[ x]  2 , but lim[ x]  1


x2 x2

b) when the limit exists but not equal to the functional value ,

Example3 The function given in the beginning of this chapter, defined as

 x, for 0  x  1

f  x    2, at x 1 ; Here, lim f  x   1  lim f  x  , but f 1  2
x 1 x 1
2  x for 1  x  2

c) where there is no functional value defined at x = a.,

x2  4
Example4 In the function f  x   lim f  x   4  lim f  x  , but f  2  is not defined.
x  2 x2 x 2

d) When the limit is infinite.


Example5 f  x   tan x at x  . Here tan x   and tan x  
2

e) When there is a wide gap in the graph ( Horizontal, vertical or both. See the figure:
1
f) When there are too much oscillations at a point, e.g., the graph of the function y  sin at x
x

= 0. See the following graph


We can proceed as near as 0 on the graph from both sides but there is no limiting value neither the

1 1
functional value there. Another example may be taken y  x sin . Like the graph of y  sin , the
x x

1
graph of y  x sin also goes through myriads of oscillations as x  0 from both sides, but the
x

oscillations become smaller ever in vertical width as we approach 0. See the graph below. So we

have a limiting value 0 of the function at x = 0. Still the function is discontinuous there as there is no

functional value defined there.

The topic of removable and non- removable discontinuity shall be dealt in detail subsequently.
The algebra of limits can be applied to continuous functions without any concern. Thus if f(x)

and g(x) are continuous functions , so also f(x) + g(x), f(x) - g(x), k. f(x) where k is a

constant, f x) . g(x) and f x) / g(x) would be continuous.(if g(x) is not 0)

Example6

sin x  
Show that f  x   sin x  cos x , g  x   2sin x ,   x   are continuous in 0, 2 
cos x

A pitfall: Example7

f  x
Give an example of a function which is discontinuous at x = a but both f  x  and g  x  both are
g  x

continuous at x = a. Does it contradict algebra of limits? Why?

Ans. Try f  x   x  a , g  x   x  a . Try another f  x   x  a , g  x   x  a


it does not contradict

algebra of limits, because Algebra of limits excludes the cases where denominator

becomes 0.

Exercise1

Give an example of a function f  x  . g  x  which is continuous at x = 0 even if f  x  is


discontinuous at x = 0. Does it contradict algebra of limits ?

 1, for x0
Ans. Try f  x    and g  x   0
0 for x0

f  x g  x f  x  . g  x
Algebra of limits says if and are continuous at a point so also but is it never

f  x  . g  x f  x g  x
says that cannot be continuous if one of and or both of them are discontinuous.

Exercise2

f  x  1
x
g  x 
What is the difference between and x

Hint. The second function is discontinuous at x = 0, as it has limit 1 there but no value, not

defined there.

§3.2: A further result in Algebra of limits: (limit of a composite

function)
If f is a function of x (e.g. f(x) = x2 ) and g is a function of f ( e.g., g(f) = sin f, ) the

composite function or function of a function , g(f(x)) = sin x2 . If limit of f(x) is L at x = a ,

consider whether limit of g(f(x)) = g(L) . This at once requires L must be f(a), i.e., f must be

continuous at x = a and (obviously L belongs to the domain of g) .Otherwise, g(L) might

not exist and we arrive at nowhere .

Under this condition Lim g(f(x)) = g(f(a)) = g( Lim f(x) )


xa xa

A sketch proof may be given as follows :

For every ε > 0 such that g  f  x    g  f  a     , we can find a δ such that f  x   f  a    , for g is

a continuous function of f. Further for every δ > 0, such that f  x   f  a    , we can find a ζ > 0

such that x  a   , for , f  x  is continuous at x = a. This gives, For every ε > 0 such that

g  f  x    g  f  a     , we can find a ζ > 0 such that x  a   which proves the function g  f  x   is

continuous at x = a.

Example8

It won’t be difficult to see that the theorem could easily proved for g = f 2
( Lim f(x) . Lim f(x) = Lim ( f(x) ) 2 as per a result of Algebra of limits) . This
xa xa xa

result also could be extended as continuous function of a continuous function has to

be continuous as it is expected at the point concerned.

A pitfall: Example9


Consider the function z  tan y .Evidently it is continuous at y = 0. Now let y   sin x . This is also
2

 
continuous at x = 0. But z  tan   sin x  is discontinuous at 0 as it tends to ∞ there. Does it
2 


contradict the above theorem ? . No. because the argument of z is there, not 0 as required by the
2

theorem. Think a little hard.

Exercise3

1
The function f  x   has obviously a discontinuity at x = 1. show that f  f  x   has no
1 x

discontinuity

anywhere. Find the point of discontinuity of f  f  f  x  .

x 1
Hint. f  f  x   and f  f  f  x   x
x (verify)
Concept of continuity is used to study of functions , particularly at points where they are

expected to change suddenly. We are thus able to know whether near such points we could

smoothly extrapolate the curve . Actually ‘rates of change’ of a curve from point to point is

the tool required to extrapolate a curve, as you might have correctly guessed. But in order to

have ‘rate of change’ to exist, or to have any meaning, continuity is the minimum requirement

, as we would see later.

Example 10 : Can you construct a function discontinuous at x = a, x = b, and x = c and

continuous everywhere else ?

( x 2  a 2 ) ( x 2  b2 ) ( x 2  c 2 )
Try   or
x a xb x c

( x 2  a 2 ) ( x 2  b2 ) ( x 2  c 2 )
 . (Functional values at a, b, c do not exist).
x a xb x c

Exercise4

Put the denominator equal to 0 and solve for finding the points of discontinuity.

sin 2 x x3  6 x  11 3sin 3 x x 2 sin 2 x


a) b) c) d)
1  cos x x  2x  2 4 cos x  2 cos x  2
2 2
tan x
e)
 1 
cos  
 sin x 

Ans. a) 0 b) denominator has no real root, hence no point of discontinuity.

1 2
c) cos 1 d) no discontinuity as the denominator has no real root. e) sin 1
2 

Example 11 : Can you construct a function which is discontinuous everywhere?

Let f(x) = 1 for all rational values of x in [2,5] and Let f(x) = 0 for all

irrational values of x in [2,5] . Try taking limit of the function anywhere in the interval, say

at x = 3.2. The limit does not exist because in any small interval we may please to take

around x = 3.2, the function oscillates infinite times between 0 and 1. Because between

any two irrational numbers, there exists a rational number and between any two rational

numbers there exist an irrational number. So the function has no limit at any point of its

domain. The question of continuity does not arise. (This is called Dirichlet function.)

Exercise5

Show that f  x  is continuous if f  x  is.

Hint :. f  x  h   f  x   f  x  h   f  x 
Exercise6

Find out the number of points of discontinuity of sin x  cos x in the interval 0  x  2 , if  a  denotes

integer part of a. Ans. 5 points.

 1 
 x sin. x , when..x  0, 
 
Example 12: Examine continuity at x= 0, of f ( x)   
and ..0..when..x  0 
 
 

1 1 1 1 1
We have , sin  1  x sin  x x sin  x 0  lim x sin  lim x  0  lim x sin  0
x x x x 0 x x 0 x 0 x

which is f(0). Hence f(x) is continuous at x = 0.

Exercise7

 sin  cos x  
 , when x
 2
 x 
Test for continuity of f  x    2 at x  . Ans. Continuous
 2

1 when x
 2

Exercise8 If b denotes fractional part of b,

 sin cos x 
 , when x
 2
 x 
Test for continuity of f  x    2 at x  . Ans. Discontinuous
 2

1 when x
 2
sin x
Example 13 : Investigate f (x)  for continuity. Its limit is 1 as x  0 and it is not
x

defined for x = 0 ., so discontinuous there.

sin x
Further note that for f (x)  x = 0 is a point of ‘Removable Discontinuity’.
x

 sin x
 , for , x  0
For, if we define f(x) such that f ( x)   x the function becomes continuous at x
 1, for , x  0

= 0.

Lim sin ax
Example 14 : Find x0 what should be value of the function if the function is made
x

continuous there?

Lim sin ax
f  0  a
sin ax
We have x 0  a xLim
0  a.1  a
x x then would make it continuous

sin x  sin a
Example15:Find xLim
a . How can you make it continuous?
x a

xa xa xa


cos sin sin
We have, Lim sin x  sin a Lim
 x a 2 2  Lim cos x  a Lim 2  cos a.1  cos a
x a
xa xa x a
2 x c xa
2 2 by defining

f  0  cos a
the function could be made continuous.

Exercise9
If
f  x 
1
1  x , find
 
f f  f  x 
and discuss its continuity.

 
  1   1   1 x 
 
f f  f  x   f  f    f 
 1 x   1   f 

1
  1 x  x
x  1
Hint.   1 
 1 x  x

§3.3:Removable and non-removable discontinuity; discontinuity of 2nd

kind.

In a previous example, we have seen what is a removable discontinuity. It is a point at which the

functional value is different from the limit at that point or not defined, leaving a point

hole there . Take another example of 3 removable discontinuities f  x  


x 2
 a 2  x3  b3  x 5  c 5 
.
 x  a  x  b  x  c 

 1 1
  x, for 0  x  ,  1 for x  ,
2 2

Take another example f  x     1  x for 1  x  1,  x  1 for 1  x  3

 2 2
 3 x 3
  2 for x  2 , and x  1  4 for x  2

Find the 3 points of removable discontinuities yourself as an exercise and remove the three

discontinuities by suitably defining the function at those points.

But, instead of point hole, if there is wide vertical gap ( or horizontal gap or a gap in both directions)

in these functions . Examine from its graph and find where it is.

 x2 for x2
 x, for x  2 
a) f  x   b) f  x    3 for x2
x 1 for x2 1 for x2

Such a function with finite number of discontinuities is called piecewise continuous functions.

Such piecewise continuous functions have some useful and interesting properties. For example, if the

domain of the function can be partitioned into a finite number of open or half closed intervals, and if

it is continuous inside each of th intervals then we can find the area under the curve or graph of

the function bounded by the two ordinates at the extremities and the x – axis. This is Riemann

integration of the function ; the area is unaffected if a finite number of ordinates are removed. But if

there is a wide horizontal gap such as

 1, for x2
f  x   , the function is not defined in the interval [2, 3] and this is evidently not a
3 for x3

piecewise continuous function. The area under the curve bounded by two
ordinates at the extremities has no meaning.

Example16

To look into the phenomenon of continuity a little closer, let us examine the Dirichlet function

  x  
1, if x is rational
0, if x is ir rational
which is everywhere discontinuous for non-existence of limit.

As in case of limits, remember the Heine’s definition the lim f  x   L should be unique
x a

through every sequence xn converging to ‘a’. Similarly, if f  x  is continuous at x = a, if

lim f  x   f  a  through every sequence xn converging to ‘a’. Similar statement for limit → ∞.
x a

Example17

 x 2 , if x is rational
Show that f  x    2 is nowhere continuous.
  x , if x is irrational

At any point x = a, rational or irrational, we can find a sequence of rational numbers xn converging to

a, so that lim f  x   a 2 ; we can also find a sequence of irrational numbers xn converging to a, so that
x a

lim f  x   a2 ; since there are two different along two different paths, the limit does not exist and
x a

hence it is discontinuous at x = a, and everywhere for that matter.


Example18

Can you give an example of a function continuous everywhere except at x = 2, x = 3.75? The

discontinuity should be non-removable.

Ans. Try f  x    1,
0,
for x  2
for x  3.75

Example 19: Distinguish between the following discontinuities

a) f  x   x  x at x  2 (jump discontinuity)

1
b)   x   sin at x0 (oscillating discontinuity)
x

1
c)   x  at x3 (infinite discontinuity)
x 3

x2  9
d)   x  at x  3 (removable discontinuity)
x3

Misc. exercise10

a) Whether the following functions ( 1 to 20 under b) are continuous?

b) If not, can you make these functions continuous by suitably defining functional values at the

point of discontinuity if any?


10 x  1 10  1
x
sin x
1) f  x  2) f  x  3) f  x 
x x x

1 x2 cos x
4) f  x   3x  5) f  x  6) f  x 
x2 cos x x

x x2
7) f  x  8) f  x  9) y  cos ec 2 x
cos x x  7 x  12
2

x x2 1
10) f  x   tan 11) f  x  12) f  x   5 2  x
2 1  sin 2 x

 x 1 
2
1 1
13) z , when y  14) y 
y  y2
2
x 1  x 1 

1  tan 2 x 1 tan x
15) y
1  tan 2 x 16)
1
at x =0 17) x at x = 0
1 2 x

4 x 2  3x 1 x 1 sin 2 x
18) 5 x at x = 0 19) x when x = 0 20) 1  cos x

Answers 1), 2), 3),6), 9),17), 18), 19) continuous except at 0 4) continuous except at 2

 3
,
5), 7), 20) continuous except at 2 2 8) continuous except at 3, 4.
10) continuous except at 1, 3. 11), 15), 16) everywhere continuous, 13) discontinuous at

1
x ,1
2 14) continuous except at – 1 .

Exercise11

Whether the functions are continuous. If not, what is the type of discontinuity ?

[sin x] [tan x]  x 1  x 2  
 
a) b) c)

Exercise12

 2, at x0

A function is defined like f  x    x  2 for x  2 . Find out whether the function is continuous at x =
 x2 1 for x  2

0, and at x = 2. If discontinuous, which kind of discontinuity it is?

Henceforth try to understand the statements making mental picture of the same. The proofs

may be omitted at first time reading and the results may be remembered to work out

problems. Thereafter the proofs shall boost confidence gathered by and by in course of

working out problems.


Ex13: Let y  t 2  t t , x  2t  t then define y in terms of x ; i.e. find y  f  x  and discuss its

continuity.

Ans. when t  0 , we have x  2t  t  t , y  t 2  t 2  2t 2 .

2 x 2 , for x0
When t  0 , x  2t  t  3t , and y  t  t  0 . Thus f  x   
2 2
.
0, for x0

Now verify that lim f  x   0  f  0   lim f  x  . so the function is continuous.


x0 x0

Ex14. If f  x   1  x  2 , for 0  x  4 , g  x   2  x for 1  x  3 , then show that fog  x  is

continuous for 1  x  2 .

 x  1, x  0,

Ex15: Show that the function f  x   1, x  0, is discontinuous at x  0 and the discontinuity could
 x  1, x  0
2

be removed by defining f  0   1 .

Discontinuity of 2nd kind:

When at least one of the limits lim f  a  h  and lim f  a  h  is infinite ( or both are) the discontinuity is
h0 h0

called discontinuity of 2nd kind.


1
Ex16: Show that the function f  x   has a discontinuity of 2nd kind. Find 10 examples like that.
x

§3.4:A little more about real number sets and sequences., limit points

and bounded sequences

The concepts of limit and continuity of functions are intimately related to sequences. So we must

spend some time investigating sequences which would , of course give further help in understanding

properties of continuous functions.

Informally, a sequence , already intuitively known, may be regarded as a function of positive

integers or the set of natural numbers. Tn or n-th term of the sequence is actually T  n  , the

functional value of T , when x = n, the argument of the function. A sequence Tn , n  1, 2,3, 4,...............

converges to a limit L, if , for every ε > 0, there exists a natural number M, such that T  n   L   ,

for all N > M. Such sequences are said to be convergent sequences . A sequence which is not

convergent , either may be divergent or oscillatory . For example, the sequence T  n   n2 is

n
divergent. whereas T  n   sin is oscillatory.
3
A number ξ is a limit point of a sequence T  n  if any neighbourhood of ξ , i.e., the open interval ]ξ –

δ , ξ + δ[ contains infinite number of members of T  n  , for any δ > 0. It is easy to understand that if

only one limit point exists for a sequence, it is convergent to the limit point as the limit; if more than

one limit points exist, the sequence oscillates between them and it is oscillatory, if no limit point

exists, it is divergent.

Exercise17

a) If ξ is a limit point of a real number set A, show that there is a sequence in A converging to ξ and

vice versa.

b) If ξ is a limit point of a real number sequence t n, n = 1,2,3……..∞, show that there is a

subsequence of tn converging to ξ and vice versa.

A sequence is bounded if there are real numbers A and B such that A  f  n   B for all n. Next, any

infinite bounded sequence has a limit point- this is called the Bolzano-Weisstras theorem.

Consider the range of the integral function f  n  . If the range contains infinite number of points, there

would be at least one point f  m    , around which infinite number of points in the range will remain

in the neighbourhood ]ξ – δ , ξ + δ[ ,otherwise the range would not be bounded if only finite number
of points remain around neighbourhoods of all points in the range, as there are infinite number of

points in the range. According to the definition of limit point, this point ξ in the range is a limit point.

The set of limit points S of a set D is called derived set of D.

There is a theorem on upper bounds and lower bounds ; the set of upper bounds of a set of

real numbers has a smallest number, and the set of lower bounds of a set of real numbers has

a largest number. Let D be a set bounded above and consider two subsets L and U of the set of real

numbers such that L = { x : x is not an upper bound of D} and U = { x : x is an upper bound

of D}. so that L U  R , the set of real numbers , If x  L and y  U , then x  y . By order

completeness property of R , either L would have the greatest member of L or U would have the

smallest member of U. ( order completeness property of R otherwise means that if x, y  R ,then

either x  y or x  y or x  y ; the two statements may be verified to be same ). Let L have the

greatest number  of it. Then  is not an upper bound of D. So there is a number b of D such that 

< b . Let ‘a’ be a real number such that  < a < b. the number ‘a’ is greater than the greatest number

of L so a U . Also ‘a’ cannot be an upper bound of D since it is smaller than a member b of D. Thus

  L and not to U. If L has not its greatest number of it, then U must be having smallest number of it ,
because of order completeness property of R. Complex numbers have no order completeness

property.

The least of upper bounds of a set S bounded above is called supremum of S or sup S.

And the greatest of lower bounds of a set S is called infimum S or inf S.

Further, a bounded sequence has greatest and least limit points. For, if S is a bounded set of

real numbers, then S  [a, b] . If S’ is the set of limit points of S, then S '  S  [a, b] . Let

G  sup S ' and g  inf S ' . It may be shown that both G and g belong to S’, i.e., they are

themselves limit points of S. Take ]p, q[ a neighbourhood of G ; then there is an ξ in S’ such that

p    q , which says that ]p, q[ a neighbourhood of  and as such it contains an infinite number of

members of S , as   S ' means that  is a limit point of S. Thus G  S '  S .similarly g  S . The

above analysis shall facilitate exploring useful properties of continuous functions.

Continuity at end points

If a function f  x  defined in the interval [a, b] is said to be continuous at x = a, if f  a   lim f  x  and


x a 

similarly it is said to be continuous at x = b if f  b   lim f  x  .


x b 

§3.5:Properties of continuous functions


It is needless to say that sum, difference and product of continuous functions are continuous.

This is in view of Algebra of limits. ( prove them as exercises). Also if f  x  and g  x  are continuous

f  x
functions in a domain and g  x   0 then must be continuous in that domain. There are further
g  x

important properties . These properties tell more about roots of functions, their location and even

help in estimating the roots approximately in some cases. These are not only true for

polynomials or n-th degree equations but transcendental functions which are represented by

converging infinite power series which cannot be expressed in ‘closed’ forms like

x 2  4ac
x4 
1  x 2 etc. ,
5 x3

such as trigonometric, inverse trigonometric, exponential and logarithmic functions etc. Closed forms

are not like sin x tan -1x etc, they are only names given to infinite converging series they stand for. In

contrast a converging infinite series like 1  x  x 2  x3 ................upto  for |x| < 1, can be expressed as

1
which is a closed form. Both polynomials ( finite power series) and functions in closed forms are
1 x

called algebraic functions in contrast to transcendental functions. An algebraic function or

expression in x is one which is derived from x by a finite number of algebraic operations on it;

algebraic operations means addition, subtraction, multiplication, division , raising to power and
extracting roots. It may be noted in passing that transcendental functions may have algebraic

irrational numbers as roots, even rational numbers and even integers. For example, ln e  x  0 has

solution x = 1. Only analysis of roots of a polynomial comprise a branch of Mathematics, called

Theory Of Equations. Some properties of functions would be revealed after the differentiation

chapter also so that something of theory of equations also may be explained thereafter. Regarding

solving polynomial equations, centuries elapsed for finding solution of quadratic, cubic and

biquadratic equations. Then Abel pointed out that equations of degree five or more cannot be solved

in general by standard algebraic methods ( in terms of roots and powers of coefficients) although an

n-th degree polynomial with rational coefficients has n roots. The fundamental theorem of Algebra

is that every equation of nth degree has at least one root, real or complex. Though known for

centuries earlier, this was first proved by a most gifted genius, Karl F Gauss in the last century once

when has 22, then at 44 and then at 82, with further improvement each time. The attempts to prove

the fundamental theorem of algebra has created many branches of Mathematics, modern algebra,

analysis, topology, spectral theory, functional analysis and differential equations, etc. A polynomial

equation however can be solved by numerical methods to any desired degree of approximation ( also

on computer).
To cheer you up, you can at least prove that a polynomial of odd degree with all of its coefficients

positive has a real root. See one exercise after monotonic functions given below.

Exercise18

Show that a polynomial y  a0 x n  a1 x n 1  .........  an 1 x  an of n-th degree is everywhere

continuous.

Hint. Prove that x n is continuous everywhere by showing lim xn  an . Use binomial theorem in
x a

n
 h
expanding  x  h   x 1   for including all real values of x.
n n

 x

§3.6:Further Properties Of Continuous Functions

A) A continuous real function f(x) in a closed interval [a, b] must be

bounded.

Suppose the range is not bounded . So we can have a sequence xn in [a, b] and a

natural number M such that f  xn   n for all n > M. Let  be a limit point of this sequence

xn as the sequence is bounded. Since the function is continuous at  , there is a δ > 0 such

that f  x   f    1 whenever x     . This implies f  x   f  x   f    f   for


x     . So n  f  xn   1  f   for infinite values of n as  is a limit point of this

sequence xn. This is quite impossible since n→∞. Thus the f  x  must be bounded and

f  x  for that matter.

A pitfall: example 20

1
A function f  x   is continuous in the interval ]0, 1] . Is it bounded ? Cite the reason for the same. (
x

the domain is not a closed interval!). Does it contradict the above theorem?

(The interval is not closed !)

C) A continuous real function f(x) in a closed interval [a, b] attains its

maximum and minimum values.

By the above theorem, the sup f = M and inf f = m , say both belong to the range of f as the

function is bounded. Then let f    M and f     m . Suppose the function never

1
attains the value M so M  f  x   0 . Thus from algebra of limits we have is a
M  f  x

continuous function in the same interval and hence bounded by the previous theorem.
1
Since M is the supremum, there is a number  in [a, b] such that f    M  for any
k

1 1
given number k. it follows that  k for any given number k and hence is
M  f   M  f  x

not bounded. This is a contradiction , so f  x  must attain its sup M. Similar argument for

m.

A pitfall: Example 21

The function f  x    x  1, for  1  x  0


 x, for 0  x  1
attains its maximum and minimum values in the interval

[-1, 1] even if it is discontinuous at a point in the interval. There is no point in thinking discontinuous

functions cannot attain maximum or minimum when the function is discontinuous.

Exercise19

Show that the function f  x   x  [ x] does not attain any maximum value even if it is bounded.

Hint. It is not continuous. (graph the function to see that it is a ‘saw tooth wave’).

Exercise20

2 x for x  [1,0]

Take a function f  x    1 .Find its point of discontinuity.
 x  2 for x ]0,1]
Show that the function has both maximum and minimum in the interval, even if it is not continuous.

Exercise21

 2 x  1, for  1  x  0
 x
 2 , for x  0
Show that the function f  x    x
 2  1, for 0  x  1 in the interval [–1, 1] is bounded but never


attains bounds or never reaches maximum or minimum values because it has a

discontinuity at x = 0.

D) If the function f(x) is continuous at an interior point ξ of [a, b] , then

there is a δ such that f  x  and f   have the same sign for all x such that

x   
.

Since f  x  is continuous at ξ, for any given ε > 0, there is a δ > 0 such that

f      f  x   f     whenever x     i.e., x ]   ,    [ .Take f   positive and

  f   , that makes f  x  positive in x ]   ,    [ , in view of the inequality

f       f  x   f    
Then take f   negative and    f   .This makes f  x  negative in x ]   ,    [ .

When these two statements are combined, we can say that f  x  and f   for one δ such

that x ]   ,    [ . This property is very important and has further applications in

deriving important results, in maxima minima considerations, in mean value theorems

and in limits of indeterminate forms, etc.

E) If the function f(x) is continuous at an end point of [a, b] , then there is

a δ such that f  x  and f   have the same sign for all x such that x     .

The argument of the previous proposition may suitably modified to get this result.

Do this as an exercise.

IMPORTANT NOTE :

The two wonderful properties as above do not require the function to be continuous

x   
throughout the interval except at the point  .

f    0 f  x  0 x   
NOTE As a deduction, if then in the small interval .

A) Intermediate value theorem


When the function f  x  is continuous in [a, b] and f  a  and f  b  are of opposite

signs, there is a point ξ in [a, b] at which f    0 . Also ξ = sup S, S being the set of

limit points in [a, b] for which f  x   0


.

Suppose f  a   0 and f  b   0 . By the previous theorem if f  a   0 , there is an interval [a,

a + δ] in which f  x   0 . Thus the number of limit points in [a, b] for which f  x   0 is

infinite and let that subset be S and let ξ = sup S. If possible let f    0 . Then   a , for

f  a   0 . Now if f    0 , then there is an σ such that f  x   0 for all points in x Є [ξ – σ ,

ξ]. Since ξ = sup S, there is an η Є S such that        which implies f    0 ,

but η Є S so that f    0 . We have arrived this contradiction assuming f    0 . So

f    0 is not possible.

Now suppose f    0 . So   b as f  b   0 . Then by the above theorem, there is a ρ

such that f  x   0 whenever x  [   ,    ] , so that we have f      0 i.e.,     S .

But this itself is a contradiction as   sup S . Thus f    0 is also not tenable. This forces
f    0 , which was to be proved. Deduce a similar proof for the case f  a   0 and

f b  0 .

Exercise22

B) When the function f  x  is continuous in [a, b] and f  a  . f  b  < 0, there is

a root of f  x  in [a, b].

Hint. and f  a  and f  b  are of opposite signs.

Exercise23

Does the negation of the above result assert that there is not root of f  x  in [a, b] ? give an example.

Ans. consider y  x 2

So the statement is not true backwards; i.e., the condition is sufficient, but not necessary.

Exercise24 remember
Let a continuous function f  x  defined in [a, b] have a finite number of roots in it, say

x1 , x2 ,......xn in it arranged in increasing order i.e., x1  x2  ......  xn .show that in each of the

intervals ]x1, x2[,]x2, x3[,]x3, x4[,…..the function retains the same sign.

Ans. The occurrence of changing sign in any interval results in existence of another root there.

Exercise25: Show that the function x  1  sin x has a real root.

 3  3
Hint : take a function f  x   sin x  x  1 , then f  0   1, f     . So there is a root in between.
 2  2

Exercise26: Show that x  cos x  0 has at least a real root.

Hint. y  x  cos x is ∞ as x →∞ and is - ∞ as x → - ∞ as cos x  1 .

Exercise27: Show that every odd function f  x    f   x  has at least one real root if it is

continuous.

Hint. f     f    , so there is a root in ]-  ,  [.

Exercise 28: Show that any polynomial of odd degree has a real root. Hint : it direct follows from

above.
Exercise 29: Show that any polynomial of degree 3 (cubic equation) has a real root. Hint : it

direct follows from above

Exercise30: Show that an odd continuous function f  x    f   x  has 0 as a root.

Hint. f  0   f  0   f  0  f  0  0
, so 0 is a root.

 x 2  1 for  1  x  0
Exercise31 : Show that the function f  x   is discontinuous at x = 0 and
  x  1 0 x2
2
for

hence never assumes 0 value in the interval of definition [–1, 2] even if the function

has opposite signs at the end points.

C) When the function f  x  is continuous in [a, b] and f  a   f b  , f  x  takes

every value between f  a  and f  b  .

Take any value k between f  a  and f  b  and design a function   x   f  x   k so that

  a   f  a   k and   b   f  b   k are of opposite signs as k is between f  a  and f  b  .

By the previous theorem, there is an ξ in [a, b] such that     0 . That is,

    f    k  0  f    k . So assumes the value k in [a, b].


D) When the function f  x  is continuous in [a, b] and Maxf  x   M , min f  x   m

the function takes every value between M and m . In fact, [M, m] is the

range of the function.

( do it as an exercise)

f  x
H) A continuous rational function ( which takes only rational(

irrational) number values) must be a constant function.

If no, let f  a   A , f  b   B , and A  B . As the function is continuous, it would assume every

real number value. We can find an irrational number  , A    B , such that f  c    ,and

a  c  b .This is a contradiction since the function takes only rational values. To reconcile the

contradiction we must have A    B . Thus the function is a constant function.

Exercise32: There is one real root of x3  3x  1  0 in [1, 2]. Divide the interval into smaller

subintervals and approach the root correct up to two places of decimal. Hint : apply the criteria of odd

function having a root, repeatedly to smaller and smaller subintervals.

Ans. 1.53
Exercise33: Design a function f  x    x  a   x  b   x so that f  a   a and f  b   b . Find at
2 2

ab
what point the function attains the value a) .b) ab
2

Exercise34:

Show that the function f  x   x  [ x] attains minimum value in any interval of length greater than 1.

Exercise35: If f  x  is continuous in [0, 1] and | f  x  | < 1, Show that f  x  has a fixed point in

[0, 1], i.e. f  x   x there.

I) A fixed point mapping   x   x may be arrived in many ways from an implicit function

g  x, y   0 . A solution of g  x, y   0 is a fixed point of   x   x and vice versa, whereas the

former is easier to solve in some cases. The variables in g  x, y   0 may or may not be

separable in g  x, y   0 and that does not affect choice of   x, y   x , out of g  x, y   0 . In

fact many choices may be available, some may be better than others in finding the solution

and even some may not lead into a solution. Details are dealt in a separate section after

differentiation including Piccard’s method of successive approximation to a root of not

only polynomial equations but any well behaved function.


The results obtained above have far reaching consequences. For example, the intermediate value

theorem guarantees existence of a root in an interval at the ends of which the function has opposite

signs. The last theorem is used in proof of Rolle’s mean value theorem which is the basis of a

number of mean value theorems leading to Taylor’s theorem, which is reconstruction of smooth

functions as power series.

Exercise36: If a spring is stretched, each point of the spring in the old position is carried though a

mapping to a point in the new position. Show that there is a point on the new position of the spring

which is the image of the same point in the old position. ( fixed point mapping)

Exercise37: Show that the equation x5  6 x  3  0 has a root in [0, 1]

Exercise38: Show that the equation x5  18 x  2  0 has a real root. Does it belong to [–1, 1].

f  x    x  1  1, x  1 , show that the fixed points given by


2
Exercise39: For the function

x  f  x   f 1  x  are {0, - 1 }

Exercise40: The cubic equation x3  15 x  1  0 has three roots as you know.

Show that all the three roots lie inside [–4, 4 ].


Exercise41: Solve the equation x3  3x  1  0 by finding values of x for which y  3x  1 crosses y  x3

or by finding the x values for which y  x3  3x crosses y  1

Exercise42: Solve the equation x  2sin x ( three roots)

E) Wavy curve method to analyse an algebraic function

As an application of intermediate value theorem. Take a function f  x    x  1 x  2  x  3 .

Plot the points x = 1, x =2, x = 3 on the number line, where the function takes 0 values. If x <1, all

three factors  x  1 ,  x  2  ,  x  3 are negative, so the value of the function is negative. For 1 < x < 2,

the factor x – 1 is positive, but both of x – 2 and x – 3 are negative so that product of the three factors

f  x  is positive. If the values of x are between 2 and 3, x – 1 and x – 2 are positive but x – 3 is

negative, so that f  x  is negative. For values of x beyond 3, All the three factors are positive and so

the value of the function is positive. This is a consequence of intermediate value theorem. The

rational function is continuous and it takes positive values for x > 1 and negative values for x < 1,( in

whatever small interval you take around x = 1 ), it is bound to have 0 value at x = 1 . Of course,

before applying the rule, we must see that all the coefficients of x be of the same sign.
This does not mean that a continuous function which does not change sign around a point should not

have a zero at the point. For example, look at the figure :

If the function is piecewise continuous, such as f  x  


 x  1 x  2  , or a point where the function
 x  3

does not exist, these points of discontinuity must be excluded from consideration and the analysis
should be separately done for each continuous piece. Look at the graph . This has a discontinuity at x

= 3.

§3.7:Monotonic functions and continuity of inverse functions.

The function y  x 2 is monotonically increasing in the interval [0, ∞[ as x2  x1  0  x2 2  x12

and monotonically decreasing in the interval [0, - ∞[ as x2  x1  0  x2 2  x12 . There are abundant

examples of monotonic functions . They occupy a special place in analysis since they always have

inverse functions. As two different arguments x1 and x2 do not lead to same value of the function (
either y1  y2 or y1  y2 ) so that the function y  f  x  admits of an inverse function x  f 1  y  as

f 1  y  does not have two values for same value of y. This is precisely requirement of or necessary

criteria of a function. Constant functions are trivially monotonic functions also and therefore we

sometimes make a distinction between them by calling them strictly monotonic. Monotonic

functions are continuous in the interval of their definition and so also their inverse functions – a fact

from Real Analysis we can freely use. The proofs are not difficult and rely on the fact that monotonic

functions are bounded and have a maximum and minimum in the domain of definition.

Exercise43: If all the coefficients in the function y  a0 x 2 n1  a1 x 2 n  ...................  an x  an1 are positive.

Show that it has a unique solution for x for every given value of y.

The function is monotonically increasing and thus admits of an inverse function, x  f  y  .

This is a small step towards visualising the fundamental theorem of Algebra – every polynomial has a

root. it is by and large a consequence of continuity of polynomials although generally we read theory

of equations in Algebra. The beauty of properties of continuous functions is that existence of roots of

not only polynomials but transcendental functions are also found out, albeit under certain conditions.

Another small step in the direction is - a polynomial of odd degree with real coefficients has at least

one real root; see a problem given below in theory of equations. Another small step in this direction is
- Every polynomial of even degree p2n  x   a0 x2n  a1x2n1  ...........  a2n  0 whose absolute term is

negative , has at least two real roots following the above problem.

an
Exercise44: A sequence is monotonic and bn  0 . Show that the sequence
bn

a1  a2 x  a3 x 2  .........  an x x
is monotonic.
b1  b2 x  b3 x 2  ...........  bn x n

an
ai xi ai an 1
Hint. Let bn be increasing , then i
   bn 1  a1  a2 x  a3 x 2  ....  an x n 
bi x bi bn 1

a1  a2 x  a3 x 2  .........  an1 x n1


 an 1  b1  b2 x  b3 x  .........  bn x  
2 n

b1  b2 x  b3 x 2  .........  bn 1 x n 1

a1  a2 x  a3 x 2  .........  an x n  a  b  b x  ...b x   b  a  a x  ...a x 


n 1 n 1

1 2 n n 1 2 n n

b1  b2 x  b3 x 2  .........  bn x n  a  a x  ...a x  b  b x  ...b x 


1 2 n n 1 2 n n

Exercise 45: Prove that a linear function f  x   ax  b must be monotonic, either increasing or

decreasing.

Hint : A st line is increasing for +ve slope and decreasing for – ve slope.

§3.8:Continuous linear operators


Exercise46: remember

A linear operator f (function)is such that f  x  y   f  x   f  y  . If it is continuous we

would show that it must be a constant multiple of x, i.e., there is some α, such that f  x    x .

(This is a st line and passes through the origin).

First, it may be seen easily that the operator carries additive identity and inverse to those elements in

the range. Taking x = y = 0, we have f  0  f  0  f  0  0  f  0  f  0  0 .

Similarly putting y = – x , we have 0  f  0  f  x    x    f  x   f   x   f  x    f   x 

Now, if x is a positive integer n, then the result follows for positive integral values of x , as ,

f  x   f  n   f 1  1...n   f 1  f 1  .....nterms  nf 1  x , say,

It also follows for negative integral values of x also, as, if x is a negative integer, put y = – x , so that

f  x   f   y    f  y    y   x .

p  p p p   p  p
If x  , a rational number, then f  p   f  q   f   ...q times   f    f    ...q times
q  q q q  q q

 p
 af  
q
 p  p
Or f  q   qf   . Thus the result is true for rational values of x also. Lastly, if x is a real number,
 q q

let x1 , x1 ,.................xn be a sequence of rational numbers converging to x .

Then f  x1    x1, f  x2    x2 ,................. f  xn    xn Since f  x  is continuous, f  x1  , f  x1  ,................. f  xn 

converge to f  x  . So f  x    x for all real numbers, x.

The importance of the result is quite wide as far as spaces with concept of open sets in topological

spaces, not only real numbers. The result is repeatedly applied below to find other similar results

useful to solve problems.

There it means that the continuous mappings are structure preserving and carry identity and

inverse elements to the identity and inverse elements of the target space respectively.

Exercise47 remember

If a function f  z  is such that f  x  y   f  x   f  y  is continuous at z = 0, show that it is

continuous everywhere.

Hint. lim f  z   lim f  a  h   lim  f  a   f  h   lim f  a   lim f  h   f  a 


z a h0 h 0 h0 h0

Exercise48 remember
If a function f  z  is such that f  x  y   f  x   f  y  is continuous at z = a, show that it is continuous

everywhere.

Exercise49 remember

If a function f  z  is such that f  x  y   f  x   f  y  is continuous at x = 1, then f  x   xf 1

.Exercise50: If f  x  is a continuous function , f  x  y   f  x   f  y  and f 1  1 show that f  x   x .

Ex51: If f  x  is a continuous function , f  x  y   f  x   f  y  and f 1  1show that

2 f  tan x   2 f sin x  ln 2
lim  .
x 0 x 2 f  sin x  2

Exercise52: remember

If a continuous function f  z  is such that f  x  y   f  x  f  y  , show that f  x   a kx


.

Hint. f  x  y   f  x  f  y  loga f  x  y   loga f  x   loga f  y 


If , . So, by a result above

loga f  x   kx  f  x   a kx

Exercise53: remember

If a continuous function f  z  is such that f  xy   f  x   f  y  , show that f  x   k loga x or 0.


Hint. Taking y = 0, we have f  0  f  x   f  0  f  x   0 ; this is one of the possibility

If f  xy   f  x   f  y  f  x n   nf  x 
, then we can prove in the lines of a problem above for positive

integer, negative integer, rational number or real number, whatever is n.

 
  a f  x  x    xn
n
f xn nf  x  f  x f xn
a a  a a
So . But this is possible when and .

f  x   loga x f  x n   log a x n f  x   k loga x


This tantamount to , .Also would lead up to the same result

f  xy   f  x   f  y 
.

Exercise54: remember

If a continuous function f  z  is such that f  x  y   f  x  / f  y  , show that f  z   ln z

Hint : Just try in the above lines.

Exercise55: remember

If a continuous function f  z  is such that f  xy   f  x  f  y  , show that f  x   xa


for some real

number a.
f  xy   f  x  f  y   f  x n    f  x  
n

We have, . ( starting from positive integer, go to negative

integer, rational number and real number as done in problems above).

log a f  x n   log a  f  x    n log a f  x 


n
log a x n  n log a x
This implies . But from law of logarithms.

f  xn  f  x f  xn  f  x  f  xn  xn
log a  log a      x n 1
Subtracting,. xn x x n
x f  x x

f  x n   f  x n 1  f  x   f  x n 1 x   x n 1 f  x   f  x n 1   x n 1  f  x z   x z
Thus

f  x  y  z   f  x  f  y   f  z 
Ex56: If , is a continuous , derivable real valued function on R, and if

f  2   4, f '  0  3 f '  2
and , then find .

 1 
f  x   lim n  x n  1 f  xy   f  x   f  y 
Ex57: Given n 
  for x >0, Show that for y >0.

1 1
Exercise58: f  x  f    f  x f  f  x
If  x  x  and is a cubic expression only, then show that

f  x   1  x3

1 a b c
f  x   ax3  bx2  cx  d f    3  2   d  3  a  bx  cx 2  dx3 
1
Hint : Let . Then  x  x x x x .
3 
a  bx  cx 2  dx 3    ax3  bx 2  cx  d  3  a  bx  cx 2  dx3 
1 1
ax3  bx 2  cx  d 
x x

x3 , x5 , x4 , x3 ....
Expanding and comparing the coefficients of etc. From both sides, d=1, c=0, b=0, a =

1 f  x   1  x3
. So, .

Exercise59: remember

1 1
f  x  f    f  x f   f  x f  x   1  xn
If  x  x  and is of n-th degree polynomial, then show that

Exercise60: remember

 1 1
f  x    x2  2
If  x x find the function.

2
 1 1  1
f  x    x2  2   x    2  f  x   x2  2
Hint : we have  x  x  x

Exercise61 remember

If a continuous function f  z  is such that f  x  y   f  x   f  y  , show that f  x   k


for some

real number k.

f  x  y  f  x  0 f  0  y   f  0  k , say f  y  k
We have for any x, y. Putting x = 0, or, for any y.
1
Exercise62: Show that x  has a root in [0, 1].
2x

Miscellaneous exercises remember

Exercise63

Would a function f  x   g  x  or f  x  .g  x  or f  g  x   necessarily be discontinuous at some point is

one of them is so? Give examples to support your answer.

Exercise64

a) Show that sum and product of two discontinuous functions may be continuous.

b) Show that sum and product of two functions one of which is discontinuous may be

continuous.

Exercise65

Find all the solutions of

a) x3  15 x  1  0 in the interval [-4, 4] (three) b) x3  3x  1  0

c) x  2sin x  0 d) xx  2 e) x 1  x  4

Exercise66
1 1
Discuss continuity of composite function f  x   .where y 
y  y 2
2
x 1

1 1
Ans. The function f  x   y 2  y  2  y  1 y  2 is discontinuous at y  1, y  2 .
  

1 1 1
When y  1 i.e., 1   x  2 . When y  2 , i.e., 2   2 x  2  1  x  .
x 1 x 1 2

Also y is not defined when x = 1. So it is not true that a composite function cannot have more

discontinuities than the 2nd function defining it.

Exercise67: Show that the Keppler’s function f  x   x   sin x , where  is a constant satisfying

0    1is monotonically increasing in R and as such, admits of a unique inverse function.

Exercise68 : Can you give an example of a function which is discontinuous at rational values except

possibly at x = 0 and continuous for irrational values? Examine the function

1 p
 q , for x  q , ratiuonal , q  0, p  0
f  x   for discontinuity.
0, for x  0 or for x irrational



p  p 1 p
At x   0 , any rational number, f    . In any interval around x   0 however small, there
q q q q

are infinite number of irrationals for which f  x   0 so that no interval around x


 p
can be found in which f  x   f     , say, a small number. Hence the function is discontinuous at
q

any non-zero rational point. But if x = c is an irrational number, for any   0

1
we have only a finite number of integers qn such that qn  . Then arranging the qn ' s in ascending

order, and taking the greatest of them  q , for any irrational number x, such that

1
f  x   f  c   0   for the interval xc  . Also , for any rational number in this interval,
q

 p 1 1
f    f  c    0    . Hence the function is continuous at any irrational point.
q q q

Since f  x   0 for x = 0, the same argument follows for x = 0 and the function is continuous for x

= 0.

1  tan 2 x
Exercise69 : Discuss continuity of composite function f  x   .
2  tan 2 x

1  y2 
Ans. The function g  y   is continuous everywhere, but y  tan 2 x is discontinuous at x  n 
2 y 2
2

.
1
1
1  y2 y2
Now, lim f  x   lim  lim  1 .
 y 2  2  y
2 y  2
x  n 
2 1
y2

 1  tan 2 x 
 2  tan 2 x , for x  n 
Hence the function is continuous if defined like f  x   
2
 1 
for x  n 
 2

1  x, 0 x2
Given that g  x   f  f  x   , where
for
f  x   , find the point of
 3  x, for 2 x3
Exercise70.:

discontinuity of g  x  if any.

1  x, for 0 x2
We have, f  x   
 3  x, for 2 x3

1  f  x  , for 0  f  x  2
It is wrong to just substitute and say g  x   f  f  x     .
3  f  x  , for 2  f  x  3

 f 1  x  , 0 x2
is the correct expression for g  x   f  f  x   .
for
Rather, g  x   f  f  x    
 f  3  x  , for 2 x3

 f 1  x  , for 0  x 1

Now, subdividing the interval [0, 2] further, g  x   f  f  x     f 1  x  , for 1 x  2 .
 f 3  x  , for 2 x3

Note that 0  x  1  1  1  x  2 , 1  x  2  2  1  x  3 , and 2  x  3  0  3  x  1 .


 f 1  x  , for 1  1 x  2

So the above expression becomes g  x   f  f  x     f 1  x  , for 2  1 x  3.
 f 3  x  , for 0  3 x 1

Now, if 1  1  x  2 ,then f 1  x   1  1  x   2  x ; if 2  1  x  3 ,then f 1  x   3  1  x   2  x . Also

when 0  3  x  1, then f 1  x   1   3  x   4  x .

 2  x, for 0  x 1

From the last three results g  x    2  x, for 1 x  2 .
 4  x, 2 x3
 for

Now lim g  x   lim  2  x   3 and lim g  x   lim  2  x   1. Hence it is discontinuous at x = 1.


x 1 x1 x 1 x 1

Similarly, lim g  x   lim  2  x   0 and lim g  x   lim  4  x   2 . Hence it is discontinuous at x = 2.


x 2 x 2 x 2 x 2

  1 1
  
 x  1 2  x x 
Exercise71: Show that the function f  x     when x  0 takes every value in the interval
0 when x  0

[ f  2 , f  2] although it is discontinuous at some point in the domain of its definition[–2, 2]. Find out

the point too..


 x  1 ..... for 2 x 0

Hint. Rewrite the function as f  x   0 for........... x  0
 
2
  x  1 2 x for ....0  x  2

It increases from – 1 to 1 in the interval of definition [– 2 ,0], it increases from 0 to 3/2 in [0, 2]

Apply the intermediate value theorem to both intervals [–2 , –1] and [0, 2] .

The function is seen to be discontinuous at x = 0 as lim f  x   1 and lim f  x   0 .


x  0  0  x  0  0 

  x2   1
 when x2  1 at x =1.
Exercise 72 : Discuss continuity of f  x    x2  1 ,
0 when x  1

 1
 x 2  1 , when 0  x  1
2
  x2  1
  when x 2  1  f x   0
Hint : f  x    x2 1 ,    for x2  1
0 when x  1  11
  2 , when 0  x 2  2
 x 1

 1
 x 2  1 , when 0  x  1
2


 f  x   0 for x2  1
0, when 0  x 2  2

1
Now lim f  x   0 ; but lim f  x   lim f 1  h   lim   . Hence the function is discontinuous.
1  h  1
2
x1 x1 h 0 h 0

1
Exercise73: Show that the function has a non-removable discontinuity at x = 0.
x
1 1
Hint ( x  0,  ; x  0,  ; there).
x x

1
Exercise74: Test for show that the function has a non-removable discontinuity at x = 0. Hint (1/x
x

→∞ there).

 1x
 e  1 , for x0
Ex75: Show that the discontinuity of f  x    1x at x = 0 is not removable.
e 1
 0, for x0

Exercise76: Find f  3  if the function is everywhere continuous and is given by

x2  2 x  2 3  3
f  x  for x 3.
3x

 
3  x 2 3  x   2 1 3 .
Hint : lim f  x   lim
x 3 x 3
x2  2x  2 3  3
3x
 lim
x 3 3x
  Now f  3   2 1  3 

since the function is continuous at x = √3.

1 p
 , for x  , ratiuonal , q  0, p  0
q 3
Exercise77: Test for discontinuity of f  x   
q
0, for x  0 or for x irrational



 a  x  x  1
 , for x  0,
  x  x
Exercise78 : Test for discontinuity of f  x   
ln a , for x  0



a     1 a     1
0  h  0 h h h
ah 1
lim f  x   lim  lim  lim  ln a
x 0  h 0   0  h    0  h  h 0  h    h  h 0 h
Hint:

a  0  h  0  h   1 a     1
0 h  0 h
a 1h  1 1
lim f  x   lim  lim  1 lim f  x   lim
x0 h0 0  h   0  h  h0 1  h a x 0  h 0  0  h    0  h 

But

a 1h  1 1
lim  1
h 0 1  h a

 a 2 xx  1
 , for x  0,
 2  x   x
Exercise79 : Test for discontinuity of f  x   
ln a , for x  0


 .

 x   x  x
Hint : use in above.
   
1/ x

 tan  x  , when x  0
Exercise80 : Test for discontinuity of f  x     4  at x = 0.

 a when x0

Hint : lim f  x   e2 . So it is continuous only if a  e2 .


x0

 sin x
 , for x  0
Exercise 81 : Test for discontinuity of f  x    x
1, for x  0

Exercise82 :

 x 2 , for x  ratiuonal ,
a) Express the function f  x    2 in terms of Dirichlet function
  x , for x  0 or for x irrational

  x  
1, for x  ratiuonal ,
0, for x  0 or for x irrational
.

Ans. f  x   x 2  2  x   1

b) Test for its discontinuity. (Take two sequences xn of rational numbers converging to x and

xn of irrational numbers converging to x and prove different limits.

 1x
 e , for x  0
Exercise83: Test for discontinuity of f  x   
0, for x  0

 x
2n

Exercise84 : Test for discontinuity of f  x   lim  sin 


n
 2 
.

 x
0, for sin 1
 x 
2n
2 0, for x  1,
f  x   lim  sin    lim x 2 n  
n
 2  x n
1 for x 1
1, for sin  1
Hint :  2 , as

x
sin  1 , i.e.,
2
Hence it is continuous except when when x is an odd integer. 2m + 1.

Exercise85 : If a function f  x  is continuous in [a, b], and x1 , x2 , x3 .........xn be n number of points

f  x1   f  x2   f  x3   .........  f  xn 
in the interval, then the average  f   for some   [a, b]
n

Exercise 86 : If a function f  x  is continuous in [a, b], and x1 , x2 , x3 .........xn be n number of points

in the interval, then the average n f  x1  f  x2  f  x3  ......... f  xn   f   for some   [a, b]

Exercise 87 : A function has same values at the end points of an interval or values of same sign. It is

continuous in the interval except at one interior point where is has a discontinuity of second kind.

Design such a function , at least the graph of it and show that it can not have a root in the interval.

Exercise 88 : If a function is monotonic in the closed interval [a, b] and takes all intermediate

value between f  a  and f  b  show that the function must be continuous in the interval.
Hint assume the contrary and arrive at a contradiction.

Exercise 89: If F  x  and f  x  be two continuous functions in a closed interval and are equal

at every rational point of it, show that they are equal at every irrational point of it too.

Exercise 90 : Using elementary ideas of vectors from high school, we may define vector functions

of real variable as a variable vector having its components different scalar functions of the real

variable. Mainly, a vector function represents two independent functions taken together.

If u  t   f1  t  i  f 2  t  j and v  t   g1  t  i  g 2  t  j be two non-zero continuous vector functions in the

domain t 0,1 , and u  0   2i  3 j , v  0   3i  2 j , u 1  6i  2 j , , v 1  2i  6 j , show

that the function defined by w  t   f1  t  g2 t   f 2 t  g1 t  in the same domain has a root and u  t  and

v  t  are parallel for that value of t.

Hint : Since w  0  2  2  3  3  5  0 , w 1  6  6  2  2  32  0 and w  t  is a continuous function, it

must have a 0 in 0,1 .


f1  t  f 2  t 
For this value of t, w  t   f1  t  g2 t   f 2 t  g1 t   0 so that    , say, so
g1  t  g 2  t 

 
u  t   f1  t  i  f 2  t  j   g1  t  i  g2 t  j   v t  .

Hence the vectors are parallel for this value of t.

1 x  2x   uv 
Exercise 91 : If f  x   ln , find f  u   f  v  and f  2 
. Ans. f  u   f  v   f  ,
1 x 1 x   1  uv 

 2x 
f 2 
 2 f  x .
 1 x 

a2  a x
Exercise92 : If f  x   , find f  x  y   f  x  y  .Ans. f  x  y   f  x  y 
2

 e1/ x  1
 if x0
Exercise 93: The function f  x    e1/ x  1 has a discontinuity at x = 0. Show that it
0 x0
 if

is non-removable.

 x
Exercise94: Show that g  x   f   can have at most two values.
 x

Ex95:Let f  x  be a continuous function in  0, 2  such that f  0  f  2  .Show that there is a point c

in the interval  0,   satisfying f  c   f  c   


1
  x
f  x    tan   x  
Ex96: Make the function  4   by suitably defining at x = 0.

1 1 1  1 tan x 1 tan x  1


lim  
     lim  1  tan x  x  lim 1   1  tan x  1  e 1 tan x
x
x x0 x
lim f  x   lim  tan   x   
x 0 1  tan x
 x 0   
Hint : x 0 x 0
 4      1  tan x  

 2 tan x  1  tan x x 
lim   2 lim  
e x 0 1 tan x  x
e x 0 x 1 tan x 
 e2 . Hence f  0   e 2
shall make the function continuous.

So far this chapter is devoted to application of concept of continuity to functions in general and

developing a theory of equations ( of polynomials and their roots) in particular. The concepts of limit

and continuity are bare necessity for analysing functions and developing further tools of differential

calculus like, differentiation, tools of integral calculus like integration etc . Thus we would return to the

subject of limit and continuity time and again in course of development of topics in Calculus and

present more problems in continuity and limit again and again e.g., after the chapter of differentiation

( e.g. L’ Hopital’s rule for calculating limits using intermediate value theorems of Lagrange and

Cauchy for calculation of limits, ) , after the chapter of improper integrals ( Leibnitz’ rule for calculating

limits) etc.
The design of concept of limit and continuity depends upon the set in which the concepts are applied

and the algebraic structure in the set. What has been dealt here is the concept of limit and continuity

applied to the set of real numbers. If we consider the set of rational numbers or the set of integers the

description of limit and continuity would be different, though the intrinsic concept remains the same.

For example, the sequence 8, 6, 4, 2, 0, -2 , ... may be regarded continuous if we restrict the concept

to integers, where as the sequence 8, 6,14, 2, 0, -2 , ... may not be. You may be exited to know that

the concepts are applied to wide variety of sets and structures like metric space ( with some sort of

distance function defined in them) or even to topological spaces ( only union and intersection of

member sets defined , no distance function necessary) etc. Broadly the concept of continuity relate

to aesthetic sense of beauty. The reader is warned not to wander away from the set of real numbers ;

this is neither necessary for now nor it is desired; it may create unnecessary confusion. A sequence

of events in a movie show may lead to an expected event ( like ‘they remained happily ever after) or

may finish up to a tragic end or may be left blank letting the observer to imagine.
CHAPTER 4: FURTHER THEORY OF EQUATIONS AND

POLYNOMIALS

Background:

The essentials of theory of equations or theory of polynomials as it may be better called ,are

consequences of properties of continuous functions, although the topic is read generally in the

domain of Algebra. The fundamental theorem of Algebra, the proof of which challenged mankind

for centuries states simply that a polynomial ( understood to be a finite power series in a single

variable devoid of negative or fractional powers and with rational coefficients) must have a root. The

statement shall be shown to be equivalent to saying that even if the coefficients of a polynomial are

chosen randomly, it can be thought to have been arrived at, by multiplying linear factors. For, it would

be proved that the theorem leads to the result that a polynomial of n-th degree has n roots or is the

product of n linear factors, real or complex ones. The great theorem does not give any clue to finding

the roots at all, but a few things are revealed about their nature and some symmetric functions
involving the roots. Standard algebraic methods are employed to solve quadratic, cubic, and

biquadratic equations ( 4th power) and solution of equations of 5th degree and above were shown not

to be possible to find out in standard algebraic methods , even theoretically, as proved by Abel, with

exception of a few special cases. Only the roots can be found numerically, employing methods of

Calculus, by successive approximation to any desired degree of accuracy; e.g., Newton – Raphson’s

method or, Piccard’s iteration (successive approximation) . It is an wonderful experience of writing the

solution instead of finding them. The procedures employ results of Differential Calculus and as such ,

have to be postponed until then. Hence the justification of including theory of equations in Differential

Calculus. Meanwhile we would see what interesting results of utility the theory of equations has to

give us.

In search of proof of the fundamental theorem of Algebra, not only Number System was expanded

from Natural numbers to Integers, Rational numbers, Real numbers, Complex numbers, Vectors,

Quaternions, Matrices, and Tensors, etc. Which were utilised elsewhere such as , in Quantum

Mechanics, Special and General Theory of Relativity and Nuclear Physics etc. Many algebraic

structures were discovered or designed for experiment such as Groups, Rings, Integral Domains,

Fields, Vector Spaces, Modules , Banach Spaces, Hilbert Spaces etc. and many branches of

Mathematics were developed , like, Topology, Functional Analysis, Spectral Theory, Real and
Complex analysis, Differential Equations etc. In short, Mathematics came to be regarded as the back

bone of the modern sciences projecting a developed world view and reforming serious subjects like

Economics and management. Mathematics was no more regarded as mystery or magic in the little

head of the Village School Master but played the role of facilitator for the modern sciences and arts.

To have a closer look at the developments, consider the equation x + a = 0 in the set of natural

numbers. The set is ‘closed’ with respect to the binary operation of addition, i.e., addition of any two

natural numbers gives us a natural number. In other words we can never get out of the set of natural

numbers just by going on adding more and more numbers to any number. The fact that the reverse

process of addition, i.e. subtraction met a road block came trailing behind . we could not subtract a

larger number from a smaller one, even we did not know whether a number would be left if we

subtracted any number from an equal one. In other words, the equation

x + a = 0 had no root for whatever natural number ‘a’. Since one could go 10 steps forward and

return 12 steps backwards, the civilisation decided long ago to agree that the equation x + a = 0 has a

solution or a root at all circumstances. That led us to the set of integers and admit 0 into the number

system. Again the binary operation of multiplication was closed in the set of integers and the linear

equation ax + b = 0 faced a road block in the set of integers while carrying out the reverse process of

division. Since the tailor could make 3 small shirts out of 2 meters of cloth, we understood that the
division process was complete in a fraction once we agreed to include fractions into our number

system . The restriction a  0 was very natural because the linear equation ax + b = 0 ceased to exist

otherwise. Since the division process was complete, arithmetic with fractions were formulated and

also we could write the rational numbers completely in decimal system devising a concept of

recurring decimals. This must have happened many many years after the fire place was installed at

home and wheel was discovered. Since all quadratic equations x 2  a  0 could not be solved in what

we called the rational number system, as the reverse process of squaring cubing etc. faced

restriction, we had to agree to admit diagonals of squares into our number systems, albeit reluctantly,

otherwise we would not have felt any dearth of words and called them irrational numbers. But this

was only negative values of ‘a’ only. The integers and the rational numbers were found to be

countable, i.e., were able to be labelled with natural numbers , discovered by the primitive for

counting; or accounting for missing thing or people in the community. It requires really a mind to

accept that the integers and the rational numbers are equivalent to the natural numbers, who contain

it. More numbers like  , the ratio of circumference of any circle to its diameter were discovered by

great talented people , who were taken to be mad by the commoners, and perhaps therefore , the

numbers were called irrational numbers; also the roots of x 2  a  0 which could not be expressed as

recurring decimals or as fractions or rational numbers. These numbers could not be raised to any
powers to equate them to a rational number, and therefore were branded as non-algebraic or

transcendental, as transcended our existing ideas and beliefs for a long time. They were so large in

number that the natural numbers failed to count them and were branded as uncountable. But to

emphasize their existence because they are so eluding, these numbers, along with the rationals,

were called real numbers. We had to denote the irrational numbers as symbols as the ancient

Romans did with number with little arithmetic formulated. But, we could be able to compare any two

real numbers, whether one is less than, equal to or more than the other. Again, the existence of root

of x 2  a  0 had to be assumed. It required a highly generous and open mind to accept the apparent

paradox that there is no number whose square is negative the squares of positive numbers as well as

squares of negative numbers, both are positive only. So i  1 was accepted as a symbol, its

algebra was formulated and was accepted as a number . Again there was dearth of words in literature

and they were called imaginary numbers or complex numbers. Perhaps we could not have landed on

the Moon had we not accepted them as numbers. The complex numbers p  iq enabled us to solve

all quadratic equation. Nobody could say whether a given complex number , which is not a pure real

number nor a pure imaginary number ,is greater than or less than another complex number. This is

lacking of order completeness property in the set of complex number which is a characteristic in the

set of real numbers, because the numbers are not only in a line, but scattered over a whole plane.
They further helped extending our ideas providing placeholders  p, q  as a single entity containing

two independent components at crossroads, and virtually to vectors  p, q, r  , quaternions

a b c d
 p, q, r, s  , independent rows of vectors e f g h , ultimately tensors. The vectors facilitated
i j k l

placeholders for many 3 independent equations in dynamics written as one, the matrices

made us writing solutions of sets of equations instead of finding them , facilitated quantum mechanics

(which ended the era of dichotomy of wave and particle theories of physics) and the tensors

facilitated theories of relativity and gravitation of Einstein . Still Mathematics is in look out for

placeholders for grand unification theory, possibly deriving the forces of gravitation, electromagnetic,

strong and weak nuclear forces from a common primitive as imagined by Einstein. Newton was a

great integrator for almost knowledge of physical sciences upto his time and could foresee far into the

future . He devised and used Calculus for definition of force and study of gravitation . Einstein, Neil

Bohr, Max Plank, were great integrators, putting an end to dilemma caused by ether. Einstein called

upon the scientific community to redefine time, mass, weight and speed or length. Maxwell was a
great integrator combining forces of magnetism and electricity although Faraday was inventor.

Einstein could use Maxwell’s equations for special theory of relativity because the mathematics was

already available from almost centuries earlier.

While exploring roots or solutions of equations, we looked at the algebraic structures in the sets

where the equation was set up. In the set of integers, the algebraic structure is called a group under

the binary operation of addition. Closure property is assured along with associativity

 a  b  c  a  b  c  . Apart from it an ‘identity’ element of addition, 0 is included in the set and

additive inverse a of every element a is included in the set of integers. Commutativity property

a  b  b  a is ignored in study of groups in general so that the results in non-commutative groups

could be carried over to commutative groups also. Rings are sets which include another binary

operation of non-communicative associative multiplication over a commutative group , a

communicative addition group. Actually the binary operations of addition , multiplication etc. Were

studied in different sets, to explore what results they give; not only addition or multiplication, but any

operation defined consistently to connect two members ( called binary operation); so the numbers

were abstractised as sets and addition and multiplication were abstractised as binary operations and

the marvellous results obtained could be utilised elsewhere. In a ring there is freedom of

a.b  0 with a  0, b  0 , which is denied in another algebraic structure , integral domain (integers
for example, ) where an identity element of multiplication (1) is also provided in the set. More

freedom is assumed in a field which is a commutative ring with identity and inverse of multiplication

included in the set, e.g. the set of rational numbers , the set of real numbers , the set of complex

numbers are all fields. Roots of polynomials (finite power series) are explored in various algebraic

structures (sets with binary operations over them) and if a polynomial is not having a root in a field,

possibility of finding its root in a bigger field is explored; e.g. a quadratic equation not having a root in

the set (field) of rational numbers is having a root is the field of real numbers, a quadratic equation not

having a root is the field of real numbers has a solution in the field of complex numbers. It is left as an

exercise to prove that if surd is a root of a quadratic equation in the field of real numbers , then its

conjugate is also a root. if a complex number is a root of a quadratic equation in the field of complex

numbers, its conjugate is also a root. The statements have been proved below in general for all

polynomials , not only for quadratic equations. By now, it goes without proof that a linear or quadratic

polynomial has at least a root in the field of complex numbers. This is the fundamental theorem of

algebra if stated in context of any polynomial. There is no need to prove it for real and complex

numbers. But we have to assume it only, for polynomials of degree higher than two, for the grand

proof is beyond this book. It is shown below that this assumption enables us to think that any
polynomial whose coefficients have been arbitrarily written ( rational numbers , of course) , has been,

derived from multiplication of linear or quadratic factors only!

§4.1:Theory of equations ( polynomial equations) (remember the

results)

Unless otherwise stated, the coefficients of a polynomial may be understood to be rational numbers.

A polynomial of nth degree equated to 0 gives a rational integral algebraic equation. ‘Integral’

because the rational coefficients may all be multiplied with LCM of their denominators and that does

not cast any difference in the nature or roots of the equation. From the definition we exclude the finite

power series with negative or fractional powers.

Note : Though polynomials are continuous functions, some of the results discussed below do not

require the continuity condition such as division algorithm, rational root theorem etc.. As such they

could have well been dealt in the chapters for Functions. But such result are given here to include

them in ‘ Theory of Equations’.

§4.2:The division algorithm,

Remainder theorem and Synthetic Division


a) The Division Algorithm :

We have known that if (x) is divided by (x)  0, we would have some quotient q(x) and

remainder r(x) i.e. αx   qx .βx   r x  .The quotient and the remainder must be unique. In the

number systems, natural numbers, integers, rational numbers, real numbers and complex

numbers this is the very meaning of the process of division, where r(x) needs be less than

(x).In case of functions in general, we give the process of division this meaning also. There is

a sort of proof for it in case of polynomials where the degree of r(x) should be less than the

degree of (x).

Let (x) = a0 + a1x +….. + anxn and (x) = b0 + b1x +….. + bmxm………………(1)

where an  0  bm

When n < m we have (x) = 0. (x) +(x) , degree of (x) is already < (x).

When n  m, let us assume that the theorem is true for all integers less than n. Construct a

1 x   x   x x  ……………(2)


a n n m
polynomial
bm
Since anxn cancels out from two terms in the right side, 1(x) is a polynomial of degree less

than n. Therefore q1(x) and r(x) can be found out such that

1 x   x q1 x   rx  .........………(3)

x   x x  = x q1 x   rx 


a n n m
Equating (2) and (3) we get,
bm

a 
or, x   x  n x n  m  q1 x   rx   x qx   rx  where q(x) is the
 bm 

expression in the square brackets.

As such the theorem is true for polynomials (x) of degree n.

Still the proof by induction is not complete until we have actually verified the theorem in some

particular case.

When (x) and (x) both are of degree 0, (x) = a0, and (x) = b0,so that x   x   0
a0
b0

When (x) is of degree 0, and (x) is of higher degree, x   0.x   x  ,degree of (x)

being less than that of (x). So the theorem is valid in the two cases. Hence it is valid for (x)
and (x) being of any degree. If q(x) and r(x) are not unique, suppose,

x   qx .x   rx   q' x .x   r' x  .

This implies x qx   q' x   r' x   rx  which is impossible since the degree of the left

side is more than that of (x) which in turn is more than the degree of right side.

So both sides have to be 0 .That proves uniqueness of quotient and remainder.

b) Remainder Theorem for Polynomials.

It states that, if a polynomial P(x) of degree n ≥ 1, is divided by x – a the remainder is P(a).

If the quotient and the remainder on dividing P(x) by x – a be Q(x) and R(x) respectively,

we have, P(x) = (x – a ) Q(x) + R(x) identically for all values of x (division algorithm).

Putting x = a throughout, we get , P(a) = R(a) as desired. This small fact enables us to

have the remainder without actually dividing. So dividing an expression p0 xn + p1 xn-1 + p2 xn-2 + p3

xn-3 +……….+pn by x – a , the remainder shall be got by replacing x with a; i.e., p0 an + p1 an-1 + p2

an-2 + p3 an-3 +……….+pn . So we can always write,

P(x) = (x – a ) Q(x) + P(a) .Evidently x – a is a factor of P(x) or a is a root of P(x) = 0, when

P(a) is 0 and vice versa. The latter statement is called factor theorem; If the expression P(x)
is divisible by x – a, then replacing x with a in the expression makes it equal to 0 , i.e., P(a) =0.in

other words, x – a is a factor of f(x).for this reason a root is also called a zero of the polynomial.

Don’t read this.

[We would use the term ‘ root of a polynomial f  x  ’ in the same sense as root of an equation

f  x   0 . If f  p   k  0, we do not say p is a root of the polynomial nor the equation, but we say k is

the value of the polynomial for x = p. A polynomial can assume any value in the range of f  x  for

corresponding value of x. Remember that the polynomial f  x  is nothing but a function of x, i.e.,

y  f  x  . The equation f  x   0 only gives the values of x for which the polynomial function is 0, i.e.,

intersection of graph y  f  x  with the x – axis.]

Let us get the quotient also by actually dividing.( n  1, for sake of common sense) and we observe,

x - a) p0 xn + p1 xn-1 + p2 xn-2 + p3 xn-3 +….+pn ( p0xn-1+(p1 + ap0)xn-2+(p2+ap1+a2p0)xn-3….

p0xn - p0 xn-1 a

- +

xn-1(p1 + ap0) + p2 xn-2


xn-1(p1 + ap0) - a(p1 + ap0)xn-2

- + .

xn-2(p2+ap1+a2p0) + p3 xn-3

xn-2(p2+ap1+a2p0) - a(p2+ap1+a2p0) xn-3

- + .

xn-3(p3 + ap2+a2p1+a3p0) +………

…………………………………………

…………………………………………

p0 an + p1 an-1 + p2 an-2 + p3 an-3 +……….+pn (written in reverse) = R

If we denote the successive quotients by q0, q1, q2, q3 etc.

q0 = p0 ;

q1 = p1 + ap0 =a q0 + p1 ;

q2 = p2+ap1+a2p0 = a q1 + p2 ;

q3 = p3+ap 2+a2p1 +a3p0 = a q2 + p3 ;


…………………………………..

R = p0 an + p1 an-1 + p2 an-2 + p3 an-3 +……….+pn= aqn-1 + pn

The last one is no quotient as no x to be multiplied with it and it is thus the remainder denoted by R.

Alternatively,

If p0 xn + p1 xn-1 + p2 xn-2 + p3 xn-3 +……….+pn be divided by x – a and let

q1 xn-1 + q2 xn-2 + q3 xn-3 +……….+qn-1 be the quotient and let R be the remainder.

We have, p0 xn + p1 xn-1 + p2 xn-2 + p3 xn-3 +……….+pn

= (x – a )( q1 xn-1 + q2 xn-2 + q3 xn-3 +……….+qn) + R, identically.

Equating the coefficients of equal powers of x from both sides, we get,

q0 = p0 ;

q1 - a q0 = p1, or q1 = a q0 + p1,

q2 - a q1 = p2, or q2 = a q1 + p2,

q3 - a q2 = p3, or q3 = a q2 + p3,

……………………………………
……………………………………

R – aqn-1 = pn,

or, R = aqn-1 + pn = p0 an + p1 an-1 + p2 an-2 + p3 an-3 +……….+pn.

Example1. If f(x) = x5 + 3x4 + 2x3 – 18x2 -15x +31, find f(3)

One has to put x = 3 or x2 = 3 in the equation throughout ,as it seems, putting oneself

in a cumbersome job of calculating (3)5 etc. Instead, let us divide f(x) by x2 – 3 and verify that f(x) =

q(x).(x2 – 3) + 4 for some polynomial q(x) , which we have to calculate by actual division . We need

not actually calculate it, but are compelled to do so, only to know the remainder, 4. Once we know the

remainder to be 4, we can put x = 3 in this equation to get f(3) = q(3).0 + 4 = 4. How simple!

Example2. Find the value of k in x3 +3x2 -12x + k so that x – 3 is one of its factors.

If f(x) = x3 +3x2 -12x + k , f(3) = 33 +3.32 -12.3 + k = 18 + k . this is the remainder

obtained when x3 +3x2 -12x + k is divided by x – 3. this may be otherwise verified by long division too.

Now x – 3 would divide the expression if 18 + k = 0 or k = - 18.


Example3. If a polynomial f(x) of degree > 2 is divided by a quadratic expression x 2 – px + q,

the remainder shall be an expression of 1st degree in x, say Ax + B. If a and b are roots of the

f ( b )  f (a ) bf (a )  af (b)
equation x2 – px + q =0, show that A  and B  .
ba ba

By division algorithm, we can write, f(x) = q(x)( x2 – px + q) + Ax + B = q(x)(x – a

)(x – b) + Ax + B.

Putting x = a and x = b in turn we get, f(a) = Aa + B and f(b) = Ab + B. Solving these two equations for

A and B, we get the result.

Example4. If a polynomial f(x) of degree > 2 is divided by a cubic expression

(x – a)(x – b)(x – c), the remainder shall be an expression of 2nd degree in x, say Ax2 + Bx + C. Find

it.

As per the question, f(x) = q(x). (x – a)(x – b)(x – c) + Ax2 + Bx + C …...................…….(1)

for some polynomial function q(x).

Putting x = a, b and c in turn throughout in this equation, we get,

f(a) = Aa2 + Ba + C ……………….(2)


f(b) = Ab2 + Bb + C …………….(3)

f(c) = Ac2 + Bc + C ……………..(4)

The equations can be solved to obtain,

f (a )( b  c)  f (b)(c  a )  f (c)(a  b) f (a )(b 2  c 2 )  f (b)(c 2  a 2 )  f (c)(a 2  b 2 )


A B
(a  b)( b  c)(c  a ) (a  b)(b  c)(c  a )

f (a )bc(b  c)  f (b)ca (c  a )  f (c)ab(a  b)


C
(a  b)( b  c)(c  a )

§4.3:Evaluation of a polynomial and other functions

Example5 :

1 1
Let f  x   a0 x n  a1 x n1  ..............an1 x  an , n   n   0 satisfy a condition f  x   f    f  x  f  .
x  x

If f  4   65 , find f  6 
1 1 a 
Ans.: f  x   f    f  x  f     a0 x n  a1 x n 1  .....  an 1 x  an   0n  n11  .....  n 1  an 
a a
 x  x x x x 

a0 a a
 a0 x n  a1 x n 1  .....  an 1 x  an  n
 n11  .....  n 1  an
x x x

Comparing the coefficients of similar powers of x from both sides,Coefficients of x n gives

an a0  a0  an  1 . Coefficients of x n 1 gives a0 an1  an a1  a1  a0 an1  a1  a1  an1  0 as an  1 .

Similarly an2  an3  an3  ...  a1  0 and a0  1 . Thus f  x    x n  1

Note : Also a polynomial may be evaluated by division algorithm.

f  x  f a
f  x   a0 x n  a1 x n1  ..............an1 x  an , n  n  0  g  x
We have , if , then xa , where

g  x g  x   b1 x n1  ..............an1 , n  1   n  1  0
is a polynomial of degree n – 1 ( ).

This means, f  x    x  a  g  x   f  a  , f  a  is the remainder when f  x  is divided by x  a . In

special case, if x  a is a root of the polynomial, f  a   0 .

Now, f  4   4n  1 .This is obviously not true for the minus sign as n   n   0 , so,

f  4   4n  1  65  n  3 . Hence, f  x   x 3  1  f  6   63  1  217

We give below examples of evaluation of functions which are not necessarily polynomials.

Example6
n
If f  x   f  y   f  x  y  evaluate  f  r  , for r being a positive integer 1  r  n .
1

Ans.: Putting x = r – 1 , y = 1, in f  x  y   f  x   f  y  we have f  r   f  r  1  f 1 .

Similarly Putting x = r – 2 , y = 1 f  r  1  f  r  2   f 1 , so f  r   f  r  2   2 f 1 .

Extending the argument further, f  r   f  r  3  3 f 1 ………. f  r   f  r   r  1    r  1 f 1 .

n n n
n  n  1
Thus  f  r    rf 1  f 1  r 
1 1 1 2
f 1

Exercise1: Determine ‘a’ for which 3 is a root of f  x   x  4 x  ax  6


3 2

Exercise2: Determine ‘a’ for which 1 is a root of f  x   a x  3ax  2


3 3 2

§4.4:Synthetic division (Due to Horner).

The above process of finding successive terms of quotient and reminder when the expression is

divided by x – a may be briefly written

as follows : p0 p1 p2 p3 ……………………….. pn – 1 pn

aq0 aq1 aq2 ……………………….aqn – 2 aqn – 1


q0 =p0 q1 q2 q3 …………………qn – 1 R

The algorithm goes like this : put the first term of the quotient q 0 = first term of the divisor p0 .

Then put the second term of the quotient q1 = second term of the divisor p1 + aq0 . Proceed in this

manner until the last term arrived , which gives the remainder and = R = aqn-1 + pn .

The algorithm may be slightly changed if the divisor is x + a instead of x – a. Then put – a throughout

for a and it is done.

Example6 : Divide 3x4  11x3  23x 2  21x  10 by x  3 and find the quotient and remainder.

Taking a = -3 , q0 = p0 ; q1 = a q0 + p1, q2 = a q1 + p2,...................... R = pn + aqn-1, are shown as under.

3 11 23 21 – 10

–9 –6 –51 90

2 17 –30 80

The quotient is 3x3  2 x 2  17 x  30 and the remainder is 80.

Exercise3

Divide x5  5 x 4  9 x3  6 x 2  16 x  13  0 by x  3 , find the quotient and the remainder.


Ans. Quotient is x 4  8 x3  33x 2  106 x  302 , remainder is 1009.

Exercise4

Find b in the equation 2 x 4  7 x3  8 x  b  0 , if it is divisible by x  3 Ans. 3.

Exercise5

2  1 is a zero of f  x   x  14 x  1
6 3
show that

The algorithm can be extended suitably for rational Algebraic functions of the form

P(x) / Q(x) where degree of Q(x) is less than that of P(X). See a chapter afterwards.

Exercise6: Divide f  x   x  5x  3x  2 by x  1, x  2 and show that f  1  1, f  2  4


3 2

Exercise7: Show that the greatest common divisor of x4  x3  6 x2  2  2 and x3  x2  5x  2 by

dividing the former by later, the latter by the remainder and so on as GCD is extracted and show that

it is x + 2 so that x = – 2 is a common root. divide the expressions by x + 2 and find the other roots.

§4.5: Prove that a polynomial of n-th degree has n roots,


(Assuming the fundamental theorem of Algebra, that every polynomial equation

has at least one root, )

Let the polynomial be f  x   p0 x  p1x  ............. pn1x  pn which has a root say α, then
n n1
x – α is a

factor so that f  x    x      x    x     q1 x n1  q2 x n2 ...........qn1 x  qn  where   x  is a polynomial of degree n –

1 . applying the fundamental theorem of Algebra to   x  , it

has a root β, and x – β is a factor of   x  . So   x    x     x  where   x  is a polynomial of

degree n – 2 ; so that f  x    x     x    x    x     x  . Ultimately we shall get,

p0 x n  p1 x n 1  ...  pn 1 x  pn  f  x    x    x    ...........  x    product of n linear factors like

 x    ,  x    etc. ; the last p0 taken for equating coefficients of x n in both sides. This is linear

factor theorem . both the theorems, the fundamental theorem of Algebra and linear factor theorem

hold for any coefficients real or complex, of the polynomial.

This means that whatever polynomial you may write with randomly chosen real coefficients, must be,

nothing but product of real linear factors or irreducible quadratic factors. In the latter case, each

irreducible quadratic factor is a product of two complex factors, the sum of the roots and products of
the roots of which are real numbers. This means that the two complex roots must be conjugates of

each other !

The fact that a polynomial has at least a linear factor is equivalent to saying that it could be divided by

that factor to give a polynomial of one degree less as quotient and leave no remainder. In effect, any

polynomial can be reduced to another polynomial of one degree less, by dividing it by a suitable linear

factor. Is not that interesting?

§4.6:A polynomial of degree n, f  x   p0 xn  p1xn1  ............. pn1x  pn cannot have

more than n roots.

By the preceding exercise, f  x  has n roots, say α, β, ...........  , so it is product of n linear factors

and the constant p0 , i.e., f  x   p0  x    x    x    .............  x    . If the function has another root

 , say, then f    0 . But f    p0           .............     ; the left side is 0 but not the

right side as none of the factors is 0. This is a contradiction . So f  x  has exactly n roots, real or

imaginary.

(Unique factorisation theorem)


In other words, a polynomial of n-th degree with it rational coefficients chosen arbitrarily, can

be shown to be the product of n linear factors, always, in unique manner.

§4.7: If a polynomial of degree n, f  x   p0 xn  p1xn1  ............. pn1x  pn has

more than n roots it must be an identity, i.e., it must be identically 0.

The conjecture f    p0           .............     would still be true when f    0 in one

situation, when p0  0 . In that case, f  x  is identically 0 , since

f  x   p0  x    x    x    ............. x     0 .

§8:The above statement is equivalent to saying that every polynomial of n-th degree

f  x   p0 xn  p1xn1  ............. pn1x  pn can be decomposed into linear factors in only one way, i.e.,

f  x   p0  x    x    x    ............. x    . (prove). This is also called unique factorisation

theorem.

Exercise8: Find the equation whose roots are – 3, – 1 and 5/3 .

Ans. 3x3  7 x 2  11x  15  0

Exercise9: If ax 2  bx  c  0 has three distinct roots, show that a  b  c  0


Hint. Substitute roots p, q, r in the equation and derive a  b  c  0

x2  a2 x2  b2 x2  c2
Exercise10: Take f  x      1 . Show that a, b, and c are three
 c  a  a  b   a  b b  c  b  c  c  a 

distinct roots of this equation and hence f  x   0 is an identity.

Hint. Find f  a  , f  b  , and f  c  , each is shown to be a root. A quadratic equation having

three distinct roots must be an identity.

Exercise11: Take f  x 
 x  a  x  b  c 2  x  b  x  c  a 2  x  c  x  a  b2
    x2 . Show that
 c  a  c  b   a  b  a  c   b  c b  a 

f  a   f b   f  c   0 and hence f  x   0 is an identity.

§4.8:Show that a polynomial of odd degree with real coefficients has at

least one real root.


Take the polynomial p  x   a0 x  a1 x  ...........  a2n x  a2n1  0 . The function p  x  is continuous
2 n1 2n

throughout and the limit lim p  x    and lim p  x    if a0  0 .Then we could find real numbers
x  x

a, b, such that a < b and p  a   0 and p  b   0 as in monotonically increases from - ∞ to ∞.

(prove it) So the function p  x  has a root between a and b.

Proof Observe that p  0  a2n1 p     p     


, also and . Then, since the polynomial is

 and 0
obviously continuous, there is a root between , i.e., a negative root

p  0  a2n1 p  0  a2n1 p    
if is positive. Also when is negative, since , there is a root between

0 and  p  0  a2n1  0
, i.e. a positive root. If , we have nothing to prove.

§4.9:Every polynomial of even degree p2n  x   a0 x2n  a1x2n1  ...........  a2n  0

whose absolute term is negative , has at least two real roots.


Proof Assume p2 n  0   a2 n a0  0 p2n   
is negative, and . Then is positive, so there is a root

0 and  a0  0 p2n    p2n  0  a2n


between , i.e., a positive root. Also if , is positive and is

 and 0 a0  0
negative, there is a root between . Thus there are two real roots when

§4.10:Take any polynomial p2n  x   a0 x2n  a1x2n1  ...........  a2n  0 of even degree.

If it attains at least one value , the sign of which is opposite of that of

a0, then show that it has at least two real roots.

Proof The above exercise is a particular case of this.

Now, a0 and p2n   , p2n   have the same sign always. If p2n   has opposite sign of

a0 for some  ,then p2n   and p2n    have opposite signs; and also p2n   and p2n    have

opposite signs. So there is a root between  and  and also between  and  .

§4.11: If a polynomial p  x  , of only odd powers of x has all its

coefficients of same sign, it has no other real root except x = 0.


Hint. Such a polynomial is an odd function evidently; f   x    f  x  . It does not change sign in

any interval not containing 0 in it.

§4.12: If a polynomial p  x  , of only even powers of x has all its

coefficients of same sign, it has no real root.

Hint. Let all coefficients have + signs. The function is increasing as x increases and increases as

x decreases also. So the only possibility of having a real root is either f  0  0 , or negative. But it

cannot be so, as f  0  is also positive as per question. Similarly prove the statement for all

coefficients negative.

Exercise12: Show that a continuous function never equal to 0 in any interval , never changes

sign in that interval. ( this is not only for polynomials for all functions).

Hint. If it changed sign, it must be 0 somewhere in the interval.

Exercise13: If the coefficients of an algebraic polynomial are all coefficients positive it has no

positive root.
Hint. If a polynomial f  x  has a positive root α, it must have a factor x – α . Let

f  x     x  x    . The function f  x  must have one more change in sign than   x  . If   x  has no

change in sign at all, then f  x  must have one change is sign. But that is a contradiction of the

conditions of the question; f  x  has no change in sign. Hence the proof.

Explanation. Note changes in sign in   x  e.g., the product of   x  and  x    is displayed in

symbols as below.

    
 
________________
    
    
_________________
     

Note that the changes in sign in   x  (top line) is 3, + to -, - to + and + to -. In the final product, (

bottom line) the changes in sign is at least one more than 3 including the ambiguous place  which

arises when we add a term with + sign with another with – sign or vice versa. There cannot be less

changes in sign than those present in the original equation. The increase in changes of sign shall be

minimised if all the  shall be taken as + ( or -). Also observe here that in the final result, ambiguity
in sign i.e.,  arises where there is continuation of sign in the original polynomial and the signs before

and after an  sign are unlike and a change in sign occurs at the end. This observation shall help in

understanding that we can safely assume either + or – sign throughout in place of a  sign.

§4.13:Descartes’ Rule Of Signs.

The number of positive roots in a polynomial of n-th degree f  x   0 cannot exceed the

number of changes in sign in f  x  .

Suppose that a polynomial   x  is formed by only negative and imaginary roots of f  x  . If a is a

positive root of f  x  , multiplying   x  by x  a adds one more change in sign into  x  a    x  . Hence

f  x  would have at least as many changes in sign in it as the number of positive roots in it. So the

number of positive roots cannot exceed the number of changes in sign in f  x  . This is the famous

Descartes’ Rule of sign in theory of equations.

The particular cases are : (prove as exercises and remember )


a) Every polynomial of odd degree has at least one real root of a sign opposite to that of its

absolute term.

b) Every polynomial of even degree with absolute term negative has at least two real roots.

c) If all the coefficients of a polynomial are positive it has no real root.

d) If a polynomial of odd power with all coefficients of same sign, it has no positive root. Etc.

All of the four have been proved above independently.

e) It easily follows that a polynomial with no changes in sign in its terms written in descending

order of the powers of x has no positive roots. Thus a polynomial consisting of positive terms

only or negative terms only has no positive roots.

f) If the coefficients of even powers of x are all of the same sign and the coefficients of odd

powers of x are all of the opposite sign to that of even powers, the polynomial has no negative

root.

g) If the polynomial has only even powers of x with coefficients of all terms having same sign , it

has no real root.

h) If the polynomial has only odd powers of x all with same sign, it has a root at x = 0 and has no

other real root.


§4.14:The number of negative roots in a polynomial of n-th degree

f  x   0 cannot exceed the number of changes in sign in f   x 

(prove it as an exercise).

The above problems are easily solved using properties of continuous functions and they are

important as they are used in theory of equations.

Thought

There is still an eluding aspect of Descartes’ Rule of Signs. The number of positive roots of the

polynomial f  x  is equal to the changes in sign in the polynomial or less than it by an even number

and a similar statement about negative roots of the polynomial.

Exercise14:Show that the polynomial f  x   x  2 x 1 and g  x   x  x  2x  x  2 have at least


4 3 5 3 2

two imaginary roots each.

Hint. Employing Descartes’ rule of signs, show that it cannot have more than one positive root,

and more than one negative root. Thus it must have at least two imaginary roots.
§4.15:If a  b is a root of polynomial equation f  x   0 , then a  b is one

root too.

If a  b is a root and a  b is not a root, let the remainder obtained by dividing f  x  by

 x  a  b  x  a  b    x  a   b be Rx  S , R, S being constants, as the degree of remainder


2

must be less than that of the divisor. If Q be the quotient, we have the identity from remainder

theorem, f  x   Q  x  a   b   Rx  S . Since a  b is a root of f  x  and also a factor


2
 

of  x  a
2
 
 b , Rx  S  0  R a  b  S . Since a surd R b cannot be equal to a rational number

Ra  S , both of them have to be 0. Then R  0 and  Ra  S  0  R  0 and S  0

.Thus x  a  b is a factor of f  x  as   
f  x   Q  x  a   b   Q x  a  b x  a  b , or a  b is a root of

2

f  x .
§4.16:Just in the same manner, show that if a  ib is a root of polynomial

equation f  x   0 , then a  ib is one root too.

Hint Use the fact that an imaginary number cannot be equal to a real number, if so

both must be 0.

If a + ib is a root of f(x) = a0xn + a1xn-1 + a2xn-2……….an , x – a – ib is a factor of f(x).

Product of x – a – ib and x - a + ib, its conjugate becomes {(x-a)2 +b2}.

Divide f(x) by {(x-a)2 +b2}, let the quotient be Q(x) and remainder be Rx +t, naturally of one

degree less than the divisor. Or, f(x) = Q(x) {(x-a)2 +b2} + Rx + t = 0

Putting x = a + ib makes f(x) = 0 as a + ib is its root and also {(x-a)2 +b2} = 0 as {(x-a)2 +b2} =

(x – a – ib )(x – a + ib).So that, R(a + ib) + t = 0  Ra + t + iRb = 0  Ra + t = 0 and iRb = 0

 R = 0 as b  0, which further gives, R.0 + t = 0  t = 0

(If not, Ra + t = iRb, which is impossible as an imaginary number cannot be equal to a real

number).R = 0 and t = 0 make (1)as f(x) = Q(x) {(x-a)2 +b2} Or,f(x) = Q(x)( x – a - b)( x -a +b)
which shows ( x -a +b) is a factor of f(x) and a – ib is its root. Hence the proof.

( The latter result for complex numbers is true even if a 0, a1, a2 ……… an etc. are not

rationals)

The above results show that if surds or complex numbers are roots of a polynomial, they occur

in pairs.

A pitfall :execise15

Show that  
3 is a root of x 2  2  3 x  2 3  0 whereas  3 is not. Why?

The equation is not with rational coefficients. In fact it is as plain as this – if conjugate quadratic surds

are not are not in the factorisation of f  x  , it cannot be a polynomial whose coefficients are rational

numbers. Use this plain reasoning to prove the following exercise.

§4.17:Except conjugate surds no other irrational (transcendental)

number can be a root of a polynomial.


In fact, transcendental numbers are defined as those which are not roots of any polynomials.

Hint. A polynomial is defined to be having rational coefficients and sum or product of two

different transcendental numbers cannot be rational number. ( Suppose a polynomial of which all

roots are rational except only two).

Exercise16: Find the equation whose two roots are 1  2 and  1  3i

Ans. x 4  5 x 2  22 x  10

§4.18: Rational root theorem : remember

To investigate about a rational root p/q of a polynomial a 0x n +a1xn-1……….an = 0 with a0, a1,….an

being integers and p and q have no common factors except 1. Consider some rational number

p/q is a root of the polynomial. Then the theorem states that p is a factor of an and q is a factor of

a0.

The proof is simple. Replace x by p/q and rewrite the eqn. as


p(a0pn-1 +a1pn-2q + a2pn-3q2 +……….……….an-1) = - anqn. This shows p is a factor of left side and

hence a factor of right side. But it does not divide qn; so it must divide an. For, if there are no common

factors between p and q, there are no common factors between p an qn. Similarly it can be shown

that q is a factor of a0.

Example6 :

Determine whether the equation 2 x3  x 2  9 x  4  0 has any rational root. if so, completely solve the

equation.

p
Factors of the first term are ±1, ±2. Factors of the last term are ±1, ±2 and ±4. If is a rational root
q

then q divides 2 and p divides – 4. So the possible candidates for rational roots are

1 1
1,  , 2, and  4 . Now we could exclude 1, – 1, and as they can be tested by direct
2 2

1
substitution. But  is confirmed to be a root as may be tested by direct substitution in the equation
2

 1
or by synthetic division by x     . Synthetic division is definitely easier process than direct
 2

substitution. Now we should not test for other roots by synthetic division , as we can divide the

 1
equation by x     , i.e. we have already got the quotient in the process of synthetic division to be
 2
x 2  x  4  0 , which we call reduced equation. The last equation is merely a quadratic equation and

can be easily solved to get the roots


1
2
 
1  17 .

§4.19:Integer roots as a special case:

If a0 = 1 in the polynomial above, then qn must divide 1 as it divides a0 when the eqn. has a rational

root. This implies q must divide 1; but 1 has no other factors other than 1. So

q = 1 and the root p/q = p , which is an integer.

Example7 : If a natural number has a rational n-th root, the root must be a natural number.

remember.

Let xn = N, where n and N are natural numbers and x = p/q, a rational number. Then the

polynomial equation xn – N = 0 satisfies all conditions of the special case of the rational root theorem

as above. Hence it must be an integer.


Better alternative analysis for integer roots is given afterwards. Instead of testing for scores of

integers for possible roots, we could narrow down the search range within some upper bound and

lower bound and in fact there is a theorem to this effect given below(Location theorem). Further, you

would soon know that the ratio of the coefficients of the last term to the first is  product of the roots;

from which we may pick up only integer divisors of this ratio which fall within the upper and lower

bounds.

Exercise17:

If f  x   a0 x n  a1 x n1  a2 x n2  .......  an2 x 2  an1 x1  an and the coefficients are all integers, and if

both f 1  a0  a1  a2  .......  an2  an1  an and f  2   a0 2n  a1 2n1  ....  an1 21  an are odd numbers,

the function has no integer root.

Hint: Assuming the contrary, let f  N   0 for some integer N. Then , by division algorithm, or factor

theorem for that matter, x  N divides f  x  ; then let f  x    x  N  g  x  , where g  x  is also a

polynomial of integer coefficients. Now f 1  g 11  N  and f  2  g  2 2  N  . since both

f 1 , f  2  and one of 1  N , 2  N must be even ( as both cannot be odd), which implies that one of

f 1 , f  2  is even, which is contrary to our assumption. Hence f  N   0 .


Exercise18:If f  x  is a polynomial with integer coefficients and for the integers a, b and c,

f  a   b, f  b   c, f  c   a in cyclic order, and then a  b  c .

Hint : Let f  x   a0 x n  a1 x n1  a2 x n2  .......  an2 x 2  an1 x1  an , coefficients being integers.

so that f  a   a0 a n  a1a n1  a2 a n2  .......  an2 a 2  an1a1  an  b ,

f  b   a0b n  a1b n1  a2b n2  .......  an2b2  an1b1  an  c ,

f  c   a0c n  a1c n1  a2c n2  .......  an2c 2  an1c1  an  a .

Then a  b is a factor of f  a   f  b  , i.e., f  a   f  b    a  b  g  a , b  ,where g  a , b  is some function

of a,b; and it is an integer. But f  a   f  b   b  c from above.

As such we have  a  b  g  a , b   b  c ,  b  c  g  b, c   c  a ,  c  a  g  c, a   a  b . Multiplying we get,

 a  b  g  a, b  b  c  g  b, c  c  a  g  c, a   b  c  c  a  a  b  .

If a  b  c  a , we have g  a , b  g  b, c  g1  c, a   1 , which cannot be true unless each of them is 1; i.e.,

g  a , b   1, g  b, c   1, g  c, a   1 , since they are integers. This forces

ac a b bc
a  b  b  c  c  a . Thus, b  ,c  ,a  . Three different numbers cannot be arithmetic
2 2 2

mean of one another unless they are equal, i.e. common difference is 0.
So, a=b=c.(Alternatively,).  a  b  0  a  b a  b  b  c  c  a  a  b 
b  c    c  a   b  a
2 2

Example8 :

If f  x  is a polynomial having integer coefficients and f  a   f  b   f  c   f  d   k , where

a , b, c, d , k are integers, and if f  e   k  2 for some integer e, and k is an integer, then show that

a bc d .

Hint : Let g  x   f  x   k . Then g  a   g  b   g  c   g  d   2 , as f  a   f  b   g  a   g  b  and

f  a   f  b   0 . So g  x  is divisible by x  a , x  b, x  c, x  d ,

so g  x    x  a  x  b  x  c  x  d  h  x  . Now g  e  is an integer as g  e   f  e   k  2 .

We have g  e   f  e   k  2 , and then g  e    e  a  e  b  e  c  e  d  h  e   2 . So h  e  is an integer

and so  e  a  ,  e  b  ,  e  c  ,  e  d  are divisors of 2. Now 2 cannot have 4 different divisors, as it

would be product of all of them. so the factors  e  a  ,  e  b  ,  e  c  ,  e  d  are not different from each

other. Thus each one of them is 0. this forces e  a  b  c  d , if there is one such e .

§4.20: Location theorem remember


If f  x  be a polynomial, even with coefficients as real numbers with f  a  f b   0 ,

i.e. f  a  and f b  are of opposite signs, there is a root between a and b.

This does not say that there is only one root between a and b. Nor it says that the function has no

root if it does not change sign e.g., vertex of a parabola( quadratic equation) touching the x – axis is

a double root ( two coincident roots) .

This is nothing but intermediate value theorem for continuous functions. Its utility lies in separating the

intervals in which roots of the equation are located so that they can be hunted out later on by

successive approximation, by subdividing the intervals to smaller subintervals and locating the root to

one of the subintervals and repeating the process for better approximation.. The theorem also applies

to transcendental functions.

Example9

Show that the function x3  2 x  4 has a root in [1, 2] and find it correctly upto 2 places of decimal.

Denote the function as f  x  and note that f 1  1, f  2  8 . Subdivide the interval [1, 2] into 10

subintervals [1.1, 1.2], [1.2, 1.3] etc. And note that again there is a sign change as we find

f 1.1 and f 1.2 . The functional values may be easily calculated using synthetic division as
explained in examples above. So the root lies between 1.1 and 1.2. Again divide the interval [1.1,

1.2] into 10 smaller subintervals and note that again there is a change in sign between

f 1.17  and f 1.18 . So the root lies in this interval.

This is just to illustrate that roots can be approximated in this manner crudely. But there are special

tools like Piccard’s iteration method, Newton-Raphson’s method to approach the roots in systematic

manner given after differentiation chapter. Also there is a method to narrow down the search to an

interval of upper bound and lower bound of roots given hereafter.

Exercise19:

Show that x 4  5 x3  3x 2  35 x  70  0 has a root between 2 and 3 and another between – 3 and – 2 .

Exercise20:

Find out that x 4  12 x 2  12 x  3  0 has a root between – 4 and – 3 another between 2 and 3 .

Exercise21: Find out that 10 x3  17 x 2  x  6  0 has a root between – 1 and 0 .

Exercise22:

Find out that x5  5 x 4  20 x 2  19 x  2  0 has a root between – 5 and – 4 another between 2 and 3
.§4.21: Evaluation of root of a polynomial

We can use the synthetic division to test whether a particular number ‘a’ is root of the

polynomial. The remainder found in the synthetic division of the polynomial by x – a would be 0 if ‘a’

is a root.

§4.22: The following points may be carefully noted for testing

whether a given number is a root.

a) The missing powers if any, in the polynomial pertain to 0 coefficients and place must be

reserved for them in the first line of the synthetic division.

b) If the remainder is 0, then ‘a’ is a root of the polynomial by factor theorem. A suspected

root may be tested in this process.

c) While testing whether a particular integer is a root of the polynomial or not, we can write

the polynomial in the reverse order an  an 1 x  an 2 x 2  ...........  a0 x n and stop as soon as a


fraction appears in the quotient (the third line). Obviously the divisor in question shall

not completely divide the polynomial and the process may be stopped there.

d) Horner’s method for evaluating a polynomial at any point x = x0.

To find the value of a polynomial an  an 1 x  an 2 x 2  ...........  a0 x n at a particular point x0, there is

a speed up process due to Horner similar to the process of synthetic division. The polynomial

  
is written as f  x   a0  x a1  x a2  x  a3  x  a4  .......  . A number of intermediate steps

calculate the intermediate quotients bn , bn1 , bn2 ,.................b2 , b1, b0  f  x0  , whose last one

(remainder) is b0  f  x0  . The method may be at once understood from the table

an an 1 an  2 ............. .............. a0
0  x0bn  x0bn 1 ............. ..............  x0b1
___ ____ ______ _______ _______ _____ ,
bn bn 1 bn  2 .............. .............. b0  f  x0 

where bn  an , bn1  an1  x0bn , bn2  an2  x0bn1 ,...........b1  a0  x0b1 , b0  f  x0 

Example10:

Find f  3 if f  x   2x5  x4  3x3  x  3


2 1 3 0 1 3
0 6 15 36 108 327
___ __ ___ ____ ____ ______________
2 5 12 36 109 b0  f  x0   324

Misc. Exercise

23) Find f  1 if f  x   x 4  8 x3  x 2  4 x  9 ans. – 26 .

24) Find f  3 if f  x   2x4  5x3  12x2  36x  109 ans. 622 .

25) Find f  1 if f  x   x 4  8 x3  x 2  4 x  9 ans. –3.

§4.23:Horner’s method for evaluating f  x  h  from f  x 

Take f  x   p0 x  p1x  .....................  pn1x  pn


n n1
Changing x to x  h  y , say, or, x  y  h , the above expression becomes,

q0 y n  q1 y n 1  .....................  qn 1 y  qn .

Or, q0  x  h   q1  x  h   .....................  qn1  x  h   qn .


n n 1

The few lines above reveal an useful information -

qn is the remainder when f  x  is divided by  x  h ; and the quotient is

q0  x  h   q1  x  h 
n 1 n2
 .....................  qn1 .

Similarly, qn 1 is the remainder when the last expression , the quotient is divided by  x  h  , and

the second quotient is q0  x  h   q1  x  h 


n2 n 3
 .....................  qn  2 .The process may be

repeated in a condensed manner by employing synthetic division. The process may be

understood from the following example.

Example11:

Find f  x  3 from f  x   2x  x  2x  5x 1
4 3 2

Dividing the expression successively by x  3 is shown below:


2 1 2 5 1
6 15 39 132
5 13 44 131  q4
6 33 138
11 46 182  q3
6 51 ,

17 97  q2
6
23  q1

Thus f  x  3  2 x  28x  97 x  182x  131 .


4 3 2

Exercise26: Find

a) f  x  3 if f  x   x 12x  17 x  9x  7 x 4  37 x 2  123x  110


4 3 2
ans.

b) f  x  1 if f  x   2x 13x  10x 19 ans. 2 x 4  8 x3  x 2  8 x  20


4 2

c) f  x  4 if f  x   x  10x  39x  76x  65 ans. x 4  6 x3  15 x 2  12 x  1


4 3 2

d) f  x  4 if f  x   x  16x  72x  64x 129 ans. x 4  24 x 2  1


4 3 2

e) f  x  h   f  x  h  if f  x   px  qx  rx  d
8 5

ans. 16axh  x 6  7 x 4  7 x 2 h 4  h6  2bh  5 x 4  10 x 2 h 2  h 4   2ch


§4.24: Relation between roots and coefficients.

If 1, 2, 3, etc. are roots of a rational integral algebraic function ,

f(x) = a0xn + a 1xn-1 + a2xn-2……….an, then

1+2+3+…n= - a1/a0; 1 2+2 3+...n-1 n = a2/a0;

123+234…….+n-2n-1n = a3/a0, . ………..

and so on upto (123……n-2n-1n) = (-1) n+1 an/a0

If 1, 2, 3, etc. are given to be roots of

f(x) = a0xn + a1xn-1 + a2xn-2……….an,……… ……….(1)

Then f(x) can be written as

f(x) = a0(x - 1) (x - 2) (x - 3)………….. (x - n)

= a0{ xn - (1+2+3+…n) xn-1 + (1 2+2 3+...n-1 n) xn-2

- (123+234………..n-2n-1n)xn-3 and so on………………(2)


(1) and (2) both represent the same function differently and as such, an identity. Equating the

coefficients of similar powers of x from both sides we get the result. Plainly speaking, the sums

of products of the roots, taken one, two, three,…… at a time equals to the ratio of the 2 nd,

3rd,4th etc. coefficients respectively to the 1st coefficient with alternate signs – and +.

The result that roots that are quadratic surds or complex numbers involving imaginary

numbers must appear in pairs may be better understood now.

Suppose  + or  +i is a root of the polynomial. Then a surd or imaginary number must

 a1 a
appear both in the sum of the roots and in the product of the roots n . But these two
a0 a0

ratios are rational real numbers as the is the way we have defined the polynomials.( for

consideration of rational or irrational roots we consider polynomials with rational

coefficients and for real or imaginary roots we consider polynomials with real

coefficients.). Hence if a quadratic surd or imaginary complex number is a root of a

polynomial, its conjugate must be also a root so as to make the sum and product of the roots

rational / real; which are simple ratios of coefficients.

It seems that we can conveniently solve any rational integral algebraic equation with the help

of this result. But no. To solve the equation, we have the n simultaneous equations involving
1, 2, 3……n-2, n-1, n and to solve them we have to eliminate all but one of them, say 1 .

This will result in a n-th degree in 1 so we are back to square one again with no gain. But the

insight gained helps finding the roots in special conditions, say if one root is give, or if roots are

in arithmetic progression etc. ; depends on the situation.

§4.24a: Relation between roots and coefficients may be used to find

out roots of a polynomial when one of the roots is known or

under some given condition.

Follow the exercises .

Exercise27 : Find all the roots of the polynomials if two of the roots are equal in each case.

a) f  x   x 3  3x 2  4 . b) g  x   x 3  3x  2

Ans. a) 2, 2 and – 1 . Hint. 2    3,  2   4

b) 1, 1, and – 2 . Hint. 2    0,  2  2  3

Exercise28 :
Find all the roots of the polynomial equation x 4  2 x3  4 x 2  6 x  21  0 if two of the roots are equal

and opposite in sign .

Ans.  3,1  i 6

Exercise29 :

Find the condition so that in the equation x3  ax 2  bx  c  0 two of the roots are equal

and opposite in sign . Ans. ab  c

Exercis30 :

Find all the roots of the polynomial f  x   x  7 x  36 if difference between two of its rootsis 3.
3 2

Ans. – 1, 2, and – 3/2 . Hint. Take the roots  ,   3, 

Exercise31:

Find all the roots of the polynomial f  x   x  7 x  36 if one root is double the other.
3 2

Ans. 3, 6 and – 2 . Hint. Take the roots  , 2 ,  and use relations between roots and

coefficients.

Exercise32 :
Find all the roots of the polynomials if the roots are in A.P. in each case.

a) f  x   x  9 x  23x  15 g  x   x3  6x2  3x  10
3 2
b)

c) f  x   32x3  48x2  22x  3 d) f  x   x4  2x3  21x2  22x  40

1 1 3
Ans. a) 1, 3, 5. b) – 1 , 2, 5. c) , , d)-4, -1, 2, 5.
4 2 4

Hint. Take the roots  ,    ,   2 and use relations between roots and coefficients.

Exercise33:

Find the condition so that the roots of f  x   x  ax  bx  cx  d are in AP.


4 3 2

Ans. a 3  4ab  8c  0

Exercise34:

Find all the roots of the polynomials if the roots are in G.P. in

a) f  x   3x  26x  52x  24
3 2

Ans. 2/3 , 2, 6. Hint. Take the roots  ,  , and  and use relation between roots

and coefficients.
8 2 1
b) f  x   54x  39x  26x  16
3 2
ans. , ,
9 3 2

f  x   27 x4 195x3  494x2  520x  192 ans.


8 4
c) , , 2,3
9 3

1  i 15
d) f  x   x  x  2x  8 2,
3 2
ans.
2

Exercise35 :

On what condition the roots of the equation f  x   x3  ax2  bx  c would be in geometrical

progression ? ans. a 3c  b 3

Exercise36 :

If the roots of f  x   x  ax  bx  cx  d are in geometric progression, show that a 2 d  c 2


4 3 2

Exercise37 :

3
Solve 2 x3  x 2  22 x  24  0 , given that roots are in the ratio Ans. 3:4.  , 2, 4
2

Exercise38:

Find all the roots of the polynomial if sum of two of its root is 0.
a)   x   4x  16x  9x  36 Ans.
3 3
,  , 4
3 2

2 2

16
Hint. Take the roots as  ,   and  ,          , use other relations between
4

roots and coefficients to find the roots.

b) f  x   x  2x  4x  5x  21 1  i 6,  3 hint.
4 3 2
Ans. Factorise the cubic equation

obtained from relations between roots and coefficients by assuming two roots equal.

c) g  x   8x  2x  27 x  6x  9
1 3
 3,  , Hint. Test for rational roots.
4 3 2
Ans.
2 4

Exercise39 :

Solve for x, y and z from the simultaneous equations

x  ay  a 2 z  a3 , x  by  b 2 z  b3 , x  cy  c 2 z  c3

Hint. Note that a, b and c are the roots of x  ty  t 2 z  t 3 i.e. t 3  t 2 z  ty  x  0 ., so that a  b  c  z ,

ab  bc  ca   y , abc  x

Exercise40 :

a2 b2 h2
Examine the equation   .........   k for imaginary roots.
x A xB xH
Hint. Let p  iq is an imaginary root so that p  iq is also a root. Putting these values in the equation

in turn and subtracting one from the other we have

 a2 b2 h2 
q   .........    0 which is not possible as the term in the bracket is
  p  A  q  p  B   q  p  H   q 2 
2 2 2 2 2

positive unless q  0 , a contradiction . Thus there are no imaginary roots.

Example 12:

If f  x   x 5  ax 4  bx 3  cx 2  dx  e has all real coefficients, and if all the roots are real numbers, then

show that 2a 2  5b . In other words, if 2a 2  5b , all the roots cannot be real numbers.

Hint : if  ,  , , ,  be the five roots of the equation f  x   0 ,

we have i)             a , ii)   b ,

    a 2   2  2   2  a 2  2b .
2
So that

   
2 2

    5  a  2b   a  2a  5b .
2 2 2
But
5  5 

§4.25:Upper bound and lower bound theorem remember


While testing a number for a root, it is convenient to narrow down the range in which the roots lie; i.e.,

to find an upper bound and lower bound of roots, which is always there since the number of roots is

finite. Though there are special methods for finding the bounds, such as a theorem given below,

employing mere common sense we can often know the bounds by regrouping the terms . Follow the

example.

Example13:

Find the bounds of positive and negative roots of f  x   x  2x 13x  38x  24  0


4 3 2

Regroup the terms as x  x  5  x  3x 13   38x  24  0 . The expression is positive for
3 2
x = 5, so

with a little common sense it can be understood that there cannot be a positive root greater than 5,

since the function is monotonically increasing. One could have regrouped as

x 2  x 2  3  x 2  x  13   38 x  24   0 which would have given us 13 as an upper bound of positive

roots; but 5 as upper bound is a better choice.

Now, the negative roots of f  x  are positive roots of f   x  remember

So we could carry out the same analysis with f   x  for lower bound of negative roots of f  x  .
We have f   x   x  2x 13x  38x  24  0 . Regrouping the terms, we get,
4 3 2

x 2  x 2  13  2 x  x 2  22   6  x  4   0 . Note that the expression is positive for x  5 , so 5 is an upper

bound of positive roots of f   x  , i.e., – 5 is a lower bound of negative roots of f  x  .

See the example below to find out how knowing upper and lower bounds help in finding integer roots.

Example14:

Investigate the function f  x   x  2x 13x  38x  24 for integer roots.


4 3 2

Since product of the roots is – 24 , and the upper bound and lower bounds are 5 and – 5 respectively,

the possible values – 4, – 3, – 2, – 1,1, 2, 3 and 4 may be tried for being roots; as 0 is evidently not a

root as the expression is not 0 by substituting 0 in the function.

The expression may be written as 24  38 x  13x 2  2 x3  x 4 and may be divided by 4 – x as follows.

4  24 38  13 2 1
6 8
32 5
In the third line of quotient and remainder, as soon as – 5 is got, which is not divisible by 4, the

process of division is stopped and – 4 is thus known not to be a root.

Then dividing by 3 – x , we get,

4  24 38  13 2 1
8 10 1 1 ,
30 3 3 0

the division is complete so – 3 is a root.

We could further shorten the process by dividing the function by 3 – x and examining the quotient

8  10x  x 2  x3 for integer roots along the same lines as given below.

2 8  10 1 1
4 3 1 ,
6 2 0

then 2 is also a root.

Dividing the quotient by x – 2, we get, x 2  3x  4  0 , which is a quadratic equation having roots, 1

and 4. Thus all integer roots are found out.


Exercise41 :

Given that sum of two roots of x 4  px3  qx 2  rx  s  0 is equal to the sum of the other two.

Prove that p3  8r  4 pq .

Exercise42 :

If the roots of px3  3qx 2  3rx  s  0 are in AP, prove that p 2 s  2q 2  3 pqr .

Exercise43 :

Find the upper bounds and lower bounds of roots of the following Hint. (regroup suitably)

a) x5  3x 4  x3  8 x 2  51x  18  0 ans. 3, – 5 .

b) x 4  2 x3  3x 2  5 x  1  0 ans. 2, 0

c) x 4  x3  2 x 2  4 x  24  0 ans. 4, – 2 .

d) x 4  2 x3  3x 2  2 x  3  0 ans. 3, – 2 .

Exercise44 :
Find the rational / integer roots of

a) x 4  2 x3  7 x 2  8 x  12  0 ans. 1, 2, – 2 , – 3 .

b) x3  9 x 2  22 x  24  0 ans. 6.

c) 3x3  2 x 2  6 x  4  0 ans. 2/3 .

d) 25 x 4  70 x3  126 x 2  414 x  243  0 ans. 1, 3.

e) x 4  9 x3  12 x 2  80 x  192  0 ans. – 4, – 4, – 4, 3.

f) 6 x 4  7 x3  8 x 2  7 x  2  0 ans. ½, 2/3 .

§4.26:Transformations of equations

A polynomial equation may be transformed to another whose roots are related to its roots in a

particular fashion, e.g. differ from them by a constant, negative of them etc. Follow the examples and

remember the results.

a) Form a polynomial whose roots are negative of roots of f  x  .


Evidently f   x  is a polynomial whose roots are negative of f  x  . Verify. Also try to prove with the

help of remainder theorem as in the following exercise.

b) This is example of reflection of a graph y  f  x  about the y – axis. What about reflection of a

graph about x – axis ? Is it y   f  x  ? Verify.

c) If f  x  not a polynomial, still then f   x  is a function ( evidently transcendental if not

polynomial) whose roots are negative of that of f  x  . Verify.

d) If f   x   f  x  the function is symmetric about the y – axis , also called an even function.

For example, polynomials , all of whose terms are of even degree. Another example is

y  cos x  cos   x  .

e) If f   x    f  x  the function is anti-symmetric about the y – axis , also called an odd

function. For example, polynomials , all of whose terms are of odd degree. Another example is

y  sin x   sin   x  .

f) Give an example of a function which is neither odd nor even.

Try a polynomial of even or odd degree of mixture of odd and even powers.
g) Show that any function f  x  can be expressed as a sum of even and odd functions.

f  x  f x f  x  f x
Verify that f  x    comprises of two terms , the 1st one is even and
2 2

the other is odd.

h) Forming a polynomial whose roots differ from the roots of a given polynomial by a

constant.

Suppose  is a root of f  x   ax  bx  cx  d . Show that   h is a root of f  x  h  .


3 2

Plainly speaking, if the y – axis is shifted to the right by h units, we must substitute every x by

x + h so that the relation between x and y is preserved. This settles the matter.

From another angle, if f  x    x    x    .............n terms , then

f  x  h    x    h    x     h   .............n terms , i.e., the roots of f  x  h  are   h,   h etc.

For numerical computation if f  x  h   a0 x  a1 x  ............an  0 ....................(1)


n n1

the original equation must be f  x   a0  x  h   a1  x  h 


n 1
 ............an  0 .......(2)
n
If we divide f  x  by x  h , the remainder an can be determined. Then take the polynomial

g  x   a0  x  h   a1  x  h   ............an 1  0 . Dividing g  x  by x  h , the remainder an 1 can


n 1 n2

be determined; and all the coefficients of (1) may be got in this way.

Example15 :

Find the equation whose roots are diminished by 3 each from the roots of the equation

f  x   x 4  x3  2 x 2  x  1  0

Dividing repeatedly the equation by x  3 we get,

1 1 2 1 1
3 12 42 129
4 14 43 130  an
3 21 105
7 35 148  an 1
3 30
10 65  an 3
3
13  an  4
 f  x  3  13x3  65x2  148x  130  0
Employing synthetic division at each step, the coefficients of f  x  3  0 are easily got.

Exercise45 :

a) If f  x   4x  32x  83x  76x  21 , find the equation whose roots each exceed by 2 of the
4 3 2

roots of this polynomial. Ans. g  x   4x4 13x2  9 .

b) Verify that the roots of f  x  are 1,  , 3 ,  , those of g  x  are 1, 1, ,  .


1 7 3 3
2 2 2 2

Exercise46 :

If f  x   x  4 x  3x  4 x  6 , find the equation whose roots each are diminished by 3 of


5 4 2

the roots of this polynomial. Ans. g  x   x5  11x4  42x3  57 x2 13x  60 .


x 3 3
Exercise47: If f  x   x3    0 , find the equation whose roots each are exceed by of
4 4 2

9 2 13 15
the roots of this polynomial. Ans. g  x   x3  x  x .
2 2 4

Exercise48 :

If f  x   x  5x  6x  3 , find the equation whose roots each are exceed by 1 of the roots
3 2

of this polynomial. Ans. g  x   x3  8x2  19 x 15 .

Exercise49 :

Find the equation whose roots are increased by 3 each from the roots of the equation

f  x   x 4  x3  2 x 2  x  1  0 .

Hint. (put –3 in place of 3 in the above example).

[Though it is as simple as that, the reasoning may be traced to remainder theorem.

By remainder theorem, we have f  x    x    Q  x   R  x  . If  is a root of f  x  ,

f    0  R   . ..........................(1)
 
Replacing x throughout by x  h , f  x  h   x    h  Q  x  h   R  x  h  . ( this does not

make any difference as x is an arbitrary real number)

 
Putting   h in place of x, we have, f   h  h     h    h  Q   h  h   R   h  h 

...............................(2)

Or, f    0.Q    R    R   . Since this is automatically true by virtue of (1),, then (2) is

true ; which implies that   h is a root of f  x  h  . The proof is quite general , for all

polynomials, so true for cubic equation too. ]

Exercise50:

Suppose  is a root of the polynomial f  x  of degree n, show that k is a root of the

x x
polynomial f   , i.e., of the equation f    0 .
k k

If f  x    x    Q  x   R  x  is the given polynomial, ...................................(1)

If  is a root, then f    0  R   ...........................(2)

x x  x   x  x
Replacing x by throughout, f        Q    R   ........................(3)
k k k  k k
Now if we choose x  k , then (3) also yields f    0  R   , which is of course

automatically satisfied in view of (2) as  is a root of the polynomial f  x  . Then (3) means

x
that k is a root of the polynomial f   .
k

x
Note that if k  1 , y  f   is nothing but stretching or magnifying the x – coordinates by
k

a factor k for same value of y and vice versa.

As an alternative simpler to understand, if f  x    x    x    .............n terms , then

n n 1
x x  x  x  x
, f            ....n terms     p1    .....  pn  0 ,i.e., k , k  etc are the roots
k k  k  k k

x
of f    0
k

n n 1
 x  x
The required equation may be obtained by    p1    ...............  pn  0 , or
k k

x n  p1kx n1  p2 k 2 x n2  ..................... pn k n  0 .

Exercise51:

Find the equation whose roots are reciprocals of roots of f  x   ax  bx  cx  d  0


3 2
Let  ,  and  be the roots of f  x  , then f  x   ax  bx  cx  d  0  a  x    x    x   
3 2

.................................(1)

b
Expanding and equating the coefficients of corresponding terms in both sides        ,
a

c d
      and    . .......................................(2)
a a

If g  x  is the equation whose roots are


1 1 1
, and respectively,
  

 1  1  1 3  1 1 1 2  1 1 1  1
 x   x   x    x      x      x  0
                
     2      1
 x3  x  x  0   x3        x2        x 1  0
  

d 3 c 2 b  g  x   dx3  cx2  bx  a  0.................................(3)


  x  x  x 1  0
a a a

Just the same equation with coefficients written in reverse order.

Prove the statement in general terms in light of the above examples.


§4.27:Reciprocal equation

An equation with reciprocal roots is different from reciprocal equation.

1
If f  x   x  px  qx  r  0 be a given equation, and if f  x   f   , i.e. if the equation does
3 2

 x

1
not change by changing x to
x

The criteria suggests that there exists a certain relationship among the coefficients of such an

equation.

1
I. Show that if  is one root of a reciprocal equation, then is also a root.

1
So the roots of a reciprocal equation occur in pairs  , etc. if the degree of a

reciprocal equation is odd, it has odd number of roots so that one of them must

be own inverse, i.e. it should be 1 or – 1 .

II. Show that coefficients of such an equation equidistant from the middle term must

be

1. Equal to each other , or,


2. Negative of each other.

Let x n  p1 x n 1  .....................  pn 1 x  pn  0 ......................................(1)

1
Be a reciprocal equation. Changing x to we get,
x

pn 1 n 1 p 1
xn  x  .....................  1 x   0 ...........................................(2)
pn pn pn

1
Since f  x   f   , comparing the coefficients, we get,
 x

pn 1 p 1
 p1 , n  2  p2 ,......  pn 2  1  pn  1 .......(3)
pn pn pn

If pn  1 , we get pn1   p1 , pn2   p2 ,.............. equidistant terms from the middle are

equal to each other.

If pn  1 , we get pn1  p1 , pn2  p2 ,.............. equidistant terms from the middle are equal

to each other.

Exercise52 remember

III. Take a reciprocal equation of odd degree where coefficients of terms equidistant from

the middle term are equal to each other. Show that the root which is its own inverse is –
1 . So division of this equation by x + 1 reduces it to one of even degree, of degree one

less than the original equation.

Exercise53 remember

IV. Take a reciprocal equation of odd degree where coefficients of terms equidistant from

the middle term are negative each other. Show that the root which is its own inverse is

–1 . So division of this equation by x – 1 reduces it to one of even degree, of degree

one less than the original equation.

Exercise54 remember

V. A reciprocal equation of even degree whose coefficients of terms equidistant from the

middle term are negative of each other might be like

x 2 m 1  p1  x 2 m  2  1  .............  0 can be divided by x 2  1 and reduced to a reciprocal

equation of degree 2m – 2 , whose coefficients of terms equidistant from the middle

term are equal to each other.

Exercise55 remember
VI. As such all reciprocal equations can be reduced to one of even degree whose

coefficients of terms equidistant from the middle term are equal to each other by

dividing it by x  1, x  1 or x 2  1 as necessary, and the degree of the resulting

1
equation may further be reduced to half of it, by the transformation y  x 
x

Hint. If ax 2 m  bx 2 m1  cx 2 m 2  .............  k  0 .

1  1  1  1 
Note that x n 1  n 1
  x n  n  x     x n 1  n 1  .
x  x  x  x 

Writing z  x  , calculate x 2  2  z 2  2 , x3  3  z  z 2  2   z  z 3  3z
1 1 1
x x x

 z  z 3  3z    z 2  2   z 4  4 z 2  2 and so on.
1
x4  4
x

Example16:

Find all the roots of x5  5 x 4  9 x3  9 x 2  5 x  1  0 .

This is a reciprocal equation whose coefficients of terms equidistant from the middle term are

negative of each other and 1 is a root which is easily verified by summing up all odd

coefficients which is equal to sum of all even coefficients. Dividing it by x – 1 , we get


x 4  4 x3  5 x 2  4 x  1  0 , which is again a reciprocal equation whose coefficients of terms

equidistant from the middle term are equal to each other. Dividing this by x2 throughout, we get

1 1
x2  4 x  5  4  2  0 .
x x

1
Transforming with y  x  , the equation becomes
x

y 2  4 y  3  0  y  1 or y3  x
1
2
  1

1  3 , or , x  3  5
2

Exercise56:

Show that the equations are reciprocal equations and solve them.

1
a) 2 x5  15 x 4  37 x3  37 x 2  15 x  2  0 ans. 1, 2, , 2  3
2

b) x 4  10 x3  26 x 2  10 x  1  0 ans. 3  2 2, 2  3

1
c) 2 x 4  5 x3  4 x 2  5 x  2  0 ans. 2, ,i, – i
2

Exercise57 : Show that x 4  3x3  4 x 2  2 x  1  0 can be converted into a reciprocal equation by

diminishing the roots by 1 each.


§4.28:Project; Solving Cubic equation by removing second term

Transform the equation ax3  3bx 2  3cx  d  0 with the transformation y  x  h , where h is a constant

to be determined so as to make coefficient of y 2 equal to 0 in the transformed equation without y 2

 b2   3bc b3 
term i.e. ay 3  3  c   y   d  2 2   0.
 a  a a 

b
(This h  ). Then multiply the roots of this equation by ‘a’ to get rid of fractions put z for ay ; the
a

equation is transformed into z 3  3Hz  G  0 . Then put y  3 p  3 q and solve the cubic equation in y,

i.e., quadratic equations in 3 p and 3 q . Derive the formula

1 1
p   G  G 2  4 H 3  , q   G  G 2  4 H 3 
2  2 

Finally from y  x  h find x.

First solve a concrete example x3  6 x 2  3x  10  0 in the procedure simplifying at each stage so that

big expressions are avoided. Find out that there are not nine solutions as it appears, but only three.

Then try deriving the formula as above.


Exercise58 :

Show that the roots of ax3  3bx 2  3cx  d  0 are in the form m, m , m 2 , where 1, ,  2 are cube roots

of unity.

§4.29:Symmetric functions of roots

Algebraic combinations of roots of a polynomial equation such as  2   2   2 ,where the

expression remains the same if we interchange say  and  .Note that  2    2   2 is a cyclic

expression but the expression changes if two of the roots are interchanged; we exclude such cases

from the present consideration and concentrate only on symmetric functions of roots. Such symmetric

functions of roots could be found out without actually knowing the roots. The example given below

shall clarify.

Example17

Find  2   2   2 when  ,  and  are the roots of x3  px 2  qx  r  0 .


We have         p , then       2
 p 2 , also       2q , then

p2   2   2   2  2         2   2   2  2q   2   2   2  p 2  2q

Misc. Exercises

In the polynomial equation , x3  px 2  qx  r  0 , if  ,  and  are the roots ,prove that

1 1 1 q
59)   
   r

60)  3
  p3  3 pq  3r

 2   2 pq
61)   3
 r

 2   2  2   2  2   2 2 p2 q  4 pr  2q2
62)   
      r  pq

63)   2 2
 q2  2 pr

64)             r  pq

 b c  pq
65)      3
c b r
Exercises66:

In the polynomial equation , x 4  px3  qx 2  rx  s  0 , if  ,  ,  and  are the roots ,prove that

a)    pr  4s
2

b)  4
 p4  4 p2q  2q2  4 pr  4s

Exercise67:

If  ,  and  are the roots of the polynomial equation x3  3x 2  2  0 , show that the roots of

x3  33x 2  12 x  8  0 are  ,  and  3 .


3
3

Exercise68:

If  ,  and  are the roots of the polynomial equation x3  qx  r  0 , show that the roots

a) of x3  qx  r  0 are      ,     and     .

  
b) of rx3  q 2 x 2  2qrx  r 2  0 are , ,
  

c) of x3  q 2 x2  2qr 2 x  r 4  0 are  2 2 ,  2 2 ,  2  2

Exercise68:
If p, q, and r are roots of x3  ax 2  bx  c  0 , find the equation whose roots are p 2 , q 2 and r 2 . Ans.

x 6   2b  a 2  x 4   b 2  2ac  x 2  b 2  0

Hint. Find the values of p 2  q 2  r 2 , p 2 q 2  q 2 r 2  r 2 p 2 and p 2 q 2 r 2 . Also note that

p 2 q 2  q 2 r 2  r 2 p 2   pq  qr  rp   2 pqr  p  q  r  .
2

Exercise69 :

If p, q, and r are roots of x 4  x3  2 x 2  x  1  0 , find the equation whose roots are p 2 , q 2 and r 2 .

Ans. x 4  3x3  4 x 2  3 y  1  0

Exercise70: If p, q, and r are roots of x3  3x 2  2  0 , find the equation whose roots are

p3 , q3 and r 3 .

Ans. x3  33x 2  12 y  8  0

Exercise71 :

The roots of the equation x3  ax 2  bx  c  0 are p, q and r . Find

1 1 1 a 2  2b 1 1 1 b 2  2ac
a)  2 2 2 2 ans. b)  2  2 ans. .
2 2
pq qr r p c2 p 2
q r c2
Exercise72

If x3  qx  r  0 , find

a)  4
ans. 2q 2

  
2
b) ans. 6q

1 q
c)    ans.
r

exercise73

If x3  2 x 2  x  1  0 , find  4
ans. 10.

Exercise74: If x 4  3x3  5 x 2  12 x  4  0 , find  5


ans. 123.

Exercise75 :

Find the polynomial whose roots are k times the inverse of the roots of x3  px  q  0

Ans. qx3  kpx 2  k 3

Exercise76 :

If  ,  and  are the roots of x3  px  q  0 find the equation whose roots are  3 ,  3 ,  3 .
Ans. x3  3qy 2   p 3  3q 2  x  q 3  0

Exercise77:

If  ,  and  are the roots of x3  px  q  0 find the equation whose roots are

      ,      ,      .

Ans. x3  2 py 2  p 2 x  q3  0 .

Exercise78 :

Show that the roots of the equation x3  p3 x 2  q3 x  p3q3  0 are cubes of the roots of the equation

x3  px 2  qx  pq  0 .

Exercise79 :

Find all the roots of x5  5 x 4  5 x3  25 x 2  4 x  20  0 if sum of two pairs of roots is 0 each.

Ans. 5, 1, 2
CHAPTER 5: MORE ABOUT THEORY OF EQUATIONS,

ANDROOTS BY NUMERICAL METHODS

Since some of the derivations about theory of equations required the concept of differentiation, this part of the

theory of equations was deferred until differentiation chapter was finished.


§5.1: Extended synthetic division:

Priorly the process of synthetic division was considered for cases where divisor was a linear expression in x

i.e. of the type a – x etc. If the divisor is a quadratic or expression of higher order, the algorithm shall be clear

from the following example.

Divide x5  5 x 4  9 x3  6 x 2  16 x  13 by x 2  3 x  2 , find the quotient and the remainder.

The actual division is as given below

x 2  3 x  2  x5  5 x 4  9 x3  6 x 2  16 x  13  x 3  2 x 2  x  1

 x5   3x 4   2 x3
 2 x 4  7 x 3  6 x 2  16 x  13
  2 x 4   6 x3   4 x 2
x3  2 x 2  16 x  13
 x3   3x 2   2 x
x 2  18 x  13
 x 2   3x   2
 15 x  11
This example shall tell you how to abridge the process of division only if the coefficient of the leading term is

1.

The quotient is found to be x3  2 x 2  x  1 and the remainder is  15 x  11 .We could have saved labor on

account of :

1) not writing the first term in each step of division since it is meant to be cancelled

3 2
2) writing only coefficients and dispensing with x , x , x etc. the degree of x in each place may be

understood by the relative place of the coefficient.

3) Changing sign of terms of the divisor x 2  3 x  2 except the first term ( which is not considered for

multiplication with quotient at each step) so that we need not change sign and add in each step , we

can simply add.

4) Instead of underlining at each step, we could consider a whole column of a particular power of x ,

say x3 and add up all the members of that column.

5) Not multiplying the first term of the divisor by successive terms of quotient since it is meant to

cancel the first term of divisor at each step .

6) The process is stopped as soon as we reach the last term of the dividend. After the process is

stopped, since the degree of the remainder should be one degree less than that of the divisor, in
the last line, the number of places reserved for the remainder should be from the right, one less

than the number of terms in the divisor, with powers of variables in ascending order from the right.

The process is illustrated as under:

1 5 9 6 16 13
1 3 2
3 6 4
quotient x3  2 x 2  x  1 and the remainder  15 x  11 When the coefficient of
2 3 2
3 2
1 2 1 1 15 11

the leading term of the divisor is not 1, say 3, the partial quotients should not be multiplied

with the divisor at each step as they are, as it would not cancel the first term at each step ( that is what our aim

is, to cancel out the first term at each step), so the product at each step should be divided by the leading

coefficient .See the following example. Remember to represent missing coefficients by 0.

To divide 6 x  5x y  8x y  6 x y  6 xy by 2 x  3x y  y . See below.


5 4 3 2 2 3 4 3 2 2
3 2
2
6 5 8 6 6
3
9 0 3
0
6 0 2
1
3 2 2 3 8

Note that when the terms of the dividend have exhausted, the process is stopped. The remainder is

2 x3 y 2  3x 2 y 3  8xy 4 , three terms from the last, one term less than those in the divisor, and the quotient is

3x 2  2 xy .

Note that in the last line, the quotient at each step is divided by 2, the leading coefficient in the divisor, but as

soon as the first term of remainder is reached. Actual division may be carried out on pen and paper and the

algorithm verified.

The process may be continued to any desired number of steps also just like division in Arithmetic after the

decimal place. The rules to determine the quotient and the remainder remain the same. See the illustration

below.
3 2 1 0 4
6 5 8 6 6
2
9 0 3
3
6 0 2
0
3 0 1
1
12 0 4

3 2 1 0 4 11 0 4

Note that division by 2 is carried out as long as we want to extend the terms of the quotient and stopped as

soon as the remainder starts.

Exercise1

Divide x  2 x  4 x  19 x  31x  p by x 3  7 x  5 and find p if the latter expression divides the former.
5 4 3 2

ans. a =15

§5.2: Sums of powers of roots and other symmetric functions of roots.

Let a polynomial of degree n is factorized and written as

f(x) = (x – a1) (x – a2) (x – a3)………. (x – an)………….(1)

Taking natural logarithms of both sides,


lnf(x) = ln(x – a1) + ln(x – a2) +ln(x – a3)………. ln(x – an)….(2)

Differentiating with respect to x,

f ' (x) 1 1 1 1
    ............... …………………(3)
f (x) x  a1 x  a 2 x  a 3 x  an

Expanding (x – a1)-1 = x -1(1 – a1x-1) = x -1 + a1x-2 + a12x -3 + a13x -4 +………….. ,

(x – a2)-1 = x -1(1 – a2x-1) = x -1 + a2x -2 + a22x -3 + a23x -4 +………….. ,

(x – a3)-1 = x -1(1 – a3x-1) = x -1 + a3x -2 + a32x -3 + a33x -4 +………….. ,

………………………………………………………………………..,

……………………………………………………………………….., etc.

with this substitution in (3) and collecting the coefficients of x –n-1 we observe that

the sum of the n-th powers of roots a n


r is equal to the coefficient of x -n-1
in the expansion of

f’(x)/f(x).

We denote the sum of i-th power of roots of f  x  by Sk.

1 1
e.g. S1    , S 3    3 , S1   , S4   etc.
 4

example1
If f  x   x  2 x  x  1 , find S4
3 2

f ' x 3x 2  4 x  1
We have f '  x   3x  4 x  1 so that
1 1 1
 3   
2
,
f  x  x  2x  x 1 x   x   x  
2

where  ,  and  are the root of f  x   0 .

f ' x
S 4   4  coefficient of x 5 in the expansion of .
f  x

In other words,

f ' x 1 1 1  1  2  3 S S S S
  
f  x x  x   x  
   x  x2  x3  .......  x  x12  x32  x 43  x54  .......
 

The coefficients can be easily obtained from synthetic division of f '  x   3x  4 x  1


2

by f  x   x  2 x  x  1 as shown below.
3 2

3 4 1
1
6 3 3
2
4 2 2
1
4 2 2
1
10 5 5
3 2 2 5 10 3 5...................
f '  x  3 S1 S 2 S3 S 4

3 2 2 5 10
  2  3  4  5  ....... ;   2  3  4  5  .......... then S   10
4 4
Thus
f  x x x x x x x x x x x

Exercise2

If f  x   x  x  1 , find S6
3
ans. 5.

Exercise3

If f  x   x  x  7 x  x  6 , find S6 and S6
4 3 2
ans. .99, 795.

§5.3: Multiple roots :

If a polynomial or any continuous derivable function f(x) (derivable required number of times) has n factors

each equal to x – a , n times , its derivative f’(x) contains n – 1 of the same factor.

Let f(x) = (x – a)n(x) . Differentiating both sides, f’(x) = n(x – a)n – 1 (x) + (x – a)n’(x) = (x – a)n – 1 (x)[ n(x)

+ (x – a)’(x)] which has a factor (x – a)n – 1 .This proves the proposition.

Exercise4

Solve the following if the equations have two or more equal roots.
Hint. Find a common factor between f  x  and f '  x 

a) x5  13x 4  67 x3  171x 2  216 x  108  0 ans. 3, 3, 3, 2, 2.

b) x 4  6 x3  12 x 2  10 x  3  0 ans. 1, 1, 1, 3.

c) x 4  9 x 2  4 x  12  0 ans. 2, 2, – 1, – 3 .

d) x 6  2 x5  4 x 4  12 x3  3x 2  18 x  18  0 ans.  3,  3,1  i.

e) x   p  q  x  p  p  q  x  p  p  q  x  p 3q  0 p, p,  p, q
4 3 2 2
ans.

Exercise5

Given that the equations below have multiple roots, solve them.

Hint. Differentiate the given function f  x  . Find out GCD of f  x  and f '  x  . Their GCD is the product of

multiple roots. Then divide f  x  by the GCD to get the reduced equation , solve it for the rest of the roots.

1 1 1
a) 8 x 4  4 x3  18 x 2  11x  2  0 ans. , , and  2
2 2 2

1 i 3 1 i 3
b) x5  x3  4 x 2  3x  2  0 ans. 2, ,
2 2

c) x4  6 x2  8x  3  0 ans. 1,1,1 and  3

1 1 3
d) 8 x3  20 x 2  14 x  3  0 ans.  , ,
2 2 2
e) x 4  9 x 2  4 x  12  0 ans. 2, 2, 1, 3

Exercise6

The function f  x   px  3qx  3rx  s has two equal roots . Find the relation among the coefficients.
3 2

 qr  ps   4  pr  q 2  qs  r 2 
2
Ans.

Hint eliminate x between f  x  and f ' x

pq  6r
Exercise7: If three roots of x  px  qx  rx  s  0 has three equal roots, show that it is
4 3 2
.
3 p 2  8q

Exercise8: The equation x  10 p x  q x  r  0 has three equal roots. Prove that pq  9a  c  0


5 3 2 4 5 4 5 5

Exercise9: Show that the equations x  px  qx  s  0 and 15qx  6 p x  25s  4 pr  0 have a common
5 3 2 2 2

root equal to two equal roots of the former.

Exercise10: Find the relation between p and q if px  qx  p  0 and x3  2 x 2  2 x  1  0 have a common root.
2

What is the relationship if they have two common roots ?

Ans. 2p  q  0, p  q  0.

Exercise11: If x n  ax 2  b  0 has two or more roots equal, the show that n nb n  2  4a n  n  2 


n2
.
Exercise12: Prove that x  qx  r  0 cannot have three equal roots.
4 2

Exercise13: Prove that x  nx


n n 1
 n  n 1 xn2  .............  n!  0 cannot have equal roots.

§5.4: Roots of transcendental functions

To prove: ei = cos  + i sin :

We know ei would be a complex number and let it be a + ib or r(cos  + i sin ) in terms of

modulus and argument. Writing this concisely

ei = r(cos  + i sin )……………….(1)

The complex conjugates of equal numbers should be equal. Thus, taking – i in place of i throughout, for

taking conjugates, we have,

e -i = r(cos  - i sin )…………………(2)

Multiplying (1) and (2), 1 = r2 (cos2  + sin2 ) = r2

So (1) can be written as ei = cos  + i sin …………….………..(3)


[ We could reach at this point without the given argument for conjugates, i.e., if two complex numbers are

equal, their conjugates shall be equal too. Take -  in place of  so that

r[cos (-)+ i sin (-)] = r[cos  - i sin ] = r(cos  + i sin )-1 = r.r/ ei (from (1))= r2 e-i

or, r e-i = (cos  - i sin ) which is precisely (2), without taking the argument for conjugates]

dφ dφ
Differentiating both sides, iei = (- sin  + icos ) =i(cos  + i sin )
dθ dθ


Or, ei = (cos  + i sin ) ………………………..(4)

dφ dθ
Comparing (3) and (4),  1 , but  1 so  and  differ by an arbitrary constant. But  should be 0
dθ dθ

when  is 0. Putting  = 0 in (1), cos  = 1 or  = 2m +, where m is any integer. Thus the arbitrary constant

can be taken as 2m, and (1) can be written as

ei = cos  + i sin  or ei(+2m) = cos  + i sin 

or cos (2m +  )+ i sin (2m +  ) …………………………..(5)

 ix eix  e ix eix  eix


We now have e  cos x  i sin x and e
ix
 cos x  i sin x so that we have. cos x  and sin x 
2 2i

……………………..(6)
Interestingly i =  1 does not affect the results of Calculus as long as we insert it as a constant and

eix  e ix eix  e ix


that it is. sin x, cos x, tan x etc. can be treated as sin x  , cos x  and these conversions
2i 2

deix
could be used in problems.(We know  ie ix )
dx

d
Now (cos x  i sin x )   sin x  i cos x  i(cos x  i sin x ) so cos x + i sin x and eix are similar. It will not be easy
dx

to understand cos x + i sin x = eix and similarly cos x - i sin x = e-ix until we finish the relative chapter of

trigonometry (see next article for a brief treatment).

§5.5: de’ Moivre’s Theorem : (cos  + i sin )n = cos n + i sin n)

Taking ei = cos  + i sin , we see that ein   cos   i sin   , but ein  cos n  i sin n
n

for integral values of n.

So (cos   i sin  )  cos n  i sin n .This is called de’ Moivre’s Theorem, which is also true for
n

rational values of n. For, if n = p/q, where p, q, are integers, q > 0, ein = ei(p/q) = (ei/q)p = (cos /q + i sin

/q)p = cos p/q + i sin p/q (as the p is an integer) = cos n + i sin n. For irrational values, the proof is

beyond the scope of elementary level but the result is permitted for use. It may be seen that the theorem
holds for  = 0 also. (To understand that it is true for irrational values , we may remember that an irrational

number is just a converging infinite series of rational numbers)

Another proof : This is by applying the method of mathematical induction

Step I : To show that the theorem is true for positive integral values of n. The theorem is also proved straight

by induction. We observe that

(cos  + i sin )0 = 1 = cos 0. + i sin 0., the theorem being true for n = 0.

Then (cos  + i sin )1 = cos  + i sin  = cos 1. + i sin 1.; the theorem being true for n = 0.

Also (cos  + i sin )2 = cos2 - sin2 +2i cos  sin  = cos 2 + i sin 2 ;

Let us assume the theorem is true for n =m and prove it for n = m + 1.

So we have (cos  + i sin )m = cos m + i sin m.

Multiplying both sides by cos  + i sin , we have,

(cos  + i sin )m+1 = (cos n + i sin n).(cos  + i sin )= cos n cos  - sin n sin  +i(cos n sin  + sin n cos

)= cos (n + ) + i sin (n + ) = cos (n + 1) + i sin (n + 1)

Which shows that the theorem is true for n = m + 1. Hence the theorem is true for all positive integral values of

m.
Step II : To show that the theorem is true for negative values of n

1 cos   i sin  cos   i sin 


Now (cos  isin )-1     cos   i sin 
cos   i sin  cos   i sin  cos 2   sin 2 
2 2 2

 cos( )  i sin( )  cos(1)  i sin(1)

So the theorem is true for n = - 1 also.

If n is a negative integer, say n = - k where k is a positive integer, we have,

1 1
(cos  isin )n  (cos  isin )-k  
(cos   i sin  ) k
cos k  i sin k

cos k  i sin k  cos(k )  i sin(k )  cos n  i sin n


  cos k  i sin k
cos 2 k  sin 2 k

So the theorem is true for negative values of n

Step III : To show that the theorem is true for rational values of n. If q is a non-zero positive integer, and p be

an integer, we have, [(cos  isin )1/q ]p  [(cos  isin )p ]1/q

1 1 cos k  i sin k
 (cosp  isinp )1/q     cos k  i sin k
(cos   i sin  ) cos k  i sin k cos 2 k  sin 2 k
k

 cos(k )  i sin(k )  cos n  i sin n

Prove that ii is a real number. (Hilbert’s number)


Exercise14:
i
 
e 2  cos  i sin i
i
In the equation e  cos   i sin  , take θ = Л/2, so that 2 2 raising both sides to ith power,

i
 i  i  
2

i   e 2   e 2  e 2  0.207879576... this is a transcendental number and of course, a real number.
i

 

§5.6: All the roots of polynomial xn + xn – 1 + xn – 2 +……………+1:or xm –

1=0:

the problem of inscribing a regular n-gon inside a circle.

Multiplying the expression by x – 1 we get xn+1 – 1 = 0, xn+1 = 1 = eim, where m is any integer. So that we

get, x = eim/(n+1), which gives us a root for any integral value of m. if m = 0, then x = 1 or x – 1 is a factor of

xn+1 – 1; this accounts for the extraneous factor x – 1 which we have multiplied to our polynomial. For

other integral values of m , we get the roots of our polynomial. When we go on increasing m by one at a

time, we get the different roots. But after some steps, when m = m2/n+1 differs from

m = m1/n+1 by a multiple of 2 π we get the same root again which we got earlier for m = m1/n+1.So

we can get maximum n roots this way; all distinct.


Further, as eim/(n+1) has modulus unity, or one, as |ei| = 1 for all , the n roots are placed each at unit distance

from the origin in the complex plain . For each successive root, the difference of the argument angles is

(m+1)/(n+1) – m/n+1 = /n+1.so the roots are symmetrically placed around the origin each at a

difference of equal angle from the successive one. Thus they are placed on a circle of unit radius dividing

the circle into n+1 sectors just in a manner of images formed between two mirrors inclined to each other at

an angle /n+1.

The case when n is odd, we have even number of terms in the polynomial

xn + xn – 1 + xn – 2 +……+1 as n varies from 0 to n. Grouping the terms like

(xn + 1) + (xn-1 + x) +(xn-2 + x2) +…………….(x (n+1)/2 + x (n-1)/2)

or, (xn + 1) +x(xn-2 + 1) +x2(xn-4 + 1) +…………………x(n-1)/2(x + 1)

we observe that , n, n - 2 , n – 4 .. etc. are all odd and as such, x + 1 is a common factor in each of the

parentheses, so that x +1 is a factor of the polynomial or – 1 is a root. This of course we have

earlier proved easily that a polynomial of odd degree has a real root opposite in sign to that of the absolute

term.

Now if the points on the circle of radius 1 , which represent the (n+1)-th roots of 1 may be joined

successively so that a polygon is obtained. This polygon must be regular, i.e., all its sides must be equal as

may be seen by joining each of any three consecutive points representing the roots of 1 on the circle to its
center and observing that the two triangles thus obtained are congruent. Alternatively if , ,  are the complex

numbers representing any three consecutive (n+1)-th roots of unity, it may be seen that |  - | = | - | = | - |

by expressing , ,  in ei form.

Example1 : If  is a root of the eqn. xn – s = 0, prove that its all roots are obtained by multiplying  with

n-th roots of unity.

Example2 : If  is arithmetic n-th root of the s, i.e.,   n s prove that its all roots are obtained by

multiplying  with n-th roots of unity.

Exercise15: If 1,  ,  ,  ,..... are n-th roots of unity, i.e. roots of x n  1  0 ,

show that 1   1   1    ..........  n .

Hint. Expand the expression and use relation between roots and coefficients.

§5.7: Comparison with number system:

In course of development of theory of numbers we noted that , in the set of natural numbers N,any two

numbers could be added and the result is a natural number again. But, in the inverse process, a bigger

number could not be subtracted from a smaller one in the set of natural numbers. in other words, the
set of natural numbers was not ‘closed’ in respect of subtraction. Practical necessity for subtraction of a

bigger number from a smaller one, or a number from an equal number made us looking for a bigger set

of numbers, the set of integers , I.

The theme can be worded differently in another way. If the eqn. x + 7 = 5 or x - c = 0 has no

solution in N, no matter. We have a bigger set I in which the eqn has a solution. Is the set I really bigger

than N ; when there is a one to one correspondence between them, from N onto I and from I onto N ?

That is another matter that they are equivalent sets. But we have found the solution of our eqn any way.

In other words, we found the integral value of x for every natural number value of f(x) where f(x) = x - c,

i.e., the existence of inverse function in the set of integers, if not found in the set of natural numbers.

This also dispenses with requirement of still a bigger set for looking for solution of f(x) = x - c , where

each of f(x), x, and c made up of any integers. The set of integers suffices for such a requirement.

More generally, if we take F(x) such that

F(x) = (x – a1)( x – a2)( x – a3) = x3 – (a1+ a2+ a3)x2 +(a1a2+ a1a2+ a1a2)x – a1a2a3 ,where all the symbols

are integers, we see that its solutions are in the set of integers as the function is constructed multiplying

factors like x – a1 etc. so that it is 0 only when one of the factors is 0. Instead if we take f(x) = x2 + bx +

c where the symbols are all integers, it is not always true that the inverse function x , or the zeroes of

the function in particular are all in the set of integers. We say that the inverse function does not exist as

it is not single valued; for each value of f(x) we get two values of x, may or may not be in the set of
integers. In this manner the number system was extended to the set of complex numbers C when we

agreed that the set of numbers should be closed under division (except by 0) .

In other words, looking for a solution of nx – m = 0 in the set of integers when n is non-zero and

is not a factor of m , we arrived at the set of rational numbers Q. Again the practical necessity urged us

to accept Q, the set of all fractions m/n where n is positive integer, non-zero as division by 0 is

impossible and positive for sake of brevity of expression. Maybe, this was when a tailor could make

three shirts for newborns out of a piece of cloth two meters long . The set is again seen to be equivalent

to N (countable).

Squaring, cubing or raising to powers could be construed to be a different process from multiplication

and in course of the reverse process, i.e. while taking roots we faced a problem. Again we asserted a

solution of xn – q = 0 where q is a positive and n is a natural number. Thus we arrived at the set of

irrational numbers n q or a surd. In particular if n = 2, the irrationals like 2 q or like p + 2 q were called

quadratic surds which have a ‘conjugate’ p - 2 q so that sum and products of the two numbers p + 2 q

and p - 2 q are rational numbers. Thus we could agree to solve all quadratic equations like x2 - q = 0

where q is a rational number , only on the condition that q > 0. In other words, Irrational numbers

included all such numbers which are not rationals but can be
n
expressed as a root of a rational integral algebraic function called a rational polynomial p( x )  a
n 0
nx
n
where n

is a positive integer and a1, a2,…an are all rationals. Neither its inverse function exists nor

its roots are in the set of rational numbers .But for each value of f(x) there are n values of x , maybe in the set

of rational numbers or in other bigger sets, real or complex numbers. But if they are irrational numbers

or complex numbers they must appear in conjugate pairs as the polynomial is a rational expression

formed out of product of its factors .

Apart from the surds, the irrational numbers included ‘transcendental numbers’ which are roots of


converging infinite power series like a
n 0
nx
n
= 0, which were represented by non-repeating non-

terminating decimals. Myriads of transcendental numbers

could be constructed such as 0. 12123123412345….

And the like, and they do represent definite numbers (converging series of decimals).
A good example is ‘squaring of a circle’. See the figure. With a view to convert a circle of radius r to a square of

equal area consider the smallest square circumscribing the circle of each side 2r and the greatest square

inscribed in the circle of each side r 2 .if f(r) be the area of the circle, proportional to r2 say kr2. A first

approximation of f(r) may be half way between areas of two squares , 4r2 and 2r2 . But it is more than 3r2 as the

four left out pieces cover more than area of the smallest square shown in the figure of area r2. A second

approximation would be half way between 3r2 and 4r2.say 3.5r2. But this is more than area of the circle and

the small pieces left out this time cover more area than r2/2. So a third approximation would be half way

between 3r2.and 3.5r2. continuing in the manner infinite times we arrive at the area of the circle. So k must be

represented by a limit of an infinite sequence 3, 3.5, 3.25, 3.125, 3.15, 3.14, 3.145 ….. which shall be of the

form 3.145…., a non-terminating non-recurring decimal. And this number is named  since thousand years.

Instead of successive averages of areas of squares A1 and A2 (say), we could have started with

nA 1  mA 2
choosing a big positive integer m and a small one, n. But still we cannot complete the calculation in
mn

only finite number of steps. The area of a circle is definite for a definite radius. Thus the

process reminds us of an alternative definition of ‘limit’ of a function f(x) at a point x = x0.in Calculus If

lim
xx 0
f x   L , for any sequence x1, x2, …..upto  converging on x0 , if the sequence f(x1), f(x2), f(x3), …. upto 

converges to L and vice versa. If only one such sequence could be found such that a1, a2, …..up to 

converging on x0 and f(a1), f(a2), f(a3), …. Up to  does not converge to L, lim


x x 0
f x  does not exist.
n
Ascertaining inverse functions of polynomials p( x )  a
n 0
nx
n
(where a0, a1, a2……an etc. are understood to be

real numbers unless otherwise stated), we are reminded of a property of continuous functions which states,

that : if a continuous function is differentiable and monotonic, its inverse function exists (in the set of real

numbers)

The set of irrational numbers one is uncountable and really a bigger set than N or I or Q. Still we could

not solve that equation x2 – q = 0 for negative values of q, until we defined a new number 1 or i to solve x2 +

1 = 0.

§5.8: Area of a circle :

Assuming the ratio of circumference to the diameter of any circle is  , it is a well-known technique to

find the area of a circle of radius r thus :


A circle of radius r may be divided into 2n sectors of sectorial angle  and the sectors may be fitted into each

other in a row in the manner shown. In the limiting case when n   ,   0 and the arrangement is more and

more like a rectangle with one side equal to r and the other side equal to half the perimeter of the circle i.e., r

so that the area becomes r2.

§5..9: Approximate Roots and Fixed Points of Functions : Picard’s

Theorem :

So far said about the roots of polynomials. What about functions that are not polynomials ?. and what about

the roots of algebraic equations or polynomials of degree 5 or more , whose roots cannot be found out just by

equation solving as told by Abel, although there is guarantee of a root existing as per fundamental theorem of

Algebra? For a majority class of functions including the polynomials, there is a theorem due to Picard for

numerical computation of root.

In fact all the three methods ( 4 , including the bisection method) are numerical methods. To explain what

numerical methods are, take the example of finding out the square root of a 4-5 digit number by division

method in primary school. In formula,

it may be represented by
;  a  b  c  d ...   a  a   2a  b  b   2a  2b  c  c.... each time multiplying the previous divisor by two before
2

adding another term in the root calculation; so that , if the root is not exactly a, it may be a + b, if not so, then it

may be a + b + c and so on, taking the approximation further and further.

To find solution(s) of a function f(x) = 0 , provided a real valued solution exists at all, Picard proceeds thus. Any

solution or root of f(x) = 0 is also a root of

f(x) + x = x or g(x) = x ; where g(x) = f(x) + x. Graphically, the problem of finding where the graph of y = f(x)

meet the X-axis is same as finding where the two graphs y = g(x) and y = x intersect. Another way of looking at

g(x) = x is that the function or operator g cannot change x even after operating on it. Thus its solutions which

are the same as the roots of f(x), are called fixed points of g(x). Mark that the concept of fixed point is similar

to our concepts of identity, constant, invariant , conservation etc. A particular point x = x0 chosen at random

may be verified whether it is a fixed point or not. If it is a fixed point, then O.K. we got a root. If it is not a root, it

may be close to a root, if we have a reasonable guess. Yes, a reasonable guess would be a far more helpful

rather than a random choice. And what is a reasonable guess ? The function domain should be divided into

intervals in which the function monotonically increases; or decreases for that matter. And those

subintervals should be picked up for choosing x = x0 in which the function changes value from positive

to negative or vice versa. Naturally a root is expected in that subinterval ; separating the intervals in

which the function contains a root and this is a reasonable guess.


Let g(x0) = x1 , as it is more often probable that x0 is not a fixed point and let us construct a sequence x1 =

g(x0) , x2 = g(x1) , x3 = g(x2) , x4 = g(x3) , x5 = g(x4) , etc. There is an easy way to know whether proceeding

through the sequence x1, x2, x3 ………. we are approaching the solution or going away from it. Naturally if the

successive difference between successive terms diminishes, then we are definitely nearing the fixed point. In

symbols,

if g(xn) – g(xn – 1) = xn+1 – xn < xn – xn – 1

g(x n ) - g(x n -1 )
or, if 1
x n  x n 1

then the sequence definitely converges to a fixed point.

In such a condition, graphically meaning that the curve g’(x) being a one piece smooth graph lying under the

line y = x ,for all x > x0 for some x0 which passes entirely below the dividing line y = x , must have a solution or

fixed point which can be arrived at , in this manner. Otherwise g(x) would be increasing over the baseline y = x

and shall never cross it. Even for those functions for which

g(x n ) - g(x n -1 )
 1 , we make a small manipulation. If g(x) admits of an inverse function we have g(x) = x  x
x n  x n 1

g(x n ) - g(x n -1 ) g -1 (x n ) - g -1 (x n -1 ) x n 1  x n
-1
= g (x) and if  1 , we have   1 so that g-
x n  x n 1 x n  x n 1 x n  x n 1
1
(x) has a fixed point which can be reached in the same manner. Now it would be shown that g(x) and g -1(x)

have the same fixed points . The inverse function is obtained ( if at all it exists) by putting x in place of y and y

in place of x. So the graphs of the function and its inverse function are mirror images of each other in the

opposite sides of the line y = x. Thus if g(x) intersects y = x at a point, g-1(x)

shall intersect y = x at the same point , being its mirror image so that g(x) and g-1(x ) have the same fixed

g(x n ) - g(x n -1 )
points. Now if  1 so that the graph of g(x) lies entirely above the line y = x,
x n  x n 1

g -1 (x n ) - g -1 (x n -1 )
then  1 and the graph of g-1(x) being the mirror image of graph of g(x), shall lie below the
x n  x n 1

line y = x and its fixed points would be the fixed points of g(x) and may be obtained in the

similar manner. So this powerful thinker, Charles Emile Piccard (1856 – 1941), a French mathematician

not only gave an existence theorem for solutions of most of the commonplace functions, but gave an algorithm

as to how to find it; and that too with simplest ideas. In short, this is how to write a solution of an equation

instead of finding it. Explore what made Picard think so axiomatic –

if a function has a root, it can be dug out. Computer programmes may be set up to successively find the

sequence of points x1 = g(x0) , x2 = g(x1) , x3 = g(x2) , x4 = g(x3) , x5 = g(x4) , etc. and


the solution may be approached to any desired numerical accuracy. We have no business to compute it

ourselves, for it may take a life time for simply finding a root! Translated to the language of

g(x n ) - g(x n -1 )
Calculus, the theorem may be restated by changing  1 to the necessary condition g’(x) < 1
x n  x n 1

instead, for differentiable functions, obviously. The problem regarding how rapidly the

sequence converges is another matter. How similar is this concept to differentiation!

x
For example, take a function g ( x)   5  x , which would be solved by Piccard’s method if we hazard a
6

1
reasonable guess for x0  6.5 say, as g '( x)   1 . But Piccard’s method won’t solve the same equation
6

x  6 x  30 as g '( x)  6  1 in this case. So care should be taken to get g ( x)  x from f  x   0 as it may be

chosen in many ways. Piccard also extended his idea to an existence theorem in differential equations. For a

differential equation the corresponding may be called the fundamental theorem of differential equations.

Example3: Solve x3  x  1  0 in Picard’s method.

Ans. We have to extract a relation g  x   x out of the equation in such a manner that g '  x   1 . Do not

confuse that g '  x   1 identically. This is not so, for, g  x   x is not true identically, but only for the roots of

the equation. Actually y  g  x  and y  x are independent functions and the intersection of their graphs are
the roots of g  x   x  0 , i.e., fixed points of g  x   x . Now, if g '  x   1 always, the two graphs evidently do

not intersect.

1
 
In this particular problem, let f  x   x  x 1  0 , or, x x 2  1  1  0 , i.e., x 
3

x 1
2
 g  x  . Also the criteria

2 x
g ' x  1 is satisfied for all x. As a reasonable guess, note that f 1  1and
1  x2 
2

f  0.5  0.625 1  0.375 , having opposite sign of that of f 1 . So there must be a real root between 0.5

and 1.

1 1
Now starting from x0  1 , we have x1   2  0.5 .
x0  1 1  1
2

1 1 1 1 1
Now x2     0.800 . Note that x3  2   0.610 . In the same way,
x  1  0.5   1 0.25  1
1
2 2
x2  1  0.8 2  1

1 1 1 1
x4    0.729 ., x5  2   0.653 ,
x3  1  0.610   1
2 2
x4  1  0.729 2  1

1 1 1 1 1 1
x6    0.701 , x7  2   0.6705 , x8  2   0.6898 .
x5  1  0.653  1
2 2
x6  1  0.701  1
2
x7  1  0.6705 2  1

In this way, the process approaches further and further to the root, albeit oscillating about the root.
Then what is the need of doing such a long and tedious process? In fact, The process explains the algorithm

given to computer for calculation of the root , rather than doing it manually; and the theory helps in developing

algorithms for computer to calculate the root.

Exercise16 : Solve by Picard’s method of iteration

a) x 2  x Ans. 0.

b) xx Ans. 1.

c) x  cos x Ans. 0.73

Hint : observe from the figure. Take x0  1 as the starting point and use the recursion formula

xn1  cos xn . Observe the points, (1, 0.54), (0.54, 0.54), (0.54, 0.86),(0.86, 0.65) etc. encircle the intersection

point and through which the dashed spiral (the iteration path)closes on the point (0.73909, 0.73909) , the

root of
x  cos x .
d) sin x  x  0.1 Ans. 0.85
x x
e) In the above figure the two st lines y   3 and y  x meet at (4,4), a fixed point of x   3 . But
4 4

written differently, the same equation 4x 12  x is not solved by Piccard’s method ; the sequence of

points diverge away. See the figure below. This is obviously because g '  x   1 . This is both ways

whether we take x0  4 or x0  4 .

f) Solve 2x  cos x  3 by Piccard’s method.


1 
Ans. : Rewrite it as x   cos x  3  g  x  .Since g '  x    1 sin x  1 , we may well start with x0 
2 2 2

1  
. Now x1  g  x0    cos  3   1.5 ,
2 2 

x2  g  x1  
1
2
 cos 1.5   3  1.535 , x3  g  x2    cos 1.535   3  1.518 ,
1
2

x4  g  x3  
1
2
 cos 1.518   3  1.527 , x5  g  x4    cos 1.527   3  1.522 ,
1
2

x6  g  x5  
1
2
 cos 1.522   3  1.524 , x7  g  x6    cos 1.524   3  1.523 ,
1
2

x8  g  x7  
1
2
 cos 1.523  3  1.524 and so on.

Thought for you

Instead of taking g ( x)  x out of f  x   0 and finding where g ( x)  y and y  x meet, we may take an functions

  x     x  extracted from f  x   0 and consider intersection of y    x  and y    x  . Think the situation

when it would trouble and the reason behind.

Thought for you

If an equation has more than one root , Picard’s method may not give all the roots if you
carve out a function g  x   x from the given function f  x   0 . Take an example, x  x gives the root x = 1

, but not the root x = 0. Similarly x 2  x gives the root x = 0

but not the root x = 1, whereas both functions are the fixed point versions of the same function f  x   x  x .
2

Draw the graphs of the three functions and compare with y = x to

figure out why.

Exercise17: Find a root of the equation x3  1  x  x 2 near 2. Ans.: 0.8408

Exercise 18 : Find a root of the equation x 3  5 x  11 . Ans.: 2.953.

x 1
Exercise19 : Find a root of the equation sin x  . Ans.: -5.5174.
x_1

Exercise20 : Find a root of the equation x  e  x . Ans.: 0.6065.

§5.10: Approximate Roots Newton’s method or method of tangents

f  x  0
There is another method due to Newton or Newton-Raphson method. For the smooth function (not

necessarily a polynomial) take a guess x  x0 expecting the real root lies at a distance h from it , we have,

f  x0  h   0 , since x0  h is assumed to be a root.


f '  x0 
f  x0  h   0  f  x0   hf '  x0  by Lagrange’s theorem, i.e., h x0  h is
f  x0 
. But at best an
Now,

f '  x0 
approximation as x0 is only a guess. So we may take x1  x0  h  x0 
f  x0  as a further approximation. In

f '  x1 
x2  x1 
this way we may reiterate
f  x1  as a third approximation and continue the iteration

f '  xn 
xn1  xn  lim
f  xn  in this manner until the limit of the sequence n  xn evolves out.

This is called ‘tangent method’ because we have used the slope of the tangent at x  x0 .; i.e.

f  x0  h   f  x 
f '  x0   for small h. Newton’s method is not free of trouble anyway. If the slope of the
h

graph is too much near the root, initial the guess value should be very close to the actual root and the iteration

may proceed toward other roots if any or diverge away from the target root altogether. Further , if the slope of

the curve is too little near the initial guess value, further iterations would be very close to it as the curve

changes too little, rather than being close to the actual root. If f '  x0   0 the

iteration engine shall cease altogether. However, the interval [a, b] in which a root lies must be an informed

 ab
guess by the criteria f  a  f  b   0 . A plain procedure may be adopted thereafter, f  
 2 
 ab  ab
may be found out and tested for f  a  f    0 . If not, f  b  f    0 is a must, for f  a  are f b 
 2   2 

 ab
of opposite signs. Then f   is a further approximation . the process may be repeated
 2 

many times for desired degree of accuracy. This process itself (called bisection method ) would suffice if

Newton’s method disappoints.

Example4 : Find a root of x3  x 2  x  100 .

Take f  x   x  x  x  100  0 . By trial, f  4  0 and f  5  0 . Hence there is a root between 4 and 5. So


3 2

take the root 4  h1 . Differentiating f '  x   3x  2 x  1 , So f  4   57 and f '  4  16


2

f  a  16
. Since h1     0.3 .so the first approximation of the root is 4+.03 = 4.3. Now f  4.3  2.3 , and
f '  a  57

f '  4.3  65.1 . So the new correction is h2  


2.3
 0.036 . So that the second approximation to the root is
65.1

4.3 – 0.036 = 4.264.

Note: Descartes’ rule of signs says that the equation x3  x 2  x  100 has only one positive root.(prove)

1
Example5 : Find 2 3 by Newton’s method.
Ans. : Let f  x   x  2  0 . So, f '  x   3x . Starting an approximate root of the equation x0  1 ,
3 2

f  x0  13  2 1 4
x1  x0   1  1  .
f '  x0  3.12
3 3

3
4
2
f  x1  4  3  91
Then x2  x1     .
f '  x1  3 4
2
72
3 
3

Exercise21: Give a scheme using Newton-Raphson method to find out cube root of a Natural number.

2 xn3  N
Ans.: xn 1  .
3xn 2

Exercise22: Give a scheme using Newton-Raphson method to find out reciprocal of a natural number.

Ans.: xn1  xn  2  Nxn  .

Example6 : Find 24 by Newton’s method.

Ans.: Starting from x0  5 in the equation f  x   x  24  0 , so that f '  x   2 x .We get


2

f  x0  52  24 f  x1  0.01
x1  x0   5  5  0.1  4.9 . then . x2  x1   4.9   4.899 .
f '  x0  2  5 f '  x1  9.8

Example7 : Find a root of f  x   x  5x  3  0 by Newton’s method.


3
x03  5 x0  3 1 5  3 1
Ans.: We have f '  x   3x  5 . Then start at x0  1 , x1  x0   1  .Now
2

3x0  5
2
35 2

 0.5  5  0.5  3  0.65 .


3
x 3  5x  3
x2  x1  1 2 1  0.5 
3x1  5 3  0.5  5
2

 0.65  5  0.65  3  0.6565 . Similarly x  0.6566 .


3
x 3  5x  3
Then x3  x1  2 2 2  0.65 
3x2  5 3  0.65  5
2 4

Misc. Exercise

Apply Newton’s method to find a root of

23) 2 x3  3x  6  0 between 1 and 2 ans. 1.784

24) x3  2 x  5  0 between 2 and 3 ans. 2.094

25) x5  4  0 between 1 and 2 ans. 1.319

26) x 3  4 x  7  0 between 2 and 3 ans. 2.59

27) x3  6 x  13  0 between 3 and 4 ans. 3.17

28) 4 x3  13x 2  31x  275  0 between 6 and 7 ans. 6.25

29) x3  2 x 2  5  0 between 2 and 3 ans. 2.69

30) x 4  12 x  7  0 between 2 and 3 ans. 2.047

31) x 4  7 x 2  18 x  8  0 between 1 and 2 ans. 0.561

32)
§5.11: Approximate roots by Secant method

We take a guess point A(x0,f(x0)) and another point B(x1,f(x1)) near the root , which is represented by the point

P in the figure, where the curve cuts the X – axis. Now draw the secant AB.

f  x1   f  x0 
The equation of the secant is y  f  x0   . This cuts the X – axis at y  0 , i.e.,
x1  x0

f  x1   f  x0   x  x  f  x0   x  x   x1  x0  f  x0  . It seems we have found


 f  x0    x  x0   1 0
x1  x0 f  x1   f  x0  f  x1   f  x0 
0

out the root; but no; this is at best an approximation; for x0 is an approximation. So we put is

x2  x0 
 x1  x0  f  x0   x  x0 f  x1   x1 f  x0  as a second approximation. This works if
f  x1   f  x0  f  x1   f  x0 
2

f  x0  f  x1   0 , i.e., x2 is a third step approximation. A fourth approximation may be given by taking x3 in

x1 f  x2   x2 f  x1 
place of x2 if still f  x2  f  x0   0 ; so that x3  and the iteration works. If
f  x2   f  x1 

xn 1 f  xn   xn f  xn 1 
still we have f  x3  f  x0   0 ; then successive approximations may be given by xn 1  .if
f  xn   f  xn 1 

f  xn1  f  xn   0 This is also called regula falsi method of method of false position.

Example8 : Find a root of f  x   x  x 1  0 approximately in the above method.


3
Ans. We need not differentiate. Now starting at x0  0.5 and x1  1 , we have

f  0.5    0.5    0.5   1  0.375 and f 1  1  1  1  1 .


3 3

Evidently the root lies between 0.5 and 1, as f  0 f 1  0 ,(negative).

x0 f  x1   x1 f  x0  0.5 f 1  1 f  0.5


Now we have x2    0.64 .Now the root lies between 0.64 and 1; it may be
f  x1   f  x0  f 1  f  0.5

verified from f  0.64 f 1  0 , f  0.64  0.0979 .

x2 f  x0   x0 f  x2  0.64 f 1  1 f  0.64 


Similarly x3    0.672 .Now, it may be verified that f  0.672 f 1  0 , as
f  x0   f  x2  f 1  f  0.64 

f  0.672  0.0245 . Now,

x0 f  x3   x3 f  x0  0.64 f 1  1 f  0.64 


x4    0.672 . Now the root lies between 0.672 and 1 as
f  x3   f  x0  f 1  f  0.64 

f  0.672  0.0245 . Though it seems we have got a little farther off, as it seems, but the next step

would show that actually it is a better approximation.Similarly

x0 f  x4   x4 f  x0  0.672 f 1  1 f  0.672 


x5    0.6822 , still better approximation.
f  x4   f  x0  f 1  f  0.672 

Example9 : Find a real root of f  x   3x  cos x 1  0 in secant method.


Ans. : By trial, we have f  0   2 and f 1  1.45976 . So f 1 f  0  0 and the root sought for lies between

0 and 1. Taking x1  0 and x2  1 ,we get

x1 f  x2   x2 f  x1  0  f  0 2
x3     0.57807 .Evidently the root is positive.
f  x2   f  x1  f 1  f  0  3.45976

Now f  x3   f  0.57807   3 0.57807  cos  0.57807  1  0.10335 ( never be afraid of this minus sign, this

is an approximation, the root is still positive) and f 1  1.45976 ,

f  x3  f  x1    0.103351.45976  0 and the root is therefore between x3 and x1 .Taking this as x4 , we have.

x1 f  x3   x3 f  x1   0.57807 1.45976   0.10335


x4    0.60596 Now
f  x3   f  x1  1.45976  0.10335

f  x4   f  0.60596  3 0.60596  cos  0.60596 1  0.00412 and f 1  1.45976 ,

f  x3  f  x1    0.103351.45976  0 and the root is therefore between x4 and x1 .Taking this as x5 , we have

x1 f  x4   x4 f  x1   0.60596 1.45976   0.00412


. x5    0.607
f  x4   f  x1  1.45976  0.00412
OTHER BOOKS BY THE SAME AUTHOR AT KDPAMAZON.COM

1. Quadratic Equations
2. Ordinary and Partial Differentiation, Taylor’s theorem and
Applications
3. Conic Sections I and II
4. Asymptotes
5. Matrices and Determinants
6. Vectors, straight Lines, Planes and Forces
7. Integration, Central Forces and LCR Circuits
8. Symbolic Logic, Mathematical Logic and Logical Gates
9. Transformations of Coordinates
10.and Lagrange’s Equations
11.Identity, Division and Symmetry

Anda mungkin juga menyukai